■ちょっとした疑問や質問はここに書いてね108■

このエントリーをはてなブックマークに追加
1ご冗談でしょう?名無しさん
■ちょっとした疑問や質問はここに書いてね107■
http://science6.2ch.net/test/read.cgi/sci/1234696461/

質問する前に
教科書や参考書をよく読もう
http://www.google.com/ などの検索サイトを利用し、各自で調べること
宿題を聞くときは、どこまでやってみてどこが分からないのかを書くこと。
丸投げはダメだからね
(丸投げ君は完全無視。答えるだけ無駄。)

質問に対する返答には、何かしらの返答ちょうだいね
★書き込む前に>>2の注意事項を読んでね
★数式の書き方(参考)はこちら>>3
(予備リンク:>>2-10
荒らし厳禁、煽りは黙殺、忘れないうちに定期age
単発質問スレを発見したらこのスレッドへの誘導をよろしくね。

定番FAQ
英語最新版
http://math.ucr.edu/home/baez/physics/
旧版日本語訳
http://research.kek.jp/people/morita/phys-faq/
他にも参考にあるサイトなどあればぜひ。
例)http://ja.wikipedia.org/
※wikipedia内の説明はすべてが正確なわけではありません。
このスレでの受け答えもそうですが。相互に補完しつつ精度を高めましょう。
2ご冗談でしょう?名無しさん:2009/03/19(木) 21:50:04 ID:??? BE:170407692-DIA(113350)
書き込む際の注意

1.)
板の性格上、UNIX端末からの閲覧も多いと推察されます。
機種依存文字(ローマ数字、丸数字等)は避けて下さい。

2.)
以下のような質問に物理板住人は飽き飽きしているので、たぶん無視されます。
しないで下さい。
「相対性理論は間違っています」「量子力学は間違っています」
「宇宙論は間違っています」「シュレディンガーの猫は変です」
「永久機関を作りました」「タイムマシンについて教えて」
「どうして〜?」関連(→「どのようにして〜?」と質問すること)
「なぜ〜?」関連(たいてい、物理学の対象ではないため)

「どうして・なぜ」:http://academy6.2ch.net/philo/
(哲学板・雑談板のほうがむいている場合が多いです。)
雑談は雑談スレで:http://science6.2ch.net/test/read.cgi/sci/1181386663/

3.)
宿題を聞くときは、どこまでやってみてどこが分からないのかを書くこと。
丸投げはダメよ。丸投げに答えるのもダメよ。 丸投げを相手にする人はお馬鹿さん。
せめてポインタを示す程度に留めましょう。

4.)
厨房を放置できない奴も厨房
3ご冗談でしょう?名無しさん:2009/03/19(木) 21:50:37 ID:??? BE:530158278-DIA(113350)
数式の書き方(参考)
●スカラー:a,b,...,z, A,...,Z, α,β,...,ω, Α,Β,...,Ω,...(「ぎりしゃ」「あるふぁ〜おめが」で変換)
●ベクトル:V=[v1,v2,...], |V>>,V↑,vector(V) (混同しないならスカラーの記号でいい。通常は縦ベクトル)
●テンソル:T^[i,j,k...]_[p,q,r,...], T[i,j,k,...;p,q,r,...]  (上下付き1成分表示)
●行列  M[i,j], I[i,j]=δ_[i,j]  M=[[M[1,1],M[2,1],...],[M[1,2],M[2,2],...],...], I=[[1,0,0,...],[0,1,0,...],...]
(右は全成分表示。行または列ごとに表示する。例:M=[[1,-1],[3,2]])
●転置行列・随伴行列:M ',tM, M†("†"は「きごう」で変換可) ●行列式・トレース:|A|=det(A), tr(A)
●複号:a±b("±"は「きごう」で変換可)
●内積・外積・3重積:a・b, a×b, a・(b×c)=(a×b)・c=det([a,b,c]), a×(b×c)
●関数・数列:f(x), f[x] a(n), a[n], a_n
●平方根:√(a+b)=(a+b)^(1/2)=sqrt(a+b) ("√"は「るーと」で変換可)
●指数関数・対数関数:exp(x+y)=e^(x+y) ln(x/2)=log[e](x/2)(exp(x)はeのx乗、lnは自然対数)
●三角比:sin(a), cos(x+y), tan(x/2)
●絶対値:|x|  ●共役複素数:z~ ●ガウス記号:[x] (関数の変数表示と混同しないよう注意)
●階乗:n!=n*(n-1)*(n-2)*...*2*1, n!!=n*(n-2)*(n-4)*...
●順列・組合せ:P[n,k]=nPk, C[n,k]=nCk, Π[n,k]=nΠk, H[n,k]=nHk ("Π"は「ぱい」で変換可)
4ご冗談でしょう?名無しさん:2009/03/19(木) 21:51:11 ID:??? BE:426020459-DIA(113350)
数式の書き方続き(参考)
●微分・偏微分:dy/dx=y', ∂y/∂x=y_x ("∂"は「きごう」で変換可)
●ベクトル微分:∇f=grad(f), ∇・A=div(A),∇xA=rot(A), (∇^2)f=Δf ("∇"は「きごう」,"Δ"は「でるた」で変換可.)
●積分:∫[0,1]f(x)dx=F(x)|_[x=0,1], ∫[y=0,x]f(x,y)dy, ∬[D]f(x,y)dxdy, ∬[C]f(r)dl ("∫"は「いんてぐらる」,"∬"は「きごう」で変換可)
●数列和・数列積:Σ[k=1,n]a(k), Π[k=1,n]a(k) ("Σ"は「しぐま」,"Π"は「ぱい」で変換可)
●極限:lim[x→∞]f(x) ("∞"は「むげんだい」で変換可)
●図形:"△"は「さんかく」 "∠"は「かく」 "⊥"は「すいちょく」 "≡"は「ごうどう」 "∽"は「きごう」
●論理・集合:"⇔⇒∀∃∧∨¬∈∋⊆⊇⊂⊃∪∩"は「きごう」で変換
●等号・不等号:"≠≒<>≦≧≪≫"は「きごう」で変換
5ご冗談でしょう?名無しさん:2009/03/19(木) 22:21:25 ID:???
ラグランジアンで一般座標で極座標を選んだ場合
循環座標の場合一般化運動量保存則はそのまま角運動量保存則になるので
一般座標での一般運動量保存則は運動量保存則と角運動量保存則を含む
ということは言えませんか?
6ご冗談でしょう?名無しさん:2009/03/19(木) 22:56:06 ID:gGMln5jc
言える。
7ご冗談でしょう?名無しさん:2009/03/19(木) 23:21:33 ID:sbFENJpB
家ない。
8ご冗談でしょう?名無しさん:2009/03/20(金) 07:10:32 ID:???
言ってもいいけど、そう言うことにどういう意味が?
9ご冗談でしょう?名無しさん:2009/03/20(金) 08:46:17 ID:???
一般化座標で(化を入れないといけないようですので)循環座標の場合
∂L/∂q=0で d(∂L/∂v)/dt=0 → ∂L/∂v=const
一般化運動量保存則が導かれます。
そのため座標の取り方で運動量保存則、角運動量保存則がとなったりしま
す。ここはちょっとした質問なのでふとした気持で質問しました。
これ以上は「ラグランジアン と ハミルトニアン」すれですね。
10ご冗談でしょう?名無しさん:2009/03/20(金) 21:34:32 ID:???
地球と月で同じ人が垂直跳びをした時の最高点高度と、
月において地球上の最高点高度と同じ高さにするために
人に重しをつけるとして、その重しの重量の出し方を
求めています。
前者だけなら v^2 - v0^2 = -2gy から v=0となる最高点高度では
y=1/2 * v0^2 * 1/g となり、空気抵抗などを無視した場合
月では地球上の6倍飛べることが出せるのですが、
ここに質量mの要素をどのように入れればいいのかがわかりません。
初心者的な質問ですいませんがどなたかご教授願います
11ご冗談でしょう?名無しさん:2009/03/20(金) 21:36:12 ID:???
F = mgh = M(g/6)h
M = 6m
12DCCLXXVI ◆E5JgEsWY1w :2009/03/20(金) 22:19:47 ID:???
質点投射ならすぐに分かると思う。
高飛びの場合、重心位置が問題となるので計算が若干面倒くさい。
無駄にちょっと詳しい計算をしたものが
物理おもしろ問題200選みたいな名前の本に載ってたと思うからそれ参照して。
13ご冗談でしょう?名無しさん:2009/03/20(金) 23:12:58 ID:???
高地にあるグランドで野球をすると球が飛びやすいとかいうのは本当ですか?
14ご冗談でしょう?名無しさん:2009/03/20(金) 23:24:20 ID:???
そりゃそうだろ
あと乾燥してるとか
15ご冗談でしょう?名無しさん:2009/03/20(金) 23:34:23 ID:???
>>14
便乗して聞くけど、気圧とか湿度とかって野球のプレイに有意に影響を及ぼすほどの差になるの?定量的に見て
16ご冗談でしょう?名無しさん:2009/03/20(金) 23:39:52 ID:???
サッカーの世界最弱決定戦を気圧の低いところでやらなかったかな
17ご冗談でしょう?名無しさん:2009/03/21(土) 00:29:56 ID:???
>>15
ドームだとホームランが出やすいというのは聞いたことがある
昔クロガネヒロシが巨人の選手が打つ時バックネットの客が息を吹けばいいとか言ってた
18ご冗談でしょう?名無しさん:2009/03/21(土) 00:39:22 ID:???
>>17
確かに東京ドームはホームランが出やすい球場の1つだけど、
それって気圧差の問題なのか、あるいは風とか球場の設計とか、気圧や湿度とは関係ない問題なのか、分かんないじゃん
気圧や、湿度による粘性の変化が定量的に見てどれだけ影響与えてるか調べないと結論でない問題だと思う
流体力学に詳しい人誰かおせーて
19ご冗談でしょう?名無しさん:2009/03/21(土) 01:05:27 ID:???
>>18
>気圧は明らかに関係あるだろ
20ご冗談でしょう?名無しさん:2009/03/21(土) 08:56:49 ID:???
定量的にはどうかねえ。
21ご冗談でしょう?名無しさん:2009/03/21(土) 09:50:25 ID:nDQUZitn
心理的な問題が大きいかと。
22ご冗談でしょう?名無しさん:2009/03/21(土) 12:34:07 ID:???
1気圧と真空じゃ明らかに違うだろ
23ご冗談でしょう?名無しさん:2009/03/21(土) 12:51:14 ID:???
それは生物学的問題かと。
24ご冗談でしょう?名無しさん:2009/03/21(土) 12:53:53 ID:???
気圧が影響を及ぼすのなら、高地(低気圧)と東京ドーム(高気圧)では逆の効果があるはずでは?
25ご冗談でしょう?名無しさん:2009/03/21(土) 13:03:22 ID:nDQUZitn
>>24
>高地
重力が小さくなる効果も無視できない。
26ご冗談でしょう?名無しさん:2009/03/21(土) 13:25:58 ID:???
地質の影響で重力が大きくなる効果も
27ご冗談でしょう?名無しさん:2009/03/21(土) 13:31:36 ID:???
ドームだとCO2濃度が高いため、温暖化が起こってボールが硬化し、飛距離が上がるとの説もある。
28ご冗談でしょう?名無しさん:2009/03/21(土) 14:35:24 ID:???
重力の影響なら高地(低重力)と東京ドーム(普通重力)で差が出るはずでは?
29ご冗談でしょう?名無しさん:2009/03/21(土) 15:52:06 ID:dFKvuTw/
空気が薄くなるから空気抵抗が減るな。
同じ初速なら遠くまで飛ぶ。
バットの空気抵抗も減って速く振れるかも。
30ご冗談でしょう?名無しさん:2009/03/21(土) 15:54:37 ID:???
酸素濃度が減るからバットの速度が遅くなる。
31ご冗談でしょう?名無しさん:2009/03/21(土) 16:07:27 ID:???
単に球場が小さいんじゃね
32ご冗談でしょう?名無しさん:2009/03/21(土) 16:14:18 ID:???
>>29
そりゃ単純に考えればそうだけどさ、
空気抵抗が減った結果として「3cm程度余分に飛ぶ」って結論なら、プレーにほとんど影響ないじゃん
だから定量的な評価が必要だと思うんだけど
33ご冗談でしょう?名無しさん:2009/03/21(土) 16:52:42 ID:???
>>18
グダグダ考える前に球場の形調べたら?
東京ドームは単に右中間左中間までの距離が近いから
ボールが予想以上に飛んでるように見えるだけです。
34ご冗談でしょう?名無しさん:2009/03/21(土) 18:11:58 ID:???
よく考えたらドームの方が気圧が高いな
謎だ
35ご冗談でしょう?名無しさん:2009/03/21(土) 18:23:47 ID:???
ここまでのレスを精査すると、東京ドームは単に狭いからホームランが出やすいってだけかもしれない。
で、ホームランの出やすさに関係しそうな物理的パラメータは
1, 気圧(つまり空気の粘性)
2. 重力加速度
3. グランドの広さ
の三つに絞られそう。
この三つパラメータと、ホームラン数の統計的相関を解析すれば立派な物理の論文になる。
この三つのパラメータに関する各グランドと試合結果のデータはすでに公開されているので、
誰でもすぐに物理的に意味のある論文が書ける。
俺はめんどくさいからやらないけど、暇なヤツは分析してみると面白い結果が得られるかもしれないぞ。
36ご冗談でしょう?名無しさん:2009/03/21(土) 18:44:08 ID:???
37ご冗談でしょう?名無しさん:2009/03/21(土) 18:53:32 ID:???
>>36
なるほどねw
物理的な分析では有意な差が出ないかもしれないけど、
経済的な分析をすれば結果を説明できるかもしれないというのは十分にありえる。
その観点も包含して、数理的に統計分析すればますます面白い論文が書ける予感。
38ご冗談でしょう?名無しさん:2009/03/21(土) 19:32:06 ID:???
>>35
やめとけ。NPBのボールは規格の範囲内で
反発係数の中央値を設定して製造されているけど
球場ごとに必ずしも一定とは言えず、それが年度ごとに変わりうる。

球場ごとの成績の偏りはパークファクターという指標でまとめられるが
年度ごとの条件ばらつきやチームの調子、戦術によるばらつきも大きく、
物理的メカニズムに立ち入った精度よい評価ができるとは思えない
ボールを捉えるという意味では球場背景の色や天候による視覚効果も入ってくるしな。

要素分解して考察するには環境を絞った実験でもするしかないと思う。
39ご冗談でしょう?名無しさん:2009/03/22(日) 01:09:38 ID:???
潮汐の原因は潮汐力で、潮汐力は天体の位置エネルギーから供給されているんですよね。
海水の潮汐は摩擦などによって熱エネルギーになってしまうので、天体の位置エネルギーが
どんどん減っていってしまうということになりますよね。
ということは、いずれ月が地球に落ちてきてしまうということになりませんか。
40ご冗談でしょう?名無しさん:2009/03/22(日) 01:11:22 ID:???
あと、重力波の輻射によるエネルギー損失もあるし、まあ理論上いつかはね

その時がくるまで太陽系がそのままである保証はないが
41ご冗談でしょう?名無しさん:2009/03/22(日) 01:15:54 ID:???
なりません。エネルギーより角運動量保存則を先に考えましょう
42ご冗談でしょう?名無しさん:2009/03/22(日) 01:18:42 ID:???
重力波って運動量持つだろ?
43ご冗談でしょう?名無しさん:2009/03/22(日) 01:39:30 ID:1ZvoSGIr
箸で茶碗を叩いたら音が鳴るよな。でも、同じ力で接触した状態から押したら音がしないのはなんでだ?



茶碗の振動が空気を振動さしてることだったり、撃力が関係しそうなのは何となくわかる。

馬鹿でもわかるように詳しく教えて。
44ご冗談でしょう?名無しさん:2009/03/22(日) 01:49:21 ID:???
>>39
実際には、月は1年に3.8cmずつ地球から離れていっている。
理由は、>>41で指摘されている通り。

地球の自転周期と月の公転周期が一致して潮汐の摩擦が
発生しなくなるとこの傾向は止まるわけだが、
今度は太陽による潮汐により、月が地球に近づいてくる。

まあ、そうなる前に地球と月は赤色巨星になった太陽に
飲み込まれるだろうけどね。
45ご冗談でしょう?名無しさん:2009/03/22(日) 02:10:48 ID:???
言葉がおかしいかもしれんが、この世の物理法則に「上限」はあるのか?

1、「上限」があるとして現在の物理学はその何%くらいまで解明できてるんだ?
2、 遠い将来、上限の99%まで解明でき、それを技術としてあますことなく社会に反映できた場合、世の中はどうなっているか?

皆さんの意見をきかせて
46ご冗談でしょう?名無しさん:2009/03/22(日) 02:29:48 ID:???
「上限」の意味が分からん
説明しろクズ
47ご冗談でしょう?名無しさん:2009/03/22(日) 03:15:47 ID:NCNHjk04
量子力学で調和振動や水素原子などの複雑な微分方程式の答えが
その方程式の漸近解と級数の積になるのはなぜだ?
偶然?
48ご冗談でしょう?名無しさん:2009/03/22(日) 03:25:26 ID:KhH6oBn9
マーチレベルの私大理学部物理学科を卒業して一流国立大大学院で哲学を専攻するのはありですか?
49ご冗談でしょう?名無しさん:2009/03/22(日) 13:23:02 ID:???
何故に理系を出て文系の大学院に・・・もちろん、ありだろう。
自分の人生なんだから、行きたければ好きにすればいい。

知り合いが行くと言ったら、とりあえず説得するが。
50ご冗談でしょう?名無しさん:2009/03/22(日) 14:15:30 ID:MhJC4Iyx
量子力学の文脈で、"dispersion-free state" って決まった意味ありますか?引用↓

In order to avoid such a mixture in single states, refining the notion of 'state', one can consider dispersion-free states in which physical quantities take definite values.

最初pure stateかなと思ったんですが、definite value をとるということは、eigenstate?
51ご冗談でしょう?名無しさん:2009/03/22(日) 15:49:35 ID:???
>>40-42, >>44
回答ありがとうございます。
太陽が赤色巨星になる方が先のようで安心しました。
52ご冗談でしょう?名無しさん:2009/03/22(日) 15:51:29 ID:???
>>51
>太陽が赤色巨星になる方が先のようで安心しました。
君が死ぬ方が先だ。安心しろ。
53ご冗談でしょう?名無しさん:2009/03/22(日) 18:14:58 ID:IStVr/rk
54ご冗談でしょう?名無しさん:2009/03/22(日) 18:16:16 ID:???
間違ってます。
55ご冗談でしょう?名無しさん:2009/03/22(日) 18:41:41 ID:Oa5nBS94
わざとだろ。ありえねえ。
56ご冗談でしょう?名無しさん:2009/03/22(日) 19:05:00 ID:XIw5ImO5
大きさを示したいんじゃないの。
57ご冗談でしょう?名無しさん:2009/03/22(日) 19:26:30 ID:???
イメージとしてはいいんじゃないの?
58ご冗談でしょう?名無しさん:2009/03/22(日) 19:47:16 ID:???
クォークだろ。
59ご冗談でしょう?名無しさん:2009/03/22(日) 19:49:08 ID:???
いいよ
60ご冗談でしょう?名無しさん:2009/03/22(日) 19:58:12 ID:xSBPvEKr
相対論の固有質量と重力質量の違いについて教えて下さい。
佐藤勝彦さんの本によれば、固有質量は物体固有の全エネルギーであり、
一方重力質量は物体の外から計った重力の強さから定義される質量であり、
前者は後者より大きいとの事ですが、何故その様になるのか分かりません。
一般相対論によればエネルギーも重力の強さに寄与する訳ですから、
重力を元に測った質量も、固有質量に一致するのではないでしょうか?
宜しくお願いします。
61ご冗談でしょう?名無しさん:2009/03/22(日) 19:58:34 ID:???
ニュートリノもクォークの仲間だろ。電荷があるないかの違いだけ。
62ご冗談でしょう?名無しさん:2009/03/22(日) 20:05:58 ID:???
全然違います
63ご冗談でしょう?名無しさん:2009/03/22(日) 20:12:07 ID:???
>>53
原子核の構成要素という意味では、クォークが適切。

>>61
仲間分けの度合いによる。
64ご冗談でしょう?名無しさん:2009/03/22(日) 20:37:09 ID:???
水素とネオンを仲間といっていいかどうかとか
まぁそういう話になっちゃうよな
65ご冗談でしょう?名無しさん:2009/03/22(日) 20:42:49 ID:yzvCA5VH
すいません、素人の質問です。
ある科学雑誌で大きな質量を持った物体が回転すると周りの空間が引きずられて、
回転するブラックホールの周りには引きずられすぎた結果ブラックホールの自転方向とは
逆向きに進めないエルゴ領域が出来る。最近地球の自転によっても空間が引きずられて
いることが確認された、と言うのがあったのですが、重力ではなく電荷などでは同じ
効果は無いのでしょうか。

例えばバリバリに帯電した物体を思いっきり回転させると磁場は出来ますが周りの空間は
引きずられたりしないのでしょうか。実験的には重力よりも容易に確認できそうですが、
そういった実験例は検索しても見あたりませんでした。

引きずられないとしたらなぜ重力だけが空間を引きずることが出来るのでしょうか。
もし引きずられるとしたら電荷を持ったブラックホールの場合、いわゆるエルゴ領域とは
別に電界の変化が自転方向とは逆に伝わらない第二の領域が出来そうですがそういう話も
きいたことがありません。
6665:2009/03/22(日) 20:44:40 ID:???
あーごめんなさい。sage忘れました。
67ご冗談でしょう?名無しさん:2009/03/22(日) 22:17:06 ID:???
>>65
電荷で空間が引きずられるとは、どういう現象をさすの?
68ご冗談でしょう?名無しさん:2009/03/22(日) 23:09:58 ID:???
>>53
禿ワロスw

ニュートロンとニュートリノを混同してるw
69ご冗談でしょう?名無しさん:2009/03/22(日) 23:15:04 ID:???
原子核はニュートリノから出来ていたのか。新しい学説だな。
70ご冗談でしょう?名無しさん:2009/03/22(日) 23:16:48 ID:???
>>60
その本に各々の正確な定義か具体例は書いていないのか?
71ご冗談でしょう?名無しさん:2009/03/22(日) 23:18:06 ID:???
>>68
>ニュートロンとニュートリノを混同してるw
Pauliは、最初 neutron と呼んだのだから間違いじゃない。
http://www.symmetrymagazine.org/cms/?pid=1000450
72ご冗談でしょう?名無しさん:2009/03/22(日) 23:22:20 ID:???
>>71
ニュートロンをニュートリノと呼んだことはないだろ
73ご冗談でしょう?名無しさん:2009/03/22(日) 23:39:31 ID:???
>>65
重力は時空のゆがみに起因する力なので、電磁気力とは違う。
74ご冗談でしょう?名無しさん:2009/03/22(日) 23:54:40 ID:???
>>73
>電磁気力とは違う。
なぜですか?
75ご冗談でしょう?名無しさん:2009/03/23(月) 00:05:48 ID:???
定義
76ご冗談でしょう?名無しさん:2009/03/23(月) 00:25:13 ID:???
>>74
物理学は「なぜですか?」という質問には答えません。

重力は時空のゆがみに起因する力であるのに対し、
電磁気力は時空のゆがみに起因する力ではなかった。
調べてみたら実際にそういう違いがあった。それだけ。
77ご冗談でしょう?名無しさん:2009/03/23(月) 00:29:34 ID:???
>>74
それを解くのがお前の仕事
78ご冗談でしょう?名無しさん:2009/03/23(月) 00:43:31 ID:???
>>76
>物理学は「なぜですか?」という質問には答えません。
もし本当なら、無力ですね。
79ご冗談でしょう?名無しさん:2009/03/23(月) 00:56:15 ID:???
テンプレぐらい読めカス
8065:2009/03/23(月) 00:57:29 ID:+h6UfSBF
>>67
残念ながら「重力で空間が引きずられる」ということの本質的な意味が
分かっていないので何とも言えません。素人向けの解説なのでかなり
語弊がある表現だろうとは思います。
重力場も電場も空間のゆがみだと思っていたのですが違うんですね。もう少し
その辺りを調べてみます。聞いた話によると重力も光速で伝わるそうなので
空間の同じ性質を別の見方をしているだけなのかと思っていたのですが誤解の
ようです。
81ご冗談でしょう?名無しさん:2009/03/23(月) 18:52:38 ID:uCJXHcNu
>>78
何故かなんて問いは意味が無い。
82ご冗談でしょう?名無しさん:2009/03/23(月) 18:57:14 ID:???
>>78
「なぜですか?」という問いに対しては無力ですな。それが何か?
83ご冗談でしょう?名無しさん:2009/03/23(月) 19:58:53 ID:???
もっとすごい夢のある世界だと恋いこがれていたから醒めたんでしょ。
いいじゃん、それで。ムキになるなよ、お前ら。
84ご冗談でしょう?名無しさん:2009/03/23(月) 20:05:40 ID:???
しかし、「なぜ?」に意味はないは言い過ぎだろう
どこかで理由なしに認める原理は必要だが、できればその数が減るのが望ましい
Keplerの法則が成立するのがなぜかは、Newton力学と逆二乗力を仮定すれば説明できるわけだし

あと、>>74の問いは、なぜ(そう考えるの)ですか?とも読める
それなら十分に意味がある質問
85ご冗談でしょう?名無しさん:2009/03/23(月) 20:23:47 ID:???
>その数が減るのが望ましい

>Keplerの法則が成立するのがなぜかは、Newton力学と逆二乗力を仮定すれば説明できるわけだし
増えてますがなw
86ご冗談でしょう?名無しさん:2009/03/23(月) 21:45:12 ID:???
>>85
じゃあ最小作用の法則と重力のLagrangianから、
Keplerの3法則と地上の落体の法則とCoriolis力と……が説明できるっていえばいい?

そんな揚げ足とっても、ねえ。
87ご冗談でしょう?名無しさん:2009/03/23(月) 22:08:41 ID:???
いずれにしろhowでしかない
88ご冗談でしょう?名無しさん:2009/03/23(月) 22:19:52 ID:???
【量子力学崩壊】有り得ない「マイナス1個の光子」観測…阪大大学院生らが成功
http://tsushima.2ch.net/test/read.cgi/news/1237783305/



マイナス1個の光子の意味についてご教示願います
89ご冗談でしょう?名無しさん:2009/03/23(月) 22:22:22 ID:???
単に「意味がある」の意味が人によって違うだけだろ
90ご冗談でしょう?名無しさん:2009/03/23(月) 22:30:04 ID:???
>>87
howとwhyの違いは?
あと、「なぜ」はwhyと一対一対応するわけでもないけど
91ご冗談でしょう?名無しさん:2009/03/23(月) 22:31:35 ID:JHeoC6g5
>>88
293 名前: 家事手伝い(九州)[] 投稿日:2009/03/23(月) 15:26:58.88 ID:GOkJsq3w
スッゲー適当に解説すると
消えた光子は異次元に飛んでったかも知れないので
二次元にいける可能性が出てきた
92ご冗談でしょう?名無しさん:2009/03/23(月) 22:35:35 ID:???
>>91
異次元ならば我々が存在する三次元よりも高次元であるのでは・・?
93ご冗談でしょう?名無しさん:2009/03/23(月) 22:38:17 ID:???
理由なしに認める原理が必要なら別の「なぜ」に置き換わっただけで
結局は「なぜ」に答えていない。別に科学はそれに答えるものでは
ないのだから、答えていないと指摘したところでムキになって
反論する必要はこれっぽっちもないはずなのだが。
94ご冗談でしょう?名無しさん:2009/03/23(月) 22:50:24 ID:???
「なぜに答える」をそんなにストイックに定義したら、科学に限らず「なぜに答える」ことは原理的に不可能じゃないの?
別の「なぜ」に置き換わるのを「なぜに一段階答えた」とみなしても十分だと思うんだけどな。
95ご冗談でしょう?名無しさん:2009/03/23(月) 23:11:13 ID:???
>>2 2)読め
96ご冗談でしょう?名無しさん:2009/03/23(月) 23:18:14 ID:???
>>88
個数と相との不確定性関係では?
97ご冗談でしょう?名無しさん:2009/03/23(月) 23:23:21 ID:hvj+ftFu
物理学じゃなかったらすいません
太陽が雲の上にあって雲から光の筋が出ているとき、なぜ筋は全て地上に垂直に降り注がずに放射状に降りそそいでいるのかどなたか分かり易くご教授下さい。
98ご冗談でしょう?名無しさん:2009/03/23(月) 23:26:34 ID:???
>>95
あのテンプレは不適切だと思ってるけど。
俺は、「なぜ」は基本的に物理で説明されるべき質問だと思ってるよ。
99ご冗談でしょう?名無しさん:2009/03/23(月) 23:27:36 ID:???
100ご冗談でしょう?名無しさん:2009/03/23(月) 23:42:36 ID:???
物理学は微妙な違いだが
「なぜ」を問うのではなくて「どうして」を問う学問なんじゃないか?
101ご冗談でしょう?名無しさん:2009/03/23(月) 23:44:57 ID:???
何故そう言えるの?
102ご冗談でしょう?名無しさん:2009/03/24(火) 00:45:38 ID:???
>>98
じゃあ、なぜ宇宙は存在するの?
103ご冗談でしょう?名無しさん:2009/03/24(火) 01:21:27 ID:???
>>90
>howとwhyの違いは?
辞書を引け。煽りでなくマジで。

>あと、「なぜ」はwhyと一対一対応するわけでもないけど
確かに、「なぜ」という質問の全てが物理学の対象でないわけじゃない。
だが、>>74の「なぜ」は物理学が答えない類の質問であることは確かだろ。
みんなは各論としてそれを指摘しているだけじゃないのか?
一般論として扱うから話がややこしくなるのでは?
104ご冗談でしょう?名無しさん:2009/03/24(火) 02:51:46 ID:???
>>103
違うって言えるのは、既存の理論に還元不可能な問いだと既に知ってるからだろう。
こういうところで質問するようなやつが「なぜ電磁力と重力の起因は異なるのか」と「なぜ電気力と磁力の起因は同じなのか」の質問に区別ができるとは到底思えない。
105ご冗談でしょう?名無しさん:2009/03/24(火) 08:52:20 ID:???
だから既に知っている人が知らない人に「違う」と教えているんでしょ。
質問スレとして典型的なやり取りでしかない。
何が問題視されているのかさっぱりわからん
106ご冗談でしょう?名無しさん:2009/03/24(火) 09:47:35 ID:???
>>76は後半を好意的に読めばともかく、>>81とか>>82を見る限り「なぜか」という問いを全否定しているようにしか読めないわけで。
なぜ>>74の質問が意味がないのか、どういう「なぜ」なら許容されるのか書かないとミスリーディングでしょ。
それは質問に対する回答としては不適切で、問題視されて当然と思うけど。

>>77の回答で質問者が理解できるかは別として、あれは一番本質を突いてると思うよ。
107ご冗談でしょう?名無しさん:2009/03/24(火) 10:58:37 ID:???
なぜなぜはだめ? じゃ、そうして?どうして?は?
108ご冗談でしょう?名無しさん:2009/03/24(火) 11:50:06 ID:???
109ご冗談でしょう?名無しさん:2009/03/24(火) 12:28:16 ID:???
>>108
カルトは消えな。
110ご冗談でしょう?名無しさん:2009/03/24(火) 13:09:47 ID:???
>>106
実際物理では「なぜか」という問いには結局のところ答えられないというのは
厳然たる事実であって否定とか肯定とかそういう次元の話ではない。

念のため言っておくが「なぜ」を別の「なぜ」に置き換えることを否定なんぞ
していないぞ。それはそれで価値がある、というかそれこそが科学の真価。

「なぜ」には結局のところ答えられない、としか言ってないのに、そちらを
否定されたと勘違いして噛みついているのではないか?
111ご冗談でしょう?名無しさん:2009/03/24(火) 13:28:58 ID:???
>>110
>実際物理では「なぜか」という問いには結局のところ答えられない
いつから、こんなアホな言説がまかりとおるようになったの?
112ご冗談でしょう?名無しさん:2009/03/24(火) 13:34:43 ID:???
>>111
お前オカルト板行けよ
113ご冗談でしょう?名無しさん:2009/03/24(火) 14:14:23 ID:???
>>111
理由なしに認める原理を必要としない物理法則を挙げてみ?
114ご冗談でしょう?名無しさん:2009/03/24(火) 14:24:28 ID:???
イチローのせいでニュー速が落ちた件
115ご冗談でしょう?名無しさん:2009/03/24(火) 14:49:12 ID:???
おまいら、冷静になれ。
なぜか?という問いこそ、
科学の原点だろ。
116ご冗談でしょう?名無しさん:2009/03/24(火) 15:01:07 ID:???
イチロー効果でかすぎだろ
ニュー速以外もいくつか落ちてんぞ……
117ご冗談でしょう?名無しさん:2009/03/24(火) 20:25:38 ID:???



          なぜ? の嵐


118ご冗談でしょう?名無しさん:2009/03/24(火) 20:53:16 ID:???
なぜ?と思わないアホには科学は無理だ。
119ご冗談でしょう?名無しさん:2009/03/24(火) 21:01:59 ID:???
green関数が意味不明すぎます。本当にありがとうございました。
120ご冗談でしょう?名無しさん:2009/03/24(火) 22:12:41 ID:FqB9QHLQ
数と位相の間に不確定性関係があるということは
位相を決定すると数が不定になるってことでしょうか?
不定になるということとマイナスになることの間には飛躍があると思うのですが。。。
121ご冗談でしょう?名無しさん:2009/03/24(火) 22:14:14 ID:???
>>119
つ 重ね合わせの原理
電荷密度分布が与えられた時に電位を求めるやり方思い出せばおk
122ご冗談でしょう?名無しさん:2009/03/24(火) 23:30:26 ID:???
>>115
原点ではあるが終点ではない
123ご冗談でしょう?名無しさん:2009/03/25(水) 00:03:27 ID:???
>>122
終点はどこじゃ?
124ご冗談でしょう?名無しさん:2009/03/25(水) 00:26:10 ID:Cu8tzU21
いま思いついた
地球に無理矢理直線の穴をあけて
そん中に物落としたら向こうに届くかな?
125ご冗談でしょう?名無しさん:2009/03/25(水) 00:33:33 ID:???
何の抵抗も無ければな。
126ご冗談でしょう?名無しさん:2009/03/25(水) 00:39:33 ID:???
>>124
俺は30年前に思い付いてた
127ご冗談でしょう?名無しさん:2009/03/25(水) 00:42:43 ID:Cu8tzU21
抵抗?
128ご冗談でしょう?名無しさん:2009/03/25(水) 00:46:59 ID:???
空気抵抗
129ご冗談でしょう?名無しさん:2009/03/25(水) 00:48:08 ID:Cu8tzU21
それで向こうまで届かないとどうなりますか?
130ご冗談でしょう?名無しさん:2009/03/25(水) 00:54:17 ID:???
>>123
終点などない。あると思っているなら科学の方法論を勉強しる。
「なぜ」を別の「なぜ」に置き換える作業が延々と続く。だがそれがいい。
131ご冗談でしょう?名無しさん:2009/03/25(水) 00:56:23 ID:???
>>129
振動しながら地球の中心に落ち着く。
地球中心のものすごい気圧と高熱に耐えられるなら、だが
132ご冗談でしょう?名無しさん:2009/03/25(水) 01:12:22 ID:aeCgn/P1

鉛直線偏差により途中で穴の側面にぶつかる
133ご冗談でしょう?名無しさん:2009/03/25(水) 01:20:22 ID:???
>>126
我輩は10万年前に思い付いてた
134ご冗談でしょう?名無しさん:2009/03/25(水) 02:02:59 ID:???
>>130
集積点はありうるだろ、終点はないにしても
135ご冗談でしょう?名無しさん:2009/03/25(水) 08:36:45 ID:Ne+ujLjP
高次元を数学で解く数学のやりやり方が過去スレに載っているよ。
136ご冗談でしょう?名無しさん:2009/03/25(水) 08:58:01 ID:???
>>134
>集積点
ゴミ論文のたまり場ですか?
137ご冗談でしょう?名無しさん:2009/03/25(水) 10:32:08 ID:???
>>134
そこが集積点であると確認する手段がないので
その問いに科学的意味はない
138ご冗談でしょう?名無しさん:2009/03/25(水) 11:22:25 ID:/mhzB2On
139ご冗談でしょう?名無しさん:2009/03/25(水) 12:30:12 ID:???
ここがゴミ集積所か
140ご冗談でしょう?名無しさん:2009/03/25(水) 13:47:13 ID:???
おかしいと思います!
電車が動いてるときに虫が普通に飛んでられるのはおかしいと思います!
昔、僕が蚊を捕まえてペットボトルに入れて拷問したとき、ペットボトルを振ったんですが普通にダメージを受けてました。
だから電車の中でもずっと壁に押しつけられる状態ならないとおかしいと思います!




おかしいと思います!
141ご冗談でしょう?名無しさん:2009/03/25(水) 14:21:26 ID:???
ペットボトルと同じくらい激しく電車を振れば、壁にぶつかりまくるでしょう。
142ご冗談でしょう?名無しさん:2009/03/25(水) 14:29:48 ID:s0UcEHU2
>>140
君は現実より君の考えのほうがが正しいと思っているの?
143ご冗談でしょう?名無しさん:2009/03/25(水) 14:33:27 ID:???
>>140
電車くらいの大きさのペットボトルを電車くらいの加速度で振り回してみ
144ご冗談でしょう?名無しさん:2009/03/25(水) 15:13:25 ID:???
12cmくらいの棒なら毎日振り振りしてますが
145ご冗談でしょう?名無しさん:2009/03/25(水) 21:14:09 ID:???
>>132
鉛直線偏差以前にコリオリの力で直接的にぶつかりますがな・・・
146ご冗談でしょう?名無しさん:2009/03/25(水) 21:46:00 ID:???
昼間、太陽だけでなく空全体が明るく見えるのはなぜですか?
空気が光を曲げているのですか?
147ご冗談でしょう?名無しさん:2009/03/25(水) 22:07:17 ID:???
そうです。
148ご冗談でしょう?名無しさん:2009/03/25(水) 22:13:15 ID:???
>>146
「レイリー散乱」でぐぐる
149146:2009/03/25(水) 22:21:38 ID:???
ありがとうございます
150ご冗談でしょう?名無しさん:2009/03/25(水) 23:14:27 ID:???
>>131
気圧も高くなるの?
151ご冗談でしょう?名無しさん:2009/03/25(水) 23:18:33 ID:???
中心は無重力だから真空になるのでは?
152ご冗談でしょう?名無しさん:2009/03/25(水) 23:24:32 ID:???
無重力なところにものすごい圧力がかかってるんだよ
153ご冗談でしょう?名無しさん:2009/03/25(水) 23:27:33 ID:???
>>151
> 無重力だから真空

意味不明
154ご冗談でしょう?名無しさん:2009/03/25(水) 23:34:53 ID:???
俺も小学生の頃、宇宙は無重力で真空だから、
宇宙船の中は宇宙服を脱げないと思ってた。
155ご冗談でしょう?名無しさん:2009/03/26(木) 00:00:58 ID:???
無限に長い円柱の表面に電荷が一様に分布している場合
円柱の表面の静電ポテンシャルって0になりますか?
156ご冗談でしょう?名無しさん:2009/03/26(木) 00:10:35 ID:???
>>155
0にしてもいい
157ご冗談でしょう?名無しさん:2009/03/26(木) 00:17:06 ID:???
>>156
ありがとうございます。
それはどうして言えることなのですか?
158ご冗談でしょう?名無しさん:2009/03/26(木) 00:20:54 ID:???
>>157
お前の使っている教科書には「電位の0点は任意に定めることができる」
って書いてないか??
159ご冗談でしょう?名無しさん:2009/03/26(木) 00:26:32 ID:???
>>158
>「電位の0点は任意に定めることができる」
なぜですか?
160ご冗談でしょう?名無しさん:2009/03/26(木) 00:41:41 ID:???
>>158
電位の定義。

エネルギーにも同じ事が言える。こう言われてピンとこないなら、力学からやり直し。
161ご冗談でしょう?名無しさん:2009/03/26(木) 00:43:30 ID:???
>>160
電位の定義は電位の0点によるってこと?
162ご冗談でしょう?名無しさん:2009/03/26(木) 00:44:20 ID:???
・・・次もう一回「なぜですか?」が来たら「教科書嫁」で終わりだな・・・
163ご冗談でしょう?名無しさん:2009/03/26(木) 00:46:14 ID:???
>>162
>教科書嫁
おまい、寂しい人生だな。
164ご冗談でしょう?名無しさん:2009/03/26(木) 00:46:27 ID:???
>>161
電位をどんなふうに定義したかがわかっている人間ならどうして0点を
任意に定められるか、なんてことは聞くまでもなく自明だよ、と言って
るんだよ。つまりお前は電位の定義をどうやったかからやり直せ。

165ご冗談でしょう?名無しさん:2009/03/26(木) 00:46:34 ID:???
教科書には定数を決めるときは、適当な場所における値を決める事により決定する

と書いてありました。
でわこの場合無限遠における静電ポテンシャルを0にして考えてもいいのですか?
大学では無限遠を0にするって習って来たので、表面を0にするという発想が受け入れに難くて・・・
166ご冗談でしょう?名無しさん:2009/03/26(木) 00:48:13 ID:???
だから、電位はどう定義されたのか、からやり直せよ。そこがわかっていたら
「どこを0にするかで悩む」なんて発想こそ、受け入れがたくなるはずだ。
167ご冗談でしょう?名無しさん:2009/03/26(木) 00:52:44 ID:???
E(x)=-gradΦ(x)
と書いたときのΦ(x)を静電ポテンシャルという、とあります。

これから0にする点を決める事ってできますか?
168ご冗談でしょう?名無しさん:2009/03/26(木) 00:54:02 ID:???
>>167
さっきから「0にする点は任意」って書いてないか??
「任意」ってことは逆に言えば「どこを0にするかも決められない」ってことだ
と俺は思うがな。
169ご冗談でしょう?名無しさん:2009/03/26(木) 00:55:23 ID:???
>>168
決められないってことは、ポテンシャルの値そのものには意味がないってこと?
170ご冗談でしょう?名無しさん:2009/03/26(木) 00:55:51 ID:???
>>167
その、

E(x)=-gradΦ(x)

にしてから、Φ(x)→Φ(x)+A  (Aは定数)

みたいに変えてもE(x)は変わらないから、どこがΦ(x)=0になる点か
決まらないよ。

171ご冗談でしょう?名無しさん:2009/03/26(木) 00:56:39 ID:???
>>165
>>155 の問題なら、無限遠の電位を0とはしない方がいい。
172ご冗談でしょう?名無しさん:2009/03/26(木) 00:56:42 ID:???
>>169
教科書読め。たいていの教科書には、「ポテンシャルの値そのものには意味がない」
って書いてあるから。
173ご冗談でしょう?名無しさん:2009/03/26(木) 00:59:50 ID:???
>>172
>「ポテンシャルの値そのものには意味がない」
じゃ、ポテンシャルってなんですか?
 E(x)=-gradΦ(x)
をみたす架空の関数で、考えると便利?な数学的道具ですか?
174ご冗談でしょう?名無しさん:2009/03/26(木) 01:02:16 ID:???
>>173
 値そのものに意味がないから架空の関数だとか言うんなら、エネルギーだって
「考えると便利な数学的道具」でしかないな。

 力学の教科書からやり直したら??
 それでまだエネルギーが架空の関数で数学的道具にしか思えないなら、
そういうもんだと思って使え。便利だから。


175ご冗談でしょう?名無しさん:2009/03/26(木) 01:02:33 ID:???
>>170
なるほど。。

>>171
そうなって来ると混乱するのです。。
点電荷の場合では無限遠での電位を0とする事になってますし。。
176ご冗談でしょう?名無しさん:2009/03/26(木) 01:05:01 ID:???
「蚊とペットボトルと電車」について答えてくれた人ありがとう!
なんで加速度によって壁にあたったり大丈夫だったりするんですか?
なんなんですか?
慣性の法則はどうしたんですか?
空気による摩擦みたいなものがあるんですか?
蚊×9.8×空気摩擦<加速度で壁にぶつかるってことですか?
どうですか?いい線行ってるんじゃないですか?
177ご冗談でしょう?名無しさん:2009/03/26(木) 01:06:01 ID:???
>>174
>エネルギーだって
>「考えると便利な数学的道具」でしかないな。
なるほど、エネルギーは見えないのでよく分からなかったのですが、
すこし分かった気がします。
178ご冗談でしょう?名無しさん:2009/03/26(木) 01:51:58 ID:???
>>176
何言ってんだかさっぱりわからんから、とりあえず落ち着け。
179ご冗談でしょう?名無しさん:2009/03/26(木) 11:21:17 ID:UvGBR8UU
教科書読め 解らなかったら、参考書読め!!
180ご冗談でしょう?名無しさん:2009/03/26(木) 12:23:14 ID:???
>>175
>点電荷の場合では無限遠での電位を0とする事になってますし。。
便利だからそうすることが多いだけ。そうしなきゃいけないというわけではない
181ご冗談でしょう?名無しさん:2009/03/26(木) 14:40:13 ID:???
>>180
そうですかー
155の場合は-logが出てくるから見栄えよくするためにするみたいな感じでしょうか?

では、無限に長い直線電流の場合でもr=0での電位を0にする
でよろしいですか?
182ご冗談でしょう?名無しさん:2009/03/26(木) 19:06:33 ID:???
>>181
エネルギーの原点は自由に決めれるんだから好きにしろよ。
勿論、原点を±∞にすると意味がなくなるからそれ以外でな。
183ご冗談でしょう?名無しさん:2009/03/26(木) 19:25:16 ID:???


なぁ、エキスパートな回答者たち。なんで物理なんか好きになったん?
「理科」は好きだった。後の生物、化学もよかった。
でも物理は数学で理科じゃないと思った。

184ご冗談でしょう?名無しさん:2009/03/26(木) 20:34:11 ID:oEVZuWVX
似たような質問が過去にもあるような気がするけど質問します。

高校物理の最初で習う力学で
 運動エネルギーが保存される=反発係数が1
というのがあるじゃないですか。

運動量保存則 & 運動エネルギー保存則
から反発係数が1を美しく導き出す方法があったら
教えてください。
185ご冗談でしょう?名無しさん:2009/03/26(木) 21:39:16 ID:???
反発係数は運動エネルギーが他のエネルギー(熱とか)に変換されない割合。
反発係数が1とは、全く変換されない場合。
これは、「問題以前の前提条件」を述べているだけに過ぎない。
「全く」、つまり「1」、理系以外の人にわかりやすく言うなら「100%」。
186ご冗談でしょう?名無しさん:2009/03/26(木) 21:50:02 ID:???
>>182
では、電位を求める問題ではこれが答えだというのはないと言う事でいいですか?
187ご冗談でしょう?名無しさん:2009/03/26(木) 21:56:40 ID:???
just avobe T_c ってT_cの右極限ですか?
188ご冗談でしょう?名無しさん:2009/03/26(木) 21:57:33 ID:???
あぁ逆だ
just above T_cってT_cの右極限って意味ですか?
189ご冗談でしょう?名無しさん:2009/03/26(木) 22:00:17 ID:???
>>186
どこに電位=0の基準を持ってきてもそれが全て答えだよ。
電位が発散するところは別だけど
(点電荷の場合の原点や、155の場合の無限遠など)

190ご冗談でしょう?名無しさん:2009/03/26(木) 22:35:47 ID:???
>>189
分かりました。
とても引っかかっていたので、助かりました!
191ご冗談でしょう?名無しさん:2009/03/26(木) 22:58:30 ID:oEVZuWVX
>>185
疑問点は
>>反発係数は運動エネルギーが他のエネルギー(熱とか)に変換されない割合。
>>反発係数が1とは、全く変換されない場合。

の理由なのですが。
反発係数が1、ということが、他のエネルギーに変換される割合が0
につながる論理がわからないのです。

192ご冗談でしょう?名無しさん:2009/03/26(木) 23:04:13 ID:???
反発係数=0.5はエネルギーが半分失われる場合ではないので、>>185 の表現はあまりよくないと思う。
193ご冗談でしょう?名無しさん:2009/03/26(木) 23:12:23 ID:???
v:はねかえる前の速さ
v':はねかえった後の速さ
v=v'なら
mv^2/2 - mv'^2/2 = 0
194ご冗談でしょう?名無しさん:2009/03/27(金) 00:05:10 ID:WhPK7MZh
>>193
それは衝突相手が静止している場合で単純化しすぎ
195ご冗談でしょう?名無しさん:2009/03/27(金) 00:08:36 ID:???
アホばっか。
196ご冗談でしょう?名無しさん:2009/03/27(金) 00:15:08 ID:???
>>191
普通に計算したら出るだろ。
197ご冗談でしょう?名無しさん:2009/03/27(金) 00:22:46 ID:???
【物理】TSURI研究所、未知の素粒子発見により爆発
http://gimpo.2ch.net/test/read.cgi/news7/1238079425/
198ご冗談でしょう?名無しさん:2009/03/27(金) 00:30:46 ID:WhPK7MZh
>>196
そりゃ計算したら出るけどさ・・・
そういう意味の質問じゃなくて、もっと本質的なイメージを知りたいというか
199ご冗談でしょう?名無しさん:2009/03/27(金) 00:39:13 ID:???
z=f(r)(r=(x^2+y^2)^1/2)で表される表面の上を質量mの失点が運動している。
z軸方向には重力があるとする。

この問題で、ラグランジアンを求めたいのですが、
ポテンシャルの表し方が分かりません。
円筒座標r,θを用いて表せとあるのですが、どなたか教えてくれないでしょうか?
200ご冗談でしょう?名無しさん:2009/03/27(金) 01:18:26 ID:???
流れを読まずに質問します。

炎天下の車のトランクの中に置いたプラボックス(RV-BOXの様な箱)の中も
高温になると思うのですが、その中は温度が上がらないので、カセットボンベを入れても
ボックス内はクーラーボックスと同じ原理で温度は上がらないので大丈夫と主張する人がいます。
その人の理屈では、空気は熱伝導率が低いので車内が高温になっても、
軽く密閉されたボックス内は高温にならないそうですが、本当ですか?
201ご冗談でしょう?名無しさん:2009/03/27(金) 02:50:31 ID:???
トランクの中に浮遊させてんの?
202ご冗談でしょう?名無しさん:2009/03/27(金) 05:10:46 ID:???
>>199
単純に重力の位置エネルギーmgzにzとr,θの関係を代入すればよい。
203ご冗談でしょう?名無しさん:2009/03/27(金) 06:24:59 ID:???
>>201
いえ、置いてあるだけです。
204ご冗談でしょう?名無しさん:2009/03/27(金) 06:33:29 ID:???
>>201
・・・スイマセン途中で送信しました。

しかし、RV-BOXは面で接地せずに、線で接地した状態(形成型)で、
1センチ程度浮いた状態です。
205ご冗談でしょう?名無しさん:2009/03/27(金) 09:34:12 ID:???
まずは回答
>200
あまり正確なことは言えませんが、それは危険だと思います。
確かに空気は熱伝導率が悪いですが、それでもカセットボンベ程度の熱容量であればすぐにまわりと
同じくらいの温度になってしまうと思います。
車内程度の温度には耐えられるようにボンベを設計しているかもしれませんが、実験はしない方がいいですww



すみません、量子コンピュータについて質問です。

制御制御NOTがわかりません。
ttp://www.appi.keio.ac.jp/Itoh_group/ohp/qc1.pdf
の14ページのような制御制御notで、Cfの値がなぜAi=Bi=1の場合のみCiの否定になるのでしょうか?
同サイトの10ページを見る限り、NOT演算と制御NOT演算以外は使っていないはずだと思うのですが、
計算結果からは実質的にAiとBiのAND演算をしてから制御NOTをしているように見えてしまいます・・・
できれば14ページの回路図(?)の読み方も教えてくれるとうれしいです。
206205:2009/03/27(金) 09:54:59 ID:???
ごめんなさい、10ページでなく、9ページでしたOTL
207ご冗談でしょう?名無しさん:2009/03/27(金) 10:16:51 ID:???
>>205
つまり空気の熱伝導率は十分に高いんだなw
208ご冗談でしょう?名無しさん:2009/03/27(金) 10:31:07 ID:UDQUBTJT
東大、ブラックホール実験とワープ実験に成功
209ご冗談でしょう?名無しさん:2009/03/27(金) 13:19:00 ID:xnneMAzI
>>200
空気の断熱効果で中の気温が変動しにくくなる効果はあると思われるが、
外の高温が続くと結局その温度まで熱せられてしまう。
210ご冗談でしょう?名無しさん:2009/03/27(金) 14:05:43 ID:???
211ご冗談でしょう?名無しさん:2009/03/27(金) 14:08:18 ID:???
物質 熱伝導率 k (W・m-1・K-1)
金属(25℃)

銀 427
銅 397
金 314
アルミニウム 238
鉄 79.5
鉛 34.7
気体(20℃)

水素 0.172
ヘリウム 0.138
酸素 0.0238
窒素 0.0234
空気 0.0234
非金属(概略値)

氷 2
コンクリート 0.8
ガラス 0.8
水 0.6
ゴム 0.2
木 0.08
アスベスト 0.08
表4.1 熱伝導率
212ご冗談でしょう?名無しさん:2009/03/27(金) 15:36:12 ID:???
ファインマン物理学の
仮想仕事の原理の問題です。

http://imepita.jp/20090327/501650
図のようなトラスで、対角線の支柱はみな5の長さ、水平の支柱はみな6の長さである。
つなぎめは蝶番でみな自由に動く。トラスの重さは無視できる。
a)図のように荷をかけたとき、支柱のどれとどれは、針金で代用できるか。
b)支柱BD,DEの内部にはたらく力を求めよ。

aはなんとなく感覚でわかったのですがbがわかりません。
僕は以下のように考えましたがどこがおかしいか教えてください。
BDが2Δxだけ縮むと点Cが(3/4)Δxだけ下がり、おもりは(3/8)Δxだけ下がるのでBD=(3/16)W
DEがΔxだけ伸びるとおもりは(4/5)Δxだけ下がるのでDE=(4/5)W

ちなみに答えは
a)AC,CE,EG,EF,ED,BC
b)BD=(4/5)W,DE=(5/12)W
です
213ご冗談でしょう?名無しさん:2009/03/27(金) 17:14:12 ID:???
>>200
伝導だけじゃないからね、車のトランクの中だと対流や輻射の方が熱輸送の主役だろう。
214ご冗談でしょう?名無しさん:2009/03/27(金) 18:30:37 ID:oRxzisDI
磁石の圧積で、斥力でる?
215ご冗談でしょう?名無しさん:2009/03/27(金) 18:40:58 ID:???
NG ID:oRxzisDI
216ご冗談でしょう?名無しさん:2009/03/28(土) 04:57:18 ID:???
素朴な疑問なんですけど
地球の中心まで穴を掘り圧力とか熱は影響受けないとして
その穴の底でジャンプしたらどの程度跳べるんでしょう?
217ご冗談でしょう?名無しさん:2009/03/28(土) 05:33:37 ID:uaSg1LPY
永遠に、飛びづつける飛行機に乗った人の時間はどうなるんすか。
地上の人と比べ、感じ方は異なりますか。
218ご冗談でしょう?名無しさん:2009/03/28(土) 05:37:48 ID:???
なんだこのあほくさい質問の連続・・・
219ご冗談でしょう?名無しさん:2009/03/28(土) 05:45:20 ID:???
4πGρh^2/6 = gH
で解けるんじゃないかしら

ちなみに
ρ = 12 [g/cm^3]
220ご冗談でしょう?名無しさん:2009/03/28(土) 11:24:04 ID:???
>>217
速度と系によって時間が多少伸びたり縮んだり。それだけ。
そもそも地球は太陽の周りを、太陽は銀河の周りを、銀河はボイドの周り(?)
を飛んでいるのだから、程度は違えども何も変わるはずも無し。

伸びたり縮んだり、は、地球だって上記の理由で「厳密には」静止系じゃ無い
はずだから。・・・なんだけど、勘違いしてるかな?
系A、系Bがあったとき、互いに観測すると時間の進みが違う。
より光速度に近い速度で等速度運動している系Bを系Aから観測すると、時間が
遅れることが観測されている。
ここで系Bを地球、系Aを太陽と置くと、公転速度(11万km/h)から、2億分の1は
遅れている計算になる。(太陽重力による遅延は単純化のため除外)
221ご冗談でしょう?名無しさん:2009/03/28(土) 12:43:44 ID:???
>>220
バカだろお前ww
222ご冗談でしょう?名無しさん:2009/03/28(土) 13:43:03 ID:???
>>221
うん。で、それが何か?
223ご冗談でしょう?名無しさん:2009/03/28(土) 13:49:33 ID:???
俺の下半身の一部も朝と夜だけ伸びるな
224212:2009/03/28(土) 18:45:10 ID:???
レスがつかないので別の場所で質問することにします。
ありがとうございました。
225ご冗談でしょう?名無しさん:2009/03/28(土) 22:22:23 ID:???
2009年3月27日から30日まで 第64回年次大会 が開催されます.
226ご冗談でしょう?名無しさん:2009/03/28(土) 22:22:33 ID:???
t=0でmoの質量を持つ物体が、静止の位置から、後方にいつでも自分に相対的にUという
速度で連続的に物体を投げながら全身するとき、その後任意の時刻での速度と進んだ距離を求めよ。

という問題で
時刻tで質量m、速度v
時刻t+dtで質量m-dm、速度v+dvとして
運動量保存より
mv=(m-dm)(v+dv)+dm(v-U)
と考えたのですが、答えが合いません。
どこがおかしいのでしょうか(><)
227ご冗談でしょう?名無しさん:2009/03/28(土) 22:29:07 ID:???
>>226
> 物体を投げながら

その物体の質量/sは?
228ご冗談でしょう?名無しさん:2009/03/28(土) 22:31:02 ID:???
問題全文ちゃんと書けよボケ
229ご冗談でしょう?名無しさん:2009/03/28(土) 22:48:39 ID:???
>>227
>>228
問題は226の通りです。

答えは速度が
v=Ulog(m0/m)
と、なっています。

>時刻tで質量m、速度v
時刻t+dtで質量m-dm、速度v+dvとして
運動量保存より
mv=(m-dm)(v+dv)+dm(v-U)

上の部分がおかしいのかな、と思ったのですが
どこがおかしいのか分からなくて。。
230ご冗談でしょう?名無しさん:2009/03/28(土) 22:58:03 ID:???
ツィオルコフスキーの公式という奴なので、wikipediaを参照。
231ご冗談でしょう?名無しさん:2009/03/28(土) 23:14:23 ID:???
>>230
ありがとうございました!
232ご冗談でしょう?名無しさん:2009/03/28(土) 23:45:03 ID:X1/EDSWs
磁石は
水中でも普通に機能するのでしょうか
233ご冗談でしょう?名無しさん:2009/03/29(日) 00:25:32 ID:???
>>226
途中までじゃなくて、最後まで自分の計算書いてみて
234ご冗談でしょう?名無しさん:2009/03/29(日) 00:29:51 ID:???
>>232
反磁性を考えたんだろう。俺も気になった。
んで、実験してきた。普通にくっつくよ。
235ご冗談でしょう?名無しさん:2009/03/29(日) 02:49:43 ID:???
質問です
無限機関について何かでエネルギーの増幅効果と位相のズレ?現象を
利用すれば可能。というモノを見た覚えがあるのですが・・・

事実上「永久の機関」の製作は可能なのでしょうか?
236ご冗談でしょう?名無しさん:2009/03/29(日) 02:55:32 ID:???
エネルギーの増幅とか言ってる時点でダメだろ。
237ご冗談でしょう?名無しさん:2009/03/29(日) 07:42:43 ID:+x+K/0ci
無限機関についてエネルギーの幅と位相のズレで、永久機関かな?なんて。
238ご冗談でしょう?名無しさん:2009/03/29(日) 08:15:43 ID:J+70jvg3
>>235
無限機関って何?定義されてないんだけど。
エネルギー増幅効果
位相のずれ現象
も定義されてない。
故に君の質問文は意味不明。
239ご冗談でしょう?名無しさん:2009/03/29(日) 08:23:35 ID:+x+K/0ci
無限機関についてエネルギーの幅と位相相異のズレで、永久機関かな。
240ご冗談でしょう?名無しさん:2009/03/29(日) 08:38:31 ID:+x+K/0ci
エネルギーの圧積で、斥力でるかな。
241ご冗談でしょう?名無しさん:2009/03/29(日) 10:59:04 ID:???
実験系と理論系の違いがいまいちよくわかりません。
未知の分野をやることになったんで自分で理論作って実験で確認しようと思うんですが…

本来は論文出して、その論文を元に実験屋が検証するって形なんでしょうか
242ご冗談でしょう?名無しさん:2009/03/29(日) 11:22:07 ID:o1SNTcMs
r=(x,y,z)、r'=(x',y',z')のとき
(r-r')/|r-r'|^3=(r/|r|^3)-(r'/|r|^3)+(3r(r・r')/|r|^5)+...
という展開の証明の仕方がわかりません。
教えてください。お願いします。
243ご冗談でしょう?名無しさん:2009/03/29(日) 11:40:53 ID:xMvofQlq
>>234さん
ありがとう
244ご冗談でしょう?名無しさん:2009/03/29(日) 12:26:33 ID:???
コーシーが分散式を導出したころの、屈折率の測定(ハーシェルとかフランホー
ファとか)と、そのころの分散の実験式の話を何かの日本語の本で読んだよう
に思うんだがどこで読んだか思い出せないので、出ていそうな本をおしえてく
ださい。広重徹『物理学史』ではありません。
245ご冗談でしょう?名無しさん:2009/03/29(日) 13:32:52 ID:???
>>242

宿題丸投げ馬鹿は死んどけよ、マジでwww
246ご冗談でしょう?名無しさん:2009/03/29(日) 13:42:26 ID:OMRIiqhv
位置変池(いちへんち)で、永久機関はいける思う。日本語へたくそでごめん。
247ご冗談でしょう?名無しさん:2009/03/29(日) 16:22:18 ID:???
意味不明な俺様用語で説明できたことになるなら
どんな荒唐無稽な機関だって可能だ。
248ご冗談でしょう?名無しさん:2009/03/29(日) 18:58:53 ID:???
ローレンツ変換は正準変換ではないですね。
249ご冗談でしょう?名無しさん:2009/03/29(日) 19:21:19 ID:Mju76fFx
一番わかりやすい相対性理論の入門書を教えてください。
250ご冗談でしょう?名無しさん:2009/03/29(日) 19:47:22 ID:???
昨日今日と参加登録せずに学会に潜ってるんですが、当日参加登録って奴はするといくら取られるんですか?
251ご冗談でしょう?名無しさん:2009/03/29(日) 21:00:26 ID:t/hEtC9T
くだらない質問なんですが
仮に横幅が30万kmあるディスプレイがあるとして
そのディスプレイの右端から左端までマウスカーソルかなんか
デイスプレイの中に表示されているものを一秒より短く移動させれば
疑似的な超光速は再現できますか?
252ご冗談でしょう?名無しさん:2009/03/29(日) 21:04:49 ID:???
251君は文系?その発想がおもしろい。ワケねえだろう。
253ご冗談でしょう?名無しさん:2009/03/29(日) 21:25:55 ID:???
>>251
質量のあるものは光速を越えられない
OK?
254ご冗談でしょう?名無しさん:2009/03/29(日) 21:29:38 ID:???
この手のご意見てもうテンプレだよね
255ご冗談でしょう?名無しさん:2009/03/29(日) 21:30:53 ID:t/hEtC9T
>>253
それを前提に質問したというか・・・・

言葉にできない、なんていえばいいのかわかんないorz
迷惑かけてすまんかった、>>251の質問はスルーしてくれ
256ご冗談でしょう?名無しさん:2009/03/29(日) 21:32:26 ID:???
>>251
電気信号の速度一つとっても光速よりだいぶ小さい。
257ご冗談でしょう?名無しさん:2009/03/29(日) 21:40:11 ID:???
> 疑似的な超光速は再現できますか?

なんでそんなことを聞くのかわからん。表示されているものを端から端まで 1秒未満で
移動させるのは、そりゃ可能だろう。単にディスプレイ上の画素の点灯をコントロールする
だけなんだから。

それを「擬似的な超光速」だと言いたいのならそれもいいだろう。

「擬似」であって本物じゃないことがわかってるなら話はそれでおしまいだ。

何でそんなことを物理板で聞くの?
258ご冗談でしょう?名無しさん:2009/03/29(日) 21:47:33 ID:???
>>255
質量の無いものは光速を越えても構わない
OK?
259ご冗談でしょう?名無しさん:2009/03/29(日) 21:52:05 ID:???
誰か人生で一度でも学会の参加費払ったことがある奴、>>250たのむ
260ご冗談でしょう?名無しさん:2009/03/29(日) 22:16:38 ID:???
交通費宿泊費込みで払ったから忘れた
261ご冗談でしょう?名無しさん:2009/03/29(日) 22:46:12 ID:???
>>250
>昨日今日と参加登録せずに学会に潜ってるんです
通報しました。罰金7000円を覚悟しろ。
262ご冗談でしょう?名無しさん:2009/03/29(日) 22:54:23 ID:???
見るだけならいいだろ
263ご冗談でしょう?名無しさん:2009/03/29(日) 22:55:50 ID:???
入口には部外者立入禁止って書いてあるわな
264ご冗談でしょう?名無しさん:2009/03/29(日) 23:11:52 ID:???
>>262
>見るだけならいいだろ
だめです。
265ご冗談でしょう?名無しさん:2009/03/29(日) 23:13:40 ID:???
で、参加費いくらなの?
266ご冗談でしょう?名無しさん:2009/03/29(日) 23:16:21 ID:???
>>265
罰金7000円と合わせて、14000円を請求いたします。
267ご冗談でしょう?名無しさん:2009/03/29(日) 23:18:10 ID:???
高すぎるな
まあ、周りは顔見知りばっかだしもう払わなくていいや
268ご冗談でしょう?名無しさん:2009/03/29(日) 23:21:50 ID:???
情熱大陸「物理学者 益川敏英・小林誠」
http://live23.2ch.net/test/read.cgi/livetbs/1238335635/
269ご冗談でしょう?名無しさん:2009/03/29(日) 23:21:55 ID:???
参加票をしっかりチェックしております。
お声をかけられたら素直にお払いください。
270ご冗談でしょう?名無しさん:2009/03/29(日) 23:38:07 ID:???
会場になってる大学の学生は黙認することも多いけどな。
271ご冗談でしょう?名無しさん:2009/03/29(日) 23:39:57 ID:???
不逞の輩は学会追放と決まっとる。
272ご冗談でしょう?名無しさん:2009/03/29(日) 23:47:20 ID:QVw87b06
高度600km近くの宇宙空間を飛ぶテポドン2号を日本の迎撃ミサイルが
打ち落とすために配備されていますが、
秋田、岩手に配備される地対空誘導弾PAC3(迎撃最高高度30−35km)、
日本海側に配備されるイージス艦に搭載された
海上配備型迎撃ミサイルSM3(迎撃最高高度200〜300Km)、
と、いずれもテポドン2号に届かないですけどなんでマスゴミは
これらの迎撃ミサイルでテポドン2号を打ち落とすことができると
報道しているのですか?物理的に不可能ではないんですか?
273ご冗談でしょう?名無しさん:2009/03/29(日) 23:48:38 ID:???
>>272
ミサイルがある程度落ちてきたところで落とせばいいんじゃないの?
274ご冗談でしょう?名無しさん:2009/03/29(日) 23:50:15 ID:???
>>272
大和魂+神風でOKさ。
275ご冗談でしょう?名無しさん:2009/03/29(日) 23:52:01 ID:???
落ちてこなけりゃ迎撃する必要もないだろうにJK
276ご冗談でしょう?名無しさん:2009/03/29(日) 23:57:53 ID:???
>>272
マスゴミは馬鹿文系の脳たりんが多いから物理的な面で考えることが出来ないんだよ。
実際、人工衛星を打ち上げるロケットであるテポドン2号を打ち落とすには、
同じように人工衛星を打ち上げるロケットで打ち落とすしかないわなw
今現在のところテポドン2号を打ち落とすことのできるミサイルはどの国も開発できていない。
だから北朝鮮が核とテボドン2号を持つことに政治的意味があるんだよ。
277ご冗談でしょう?名無しさん:2009/03/30(月) 00:03:46 ID:???
>>276
>どの国も開発できていない。
中国は実験成功してますが。
278ご冗談でしょう?名無しさん:2009/03/30(月) 00:05:23 ID:QVw87b06
>>273-276
マスゴミの作成したイメージ映像では宇宙空間を飛んでいるテポドンに
日本のミサイルが直撃していました。こんな嘘を垂れ流して良いんでしょうか?
一応抗議の電話を入れておきます。皆さん回答ありがとうございました。
279ご冗談でしょう?名無しさん:2009/03/30(月) 00:07:56 ID:???
>>278
おう頼んだぞ!
280ご冗談でしょう?名無しさん:2009/03/30(月) 02:06:05 ID:???
質量mである質点の鉛直方向一次元運動を考える。鉛直上向きをx軸の正の向きにとり、重力加速度をgとする。
また、質点の速度vに比例する空気抵抗を考慮し、抵抗力を-mbvとする。(bは正の比例定数)

以上が問題文です。ここから、運動方程式を立て、速度、位置、時間が十分に経過した後の質点速度を求めるのですが
(d/dt)^2 x= -g-bv   −@
(d/dt)^2 x= -g+bv   −A
の、どちらが正しい運動方程式となるのでしょうか?
また、位置の解はどのようになるのでしょうか?
文中に、上(下)に投げる(落とす)、などがないため、抵抗力の符号をどのように扱ってよいのかがわかりません。

よろしくお願いします。

281ご冗談でしょう?名無しさん:2009/03/30(月) 02:32:02 ID:???
>>280
なんで左辺は位置を時間微分してるのに右辺は速度とか言う意味不明な物理量使ってるの?
282ご冗談でしょう?名無しさん:2009/03/30(月) 03:05:36 ID:9RWRSiK4
>>272
報道される事象をよく吟味してください。

PAC-3で撃墜しようとしてるのは通常飛翔中のテポドンでは無く、「打ち上げに失敗して日本に落下しそうになった場合」です。

ですので、高度は打ち上げに失敗したテポドンがPAC-3の有効射高以下に降下した時に迎撃するので、特に問題はありません。

故にこの件は物理学的な質問でも回答でもありません。
283ご冗談でしょう?名無しさん:2009/03/30(月) 08:52:51 ID:???
>>272
朝、テレビで同じようなのやっててワロタw
明らかに宇宙空間のテポドンを迎撃ミサイルが宇宙空間で撃破してたな。
その時惑星が近くにあって更にフイタw
どんだけ高高度を迎撃ミサイルが飛ぶんだよ。もうなんかのギャグなじゃね?
284ご冗談でしょう?名無しさん:2009/03/30(月) 09:36:19 ID:haWAV4HL
これは、なんの数式でしょう。歌であらわしてみました。

http://www.youtube.com/watch?v=ydeFr7pUv9s&NR=1
285ご冗談でしょう?名無しさん:2009/03/30(月) 09:55:45 ID:???
これは、なんの数式でしょう。歯であらわしてみました。

http://www.youtube.com/watch?v=33jSsrDDTKI
286ご冗談でしょう?名無しさん:2009/03/30(月) 10:50:09 ID:haWAV4HL
統一理論でした。
287ご冗談でしょう?名無しさん:2009/03/30(月) 14:07:16 ID:???
>>280
v>0のとき速度は上向きか下向きか
そのとき -mbv は上向きか下向きか
v<0のときは各々どうか
288ご冗談でしょう?名無しさん:2009/03/30(月) 18:26:50 ID:???
>>281
私も詳しくは分かってないのですが、b[1/s]という単位を持っていれば特別おかしいことはないと思います。
運動方程式といいつつも、ma=Fの形になっていないのは、解答欄自体が(d/dt)^2 x=[  ]の形になっているためです。

>>287
記載しておらず、申し訳ありません。
この問いは、
(d/dt)^2=(a)
(a)に入る式を答えよ。
と、いう形式で出されています。ゆえに、場合分けが出来ない状況になっていると思われます。
289ご冗談でしょう?名無しさん:2009/03/30(月) 18:29:40 ID:???
>>288
>>281は疑問ではなく速度も微分形式使えよヴォケという意味を穏当に伝えたものです
290ご冗談でしょう?名無しさん:2009/03/30(月) 19:09:08 ID:???
>>288
>場合分けが出来ない状況になっていると思われます。
場合分けできないのではなく、する必要がない。
「時間が十分に経過した後の質点速度を求める」という問題設定なので
どういう場合なのかは一意に決まってしまうから。
291ご冗談でしょう?名無しさん:2009/03/31(火) 00:13:50 ID:???
>>289
そういう意味でしたか。これからは気をつけます。

>>290
最終的な「問い」として求めますが、途中段階(運動方程式を立てろ)ではそのような言葉は一切出てきていないんです。
どちらにしても、そう解釈するほか、この問いは解くことができないですね。
ありがとうございました
292ご冗談でしょう?名無しさん:2009/03/31(火) 00:26:10 ID:???
>>291
> 抵抗力を-mbvとする。(bは正の比例定数)
と問題に書いてあって
> (d/dt)^2 x= -g+bv   −A
こうなるかもしれないと思ったのはどうして?
293ご冗談でしょう?名無しさん:2009/03/31(火) 01:25:23 ID:???
2枚の導体円板(半径をaとする)の電極を距離d離して平行に配置し、隙間を誘電体で
埋めてコンデンサを作ったところ、その静電容量はCであった。
このとき、コンデンサを電圧Voに帯電させた状態で、2つの電極の間を時刻t=0に抵抗Rで繋いだ。
ここで時刻tにおけるコンデンサの電圧をV(t)とすると
V(t)に対する微分方程式ってどうなりますか?

回路方程式より
V(t)+RI=0と思ったのですが、Iが与えられてないので色々考えたのですが、分からなくて。。
294ご冗談でしょう?名無しさん:2009/03/31(火) 01:36:08 ID:???
>>293
時刻 t で極板にたまっている電荷を Q(t) とおいていろいろ式を立ててみる
295ご冗談でしょう?名無しさん:2009/03/31(火) 01:38:17 ID:???
>>294
I(t)=dQ(t)/dtしか思いつきません。。
296ご冗談でしょう?名無しさん:2009/03/31(火) 01:41:01 ID:???
294にQ(t)=cV(t)代入で

V(t)+RCdV(t)/dt=0でおkですか?
297ご冗談でしょう?名無しさん:2009/03/31(火) 03:25:48 ID:???
>>292
鉛直上向きが正であるなら、mg(重力)は負。
ならば、その逆に働く抵抗力(抵抗力)は正かなぁ、と。
視点が座標軸を基準にして考えるとこうなりました。
mg=mbvになったとき、ma=0になる、というのも(間違ってますかね)、挙げた理由の一つです。
298ご冗談でしょう?名無しさん:2009/03/31(火) 03:44:39 ID:???
>>297
> mg(重力)は負
g<0 としてるの?それで抵抗がないとき
> (d/dt)^2 x= -g
でつじつまは合う?
299ご冗談でしょう?名無しさん:2009/03/31(火) 04:40:20 ID:???
>>298
合う、と思います。
g=9.81....で方向が負である、という意味であるなら。
300ご冗談でしょう?名無しさん:2009/03/31(火) 12:05:51 ID:9qdRP4Aw
気体分子 (剛体円板) の運動をシミュレートするお遊びプログラムを作ろうとしてます。
たとえば、2つの部屋に温度 (分子の運動速度) の違う気体を入れておいて、仕切りを取ったらぶわーって混ざるとことかを視覚化するのが目的です。

混ざるといったらエントロピー増大とかの用語が頭をよぎるんで、これを読んでみたんですけど
http://www.amazon.co.jp/
%E9%AB%98%E6%A0%A1%E6%95%B0%E5%AD%A6%E3%81%A7%E3%82%8F%E3%81%8B%E3%82%8B%E3%83%9C%E3%83%AB%E3%83%84%E3%83%9E%E3%83%B3%E3
%81%AE%E5%8E%9F%E7%90%86-%E3%83%96%E3%83%AB%E3%83%BC%E3%83%90%E3%83%83%E3%82%AF%E3%82%B9-%E7%AB%B9%E5%86%85-%E6%B7%B3/
dp/4062576201
この本の内容はほぼ理解したが、まだ自分の考えているケースに応用はできない (というか、できるかどうかまでわかっていない) という感じです。

エントロピーは「質量 \{M_i\} 位置 \{\vec X_i\} 速度 \{\vec V_i\} (i = 1, 2, .., n) の粒子からなる系」に対して定義することのできるものなのでしょうか?
可能なら、式でいうとどうなるのでしょうか。
301ご冗談でしょう?名無しさん:2009/03/31(火) 13:33:41 ID:???
>>299
g>0 なんだな。それで
> (d/dt)^2 x= -g+bv
とすると、
> mg=mbvになったとき、ma=0になる
こうなるときは v>0 で、上向きに動いていることになるが?
302ご冗談でしょう?名無しさん:2009/03/31(火) 14:23:43 ID:FdMAJhhU
S行列って遷移行列の散乱バージョンってこと?
303ご冗談でしょう?名無しさん:2009/03/31(火) 20:55:52 ID:xEQdetyf
田崎『熱力学』の演習問題7.7の質問です。

示量変数の組(X,Y)の系(Xは1成分でF[T;X,Y]はXについて二回微分可能で下に凸)で、Xを自由に動かせるように
したときの平衡でのXの値X*(T;Y)が次式で定まること
F[T;X,Y]_X | X=X*(T;Y) = 0
および
X*(T;Y)_T = [ {F[T;X,Y]_XX}^(-1) * S(T;X,Y)_X ] X=X*(T;Y)
は示せたのですが、その後の「温度が上がると系が吸熱する方向に、温度が下がると系が発熱する方向にX*が動く」
「YiをYの一つの成分とするときにX*(T;Y)_Yiについて同様の考察をせよ」という問いについてです。
解答を見てもこの部分は略されてしまっているためここで行き詰まっています。どなたかご教授くだされば幸いです。
304ご冗談でしょう?名無しさん:2009/03/31(火) 23:21:45 ID:???
ル シャトリエ
305ご冗談でしょう?名無しさん:2009/03/31(火) 23:38:38 ID:???
>>301
つまり、どういう場合においても
ma=-mg-mbv
であることには変わりない。ってことでいいんですよね。
理由はv自身に正負があるため。ですね。

やっと説明している内容が理解できました。
306ご冗談でしょう?名無しさん:2009/03/31(火) 23:50:23 ID:???
>>303
X*をTで微分した量が求まってるから、正負を吟味すればいいんじゃない?
307ご冗談でしょう?名無しさん:2009/04/01(水) 00:47:46 ID:UiWqD+mR
高校物理なのですがここで質問してもよろしいでしょうか?

図を描かないと分かりにくいかと思うのですが一応…

問題.質量MのQにばね定数kのばねをとりつけ、質量mのPをばねに押し当てて自然長からl縮んだ状態にし、手を離す。
ばねから離れた後のPの速さを求めよ。

解説ではmに働く力をv、Mに働く力をVとして、運動量保存則がmv=MVとなっているのですが何故こうなるのでしょうか?
今まで運動量保存則(衝突の場合であれば)は(衝突前の運動量)=(衝突後の運動量)と思っていたのですがこれは間違っていますか?
この問題では何と何がイコール関係なのかが良くつかめません。

ちなみに出典は物理のエッセンス(河合出版)の力学76です。
308ご冗談でしょう?名無しさん:2009/04/01(水) 01:00:35 ID:???
>解説ではmに働く力をv、Mに働く力をVとして、運動量保存則がmv=MVとなっているのですが
ほんとに「働く力」がvとかVとか書いてあったの? 離れた後の速さではなくて?

>運動量保存則(衝突の場合であれば)は(衝突前の運動量)=(衝突後の運動量)と思っていたのですがこれは間違っていますか?
それであってるよ。今の場合なら(ばねが縮んだ状態の運動量)=(ばねから離れた状態の運動量)になるな。
309ご冗談でしょう?名無しさん:2009/04/01(水) 01:58:16 ID:UiWqD+mR
>>308
すみません。離れた後の速さです。

この問題の状況が良く理解できないのですが、物体Qが動くのは手を離した後ということでいいのですか?
よろしければ縮んだときの運動量と離れた後の運動量をそれぞれ教えていただきたいです。
310ご冗談でしょう?名無しさん:2009/04/01(水) 02:20:30 ID:???
縮めたときどうなっているかは問題設定だろう。バネが縮んだ状態で全体を手で押さえて止めてるんじゃないの?
311ご冗談でしょう?名無しさん:2009/04/01(水) 02:34:16 ID:UiWqD+mR
>>310
そうだとすると縮めたときの運動量は0でいいのですか?
312ご冗談でしょう?名無しさん:2009/04/01(水) 03:28:26 ID:???
素粒子(ストリングでも何でもいいです)間の空間ってのは、真空なんでしょうか?

光や電磁波は、その空間を通過しますでしょうか?
313ご冗談でしょう?名無しさん:2009/04/01(水) 08:36:22 ID:???
>>312
>素粒子(ストリングでも何でもいいです)間の空間ってのは、真空なんでしょうか?
そうです。
>光や電磁波は、その空間を通過しますでしょうか?
そうです。
314ご冗談でしょう?名無しさん:2009/04/01(水) 17:29:01 ID:???
デジタル信号処理に
カイザー窓関数が最適な理由を教えてください。
315ご冗談でしょう?名無しさん:2009/04/01(水) 18:27:34 ID:LSH1pMbM
最悪、スペースシャトルに最高の水素エンジン車組み込めば、できるか 宇宙船
316ご冗談でしょう?名無しさん:2009/04/01(水) 18:49:05 ID:LSH1pMbM
木星まで、行って帰れるくらいだが。量産化も簡単だし。
317ご冗談でしょう?名無しさん:2009/04/01(水) 19:27:27 ID:LSH1pMbM
料金は、18万円ってとこだろ。
318ご冗談でしょう?名無しさん:2009/04/01(水) 21:16:42 ID:dcQ6OP+k
超聖水に蜂蜜と、小さい塩一粒入れると、回復水ができるよ。
牛乳と蜂蜜と、小さい塩一粒入れてもできるよ。
319ご冗談でしょう?名無しさん:2009/04/01(水) 22:37:16 ID:aXAGLQid
完全性の条件というものがよくわからないので質問します。

・規格化直交条件
∫φ(x)φ*(x')dx=δ_{xx'}
(クロネッカーのデルタです)
は直交しているときにはゼロ、してないときには
1ということで理解していますが、

・完全性の条件
Σ_{i}φ(x)φ*(x')=Δ(x-x')
(紛らわしいのでΔをデルタ関数として使ってます。)
がどういうことを意味しているのかよくわかりません。
猪木を見てもいきなり唐突に「完全性」という
言葉が使われていて意味がよくわかりません。
僕の頭の中では
これを積分したら規格化直交条件になるのだろうという
くらいのイメージしかありません。(たぶん
間違ってるでしょうけど)

ということで、完全性の条件について説明を
よろしくお願いします。
320ご冗談でしょう?名無しさん:2009/04/01(水) 23:02:18 ID:???
>>319
書き方滅茶苦茶

ブラケットで書いたほうがわかりやすい気が
規格直交
<ψ_i|ψ_j> = δ_{ij}
完全性
Σ_i (|ψ_i><ψ_i|) = 1 (単位行列)

完全性は大雑把に言って、
|ψ_i> たちでベクトル空間全体を張ってるってこと

3次元のベクトル空間で {(1,0,0)^T, (0,1,0)^T} は
規格直交条件はOKだけど、完全性は満たしてない
{(1,0,0)^T, (0,1,0)^T, (0,0,1)^T} なら完全性も満たしてる
321ご冗談でしょう?名無しさん:2009/04/01(水) 23:14:01 ID:aXAGLQid
>>320
すいません。
iは状態を指定する添え字で
波動関数につきます。

Σ_i (|ψ_i><ψ_i|) = 1

この式によって状態ベクトルが
ヒルベルト空間を張っていることを
あらわしているとイメージすることができないのです。
有限次元と無限次元だと明確な
定義も異なるでしょうし。

ブラケットでもわからないので
使わない場合だともっとアボーンです。

Σ_{i}φ_i(x)φ_i^*(x')=Δ(x-x')
で、なんでシグマをとると
デルタ関数になるのかがわかりません。

しつこくてすいません。
322ご冗談でしょう?名無しさん:2009/04/01(水) 23:27:49 ID:???
>>320 の例で
e_1=(1,0,0)^T, e_2=(0,1,0)^T, e_3=(0,0,1)^T
とすると Σ_i e_i e_i^T が3行3列の単位行列になるということ。
323ご冗談でしょう?名無しさん:2009/04/01(水) 23:37:28 ID:caOZTkgI
超神癒水は、超聖水に、蜂蜜をいれるだけ。
324ご冗談でしょう?名無しさん:2009/04/01(水) 23:38:42 ID:caOZTkgI
超癒水は、癒し豆腐の水を精製すること。
325ご冗談でしょう?名無しさん:2009/04/02(木) 00:12:19 ID:PbwriFUe
全部顔文字に見える魔法

今年から物理に入る工房ですた(´・ω・)
326ご冗談でしょう?名無しさん:2009/04/02(木) 00:23:42 ID:???
>>321
>デルタ関数になるのかがわかりません。
1=Σ_i|ψ_i><ψ_i|
を<x|と|x'>ではさめばいい。
327ご冗談でしょう?名無しさん:2009/04/02(木) 03:02:46 ID:???
これまで文系一本できたオッサンなのですが
突然物理に興味が出てきて、物理を知りたくてしょうがないのですが
何をどうすればよいのか解らず
結局高校レベルの数学と物理の参考書を購入致しました

ここからこれらを解く以外にいったいどうすればよいのやら皆目見当がつきません

アドバイスしていただきたいです
328ご冗談でしょう?名無しさん:2009/04/02(木) 03:48:40 ID:???
中学の教科書から読み始めろ
329ご冗談でしょう?名無しさん:2009/04/02(木) 06:49:54 ID:zMpfy1m3
ドアを閉める機械、器具があったら、売れるだろうなあ。家庭用なんか。
330ご冗談でしょう?名無しさん:2009/04/02(木) 07:55:39 ID:???
>>327
おいくつですか?この板より年齢板のほうがよいかも。
ベクトルとスカラの区別できますか?
微分積分知っていますか?
ひとまず以下のリンクで質問してください。
http://science6.2ch.net/test/read.cgi/sci/1221226036/l50
331ご冗談でしょう?名無しさん:2009/04/02(木) 13:06:29 ID:E1U6B5uY
ちんこを手でシコシコ擦ると精液がドピュドピュでますよね。
これって、摩擦エネルギーが生命エネルギーに変わったってことですか?
332ご冗談でしょう?名無しさん:2009/04/02(木) 13:07:01 ID:???
ちがいます
333ご冗談でしょう?名無しさん:2009/04/02(木) 13:41:41 ID:YtsxTN7f
「穴」が存在することを証明して下さい。
334ご冗談でしょう?名無しさん:2009/04/02(木) 14:51:29 ID:kU/3WWZg
質問です。
酸素1molと水素2molの混合気体を断熱したボンベ内で爆発させたとき、ボンベ内の温度が上昇しました。
この変化の内部エネルギー変化は何KJになりますか?
335ご冗談でしょう?名無しさん:2009/04/02(木) 16:19:33 ID:???
丸投げ死ねよチンカス
336ご冗談でしょう?名無しさん:2009/04/02(木) 16:32:16 ID:U0SAv/VN
アメリカのベンチャー企業が、新しい浮遊船を開発したと聞いたのですが、どうか教えてください。
337ご冗談でしょう?名無しさん:2009/04/02(木) 18:43:23 ID:???
>>336
「浮遊船」って何?
338ご冗談でしょう?名無しさん:2009/04/02(木) 19:36:33 ID:???
風呂はいってて思ったんだけど
何でバブって細かく砕いたら浮くの?
339ご冗談でしょう?名無しさん:2009/04/02(木) 20:00:33 ID:???
細かくすると体積に対する表面積の比率が上がって
浮力が高くなるから
340ご冗談でしょう?名無しさん:2009/04/02(木) 21:27:54 ID:???
>>338
知らんけどきっと表面張力の効果だろう。
341ご冗談でしょう?名無しさん:2009/04/02(木) 21:33:22 ID:???
>>339 表面積と浮力って関係ないだろ?
342ご冗談でしょう?名無しさん:2009/04/02(木) 21:40:59 ID:???
バブのように表面から気泡を生じるものなら関係あるな
343ご冗談でしょう?名無しさん:2009/04/02(木) 21:55:11 ID:???
科学板の質問スレから誘導されてきました
よく戦闘機がビルの付近を通過すると窓ガラスが割れるという現象がフィクションでありますが
人間大の大きさの物が窓ガラスから1メートルほどの距離で通過する際、窓ガラスを割るのに必要な速度はどれぐらいでしょうか?
やはり音速程度の速度が必要なのでしょうか
それとも音速以下の速度でも出来る事なのでしょうか
344ご冗談でしょう?名無しさん:2009/04/02(木) 22:34:48 ID:???
[E=hν] [P=h/λ] で光のエネルギーと運動量を把握できますが
変換するのに[E=1/2mv^2]へ[hν]と[h/λ]を代入して[hν=1/2m(h/λ)^2]
えっとここで出てくる[m]はどういう値を代入すればいいのでしょうか?
またシュレディンガー方程式で出てくる[m]の値は
光の場合どうしたらいいのでしょうか?
光子の存在確率と電磁波の波動関数との関係はどのように理解したら
よいのでしょうか?
シュレディンガー方程式は基本的に電子を考えているような感じですが。
素人に毛が生えた程度の人間なのでお手やわらかにお願いします。
345ご冗談でしょう?名無しさん:2009/04/02(木) 22:42:51 ID:???
まだ生えてないんじゃないの?w
346ご冗談でしょう?名無しさん:2009/04/02(木) 22:47:16 ID:???
>>344
つ [特殊相対論]
347ご冗談でしょう?名無しさん:2009/04/02(木) 23:20:15 ID:???
>>344
随分とツルッツルだねぇ。
348ご冗談でしょう?名無しさん:2009/04/02(木) 23:25:45 ID:???
訂正
[E=1/2mv^2]→[E=p^2/2m]

>>346
m=E/c^2でエネルギーの値をmに変換して代入ということですか。
349ご冗談でしょう?名無しさん:2009/04/02(木) 23:44:29 ID:???
単位の次元も考えられないアホに物理は無理。
350ご冗談でしょう?名無しさん:2009/04/02(木) 23:46:08 ID:???
>>348
そもそも、E=1/2mv^2とはニュートン力学の結論であって、
相対論では成立しない。相対論では、次のようになる。

 E=mc^2/√(1-v^2/c^2)
351ご冗談でしょう?名無しさん:2009/04/02(木) 23:52:49 ID:???
mってm_0?
352ご冗談でしょう?名無しさん:2009/04/02(木) 23:54:27 ID:???
E=c√(m^2 c^2 + p^2)
v=∂E/∂p
にしといた方がいい
353ご冗談でしょう?名無しさん:2009/04/02(木) 23:54:38 ID:???
>>351
>m_0
目が痒いの?
354ご冗談でしょう?名無しさん:2009/04/03(金) 00:46:37 ID:wFBWDBID
マンゴープリンと、黒砂糖で、アルツハイマーが、少し治るという噂があるけど、試してみる価値はある?
355ご冗談でしょう?名無しさん:2009/04/03(金) 00:50:59 ID:wFBWDBID
みつばで、治るということにあるみたい。
356ご冗談でしょう?名無しさん:2009/04/03(金) 01:04:44 ID:???
皇潤とかナントカベリーでも飲んどけよ
357ご冗談でしょう?名無しさん:2009/04/03(金) 01:49:05 ID:???
>>342 kwsk
358ご冗談でしょう?名無しさん:2009/04/03(金) 02:09:35 ID:???
>>357
 ○○○○
○■■■■○
○■■■■○
○■■■■○
○■■■■○
 ○○○○

 ○
○■○
 ○

小さい方が浮き袋=気泡の割合が多いってことではないかと
359ご冗談でしょう?名無しさん:2009/04/03(金) 03:06:25 ID:???
>>358 なるほど わかりやすい!
 じゃああれが浮くのは炭酸ガスのせいなんだね^^
360ご冗談でしょう?名無しさん:2009/04/03(金) 03:38:07 ID:???
>>359

( ゜д゜)
361ご冗談でしょう?名無しさん:2009/04/03(金) 05:06:13 ID:wFBWDBID
362ご冗談でしょう?名無しさん:2009/04/03(金) 05:07:19 ID:wFBWDBID
363ご冗談でしょう?名無しさん:2009/04/03(金) 08:51:04 ID:???
>>357
あるよ。
以上。
↓次の方どうぞ
364ご冗談でしょう?名無しさん:2009/04/03(金) 16:19:35 ID:Kv5Ojxf5
最悪、スペースシャトルに最高の水素エンジン車と圧縮縮退路組み込めば、できるか 宇宙船
カローラ並みだな、この宇宙船。ポテンシャル20%しか使わない。キャパシティ残り80%はどう使うんだろう 工業科
コロニーには、交信ポイントを付けたほうがいい 文系

365ご冗談でしょう?名無しさん:2009/04/03(金) 18:21:11 ID:Kv5Ojxf5
宇宙船、日本にセラミックのビルあるから、作れるじゃん。
366ご冗談でしょう?名無しさん:2009/04/03(金) 18:33:08 ID:Kv5Ojxf5
宇宙に出てから、パッチで、攻撃武器付ければいい。
367ご冗談でしょう?名無しさん:2009/04/03(金) 18:56:05 ID:???
今バナナを食べてて思ったんだけど
まだ若くて甘くないバナナと熟して甘くなったバナナって
同じカロリー?

目の前のテーブルに置かれた我が家のバナナは
日に日にカロリーが増しているの?
368ご冗談でしょう?名無しさん:2009/04/03(金) 19:05:17 ID:???
ごはんをずっと噛んでると甘くなるでしょ
369ご冗談でしょう?名無しさん:2009/04/03(金) 19:10:56 ID:???
ご飯は丸呑みと噛んで食べるのとでカロリー違うの?
370ご冗談でしょう?名無しさん:2009/04/03(金) 19:23:13 ID:???
ごはん自体の熱量という観点と、実際に吸収される熱量という観点では答えが違ってくる。
371ご冗談でしょう?名無しさん:2009/04/03(金) 19:33:28 ID:???
食べ物のカロリー表示は胡散臭い
372ご冗談でしょう?名無しさん:2009/04/03(金) 20:11:14 ID:???
算出方法が2通りあるらしいからなぁ。
373ご冗談でしょう?名無しさん:2009/04/03(金) 20:14:58 ID:???
>>371
科学板で、根拠もないおまえの見解なんかゴミだよ。
374ご冗談でしょう?名無しさん:2009/04/03(金) 21:39:16 ID:???
>>373
い、いきなり何を言い出すんだ君は
375ご冗談でしょう?名無しさん:2009/04/03(金) 22:49:37 ID:???
376ご冗談でしょう?名無しさん:2009/04/03(金) 23:02:27 ID:???
いやです。
377ご冗談でしょう?名無しさん:2009/04/04(土) 00:08:42 ID:???
本人に直接聞いて来いよ馬鹿。
378ご冗談でしょう?名無しさん:2009/04/04(土) 14:48:50 ID:wV9FRgAT
今の時代”天才”って誰なんでしょうかね?
思い浮かぶのは”ウイッテン”くらいで若手は皆無です。
情報の集約度から考えると昔よりも多いはずなんですけれどもねぇ。
だれか物理の”天才”教えて下さい。
379ご冗談でしょう?名無しさん:2009/04/04(土) 17:47:03 ID:7PuprbOj
>>369
分解酵素によって、違ってくる。
380ご冗談でしょう?名無しさん:2009/04/04(土) 17:50:00 ID:7PuprbOj
>>375
垂直ジャッジ
381ご冗談でしょう?名無しさん:2009/04/04(土) 19:48:25 ID:???
>>360
気を確かに。
382ご冗談でしょう?名無しさん:2009/04/04(土) 20:44:08 ID:???
>>379  人体内でもエネルギー保存則は成り立つのよ。
383ご冗談でしょう?名無しさん:2009/04/04(土) 20:51:51 ID:???
>>382
で?
384ご冗談でしょう?名無しさん:2009/04/04(土) 20:58:27 ID:???
きっちり消化されてエネルギーになるか、
消化されなくてうんこになって出てくるかの違いだろ
385ご冗談でしょう?名無しさん:2009/04/04(土) 22:56:45 ID:???
>>344
なんですけど
[hν=1/2m(h/λ)^2]書き間違い修正→[hν=(h/λ)^2/2m]
光の場合[m]に何を入れたらいいかという話で
単位の次元がと言われましたが
単位は組み合わせてできるというのはわかりますが
この場合どうまずいのかわかるレベルにありません。
自称素人に毛が生えた程度と申しましたが
毛も生えていない小学生でもかまいせん。
どうぞ回答よろしくお願いします。
それと光の場合光子の存在確率と運動量とエネルギーと
シュレディンガー方程式マックスウェルの波動方程式との
関係はどんなものなのでしょうか?
386ご冗談でしょう?名無しさん:2009/04/04(土) 23:42:01 ID:???
>>385
・光子では E=p^2/(2m) は成り立たない
・光子はシュレーディンガー方程式に従わない
387ご冗談でしょう?名無しさん:2009/04/04(土) 23:43:30 ID:???
光つまり電磁波は質量は0だよ
電磁場を量子化しないとダメだろう。
388ご冗談でしょう?名無しさん:2009/04/05(日) 00:09:08 ID:???
>>386
>>387
ご回答ありがとうございます。
量子力学も基礎をかじった程度に場の量子論というやつを
今勉強しております。電磁場の量子化難しいです。
過去にも物理を学ぶ課程で何回も挫折しそうになっていますが
どうにか量子力学まで来ました。
消滅生成演算子とか繰り込みとか出てきてますが
光子の存在確率と運動量・エネルギー・シュレディンガー方程式
マックスウェルの波動方程式との絡みの説明が欲しいんですけど
そういうのは必要ないのでしょうか?
389ご冗談でしょう?名無しさん:2009/04/05(日) 00:10:29 ID:???
>>385
特殊相対論は勉強したの?
390ご冗談でしょう?名無しさん:2009/04/05(日) 00:17:10 ID:???
>>389
ええ。勉強したなどとは言えないレベルだと思いますが。
ローレンツ変換から時間の遅れ・質量からエネルギーが導かれるというのを。
391ご冗談でしょう?名無しさん:2009/04/05(日) 00:22:41 ID:3fJc7x4E
nモルの理想気体に対するヘルムホルツの自由エネルギーFは、F=nCvT−nCvT・lnT−nRT・lnVと表せますが、
微分式dF=−SdT−pdVから求めることができません。導出方法を教えていただけないでしょうか?
392ご冗談でしょう?名無しさん:2009/04/05(日) 00:45:47 ID:???
丸投げなら死ねよ?
393ご冗談でしょう?名無しさん:2009/04/05(日) 00:47:19 ID:???
>>388
>>350>>352 が理解できる程度に特殊相対論を勉強するのが先
394ご冗談でしょう?名無しさん:2009/04/05(日) 00:56:12 ID:???
>>393
そうですか。あやふやにショートカットしてるのがまずいんですね。
頑張ります。でも学業じゃないんです。興味心でして。
辛面白いですね。物理。
395ご冗談でしょう?名無しさん:2009/04/05(日) 03:41:50 ID:84XyC5yv
磁束の時間変化がボルトになるのって実験則であって
数式で証明できていないって本当ですか?
396ご冗談でしょう?名無しさん:2009/04/05(日) 03:47:39 ID:???
数式で証明って何?
物理の法則なんてみんな実験則だろ。
397ご冗談でしょう?名無しさん:2009/04/05(日) 03:48:47 ID:???
磁束の時間変化がボルトになるって何?
398ご冗談でしょう?名無しさん:2009/04/05(日) 03:57:46 ID:84XyC5yv
>397
すみません。ボルト→起電力

>396
そうですね・・・。物理ってなぜそのような法則になるのかって説明できないのですか?
399ご冗談でしょう?名無しさん:2009/04/05(日) 05:31:57 ID:???
>>398
より原理的な法則に帰納する事で説明される場合は有る。
しかしそれは別の「なぜ」に取って代わられただけとも言える。
400ご冗談でしょう?名無しさん:2009/04/05(日) 06:02:22 ID:???
物理ってのは何百何万の実験から
それっぽい法則見つけて
実際にそれでやってみたらなんとなく
うまく行ってるっぽい って学問

どんな天才も万有引力やエネルギー保存則ひとつとっても正しいかどうか分からない。
401ご冗談でしょう?名無しさん:2009/04/05(日) 07:38:28 ID:???
それで原爆まで出来てしまうんだからスゴイ学問だよな
402ご冗談でしょう?名無しさん:2009/04/05(日) 07:52:00 ID:T/5jmszd
証明じゃないけど、スクエア方程式を2つ使った過去スレで、あらわせるよ。
ヒントくらいだけど。
403ご冗談でしょう?名無しさん:2009/04/05(日) 07:54:06 ID:T/5jmszd
ナッツが、東大に送った調和理論の参考になるよ。
404ご冗談でしょう?名無しさん:2009/04/05(日) 07:55:12 ID:???
>>399
すこし別かもしれないが例えば複素数の波動関数を使えば計算うまく行くが
「なぜ複素数なのか」と理由を探せばより深く理解がすすむのでは?
405ご冗談でしょう?名無しさん:2009/04/05(日) 09:19:21 ID:Jby1/rcG
>404
そういえば、サクライに偏光と絡めて複素数の理由が書いていたような。
406ご冗談でしょう?名無しさん:2009/04/05(日) 09:23:08 ID:???
>>398
2つ前のスレで演繹と帰納に関する討論があった。
科学的思考とは、実証可能な「間違っている解釈」をふるい分けることで、
「今のところ間違っていないように見える解釈」を判定しようという方法論。

「正しい解釈」は悪魔の証明なので実証できない。
407ご冗談でしょう?名無しさん:2009/04/05(日) 09:25:43 ID:???
つまり、こじつけでも良いから、具体例がほしいと。
408ご冗談でしょう?名無しさん:2009/04/05(日) 09:35:47 ID:Jby1/rcG
ファインマンの経路積分とかチョーこじつけだな。(そんな詳しく知らないけど)
あれを初めてみたとき「どうして?」って思ったけど、
あれが成り立つ理由っていうのはわからないの?
409ご冗談でしょう?名無しさん:2009/04/05(日) 09:40:52 ID:???
お前が何も知らないことは分かる。
410ご冗談でしょう?名無しさん:2009/04/05(日) 09:44:18 ID:Jby1/rcG
なにぉ。結構前だけど、ちゃんと猪木に載ってたやつを読んだわい。
411ご冗談でしょう?名無しさん:2009/04/05(日) 09:55:45 ID:???
何なんだよこのジジイは
412ご冗談でしょう?名無しさん:2009/04/05(日) 10:33:00 ID:???
ただの経路積分だろ
何が分からないんだ
413ご冗談でしょう?名無しさん:2009/04/05(日) 12:46:34 ID:???
数学が弱くても物理を語るのが許されたのは、アインシュタインまでだ。
414ご冗談でしょう?名無しさん:2009/04/05(日) 12:50:26 ID:???
そこはファラデーだろ
415ご冗談でしょう?名無しさん:2009/04/05(日) 13:54:54 ID:???
数学が弱くてもって何を根拠にww
416ご冗談でしょう?名無しさん:2009/04/05(日) 21:55:28 ID:???
なぜか、数学が苦手だったって妙な俗説があるんだよな。wikipediaによると実際には
12才くらいで微積分を理解するくらい得意だったのに。
417ご冗談でしょう?名無しさん:2009/04/05(日) 22:01:52 ID:???
ポアンカレやヒルベルトほどはできなかったってだけだろ >アインシュタインの数学
418ご冗談でしょう?名無しさん:2009/04/05(日) 22:30:22 ID:???
でファラデーが数学苦手だったって言うのはホント?
419ご冗談でしょう?名無しさん:2009/04/05(日) 22:45:56 ID:???
ボンカレーの昼弁当なら、おれも負けないぜ。
420ご冗談でしょう?名無しさん:2009/04/05(日) 22:47:23 ID:???
>>418
>でファラデーが数学苦手だったって言うのはホント?
違います。数学はまったく勉強していないから苦手もなにも・・・
421ご冗談でしょう?名無しさん:2009/04/05(日) 22:50:37 ID:???
数学より英語が苦手なのは将来ダメ?
422ご冗談でしょう?名無しさん:2009/04/05(日) 23:00:54 ID:???
ダメです。
423ご冗談でしょう?名無しさん:2009/04/05(日) 23:06:30 ID:???
理系はある意味文系より英語できなくちゃいけない
てか、
理系=英語できない
文系=理系科目できない
の代名詞みたいに使われるのはどうにかしてほしい
424300:2009/04/05(日) 23:09:39 ID:???
>>300 はやはり的外れな質問でしたでしょうか?
425ご冗談でしょう?名無しさん:2009/04/05(日) 23:12:49 ID:???
アインシュタインは数学に苦手意識があった。もちろん一般人が近づけるレベルではない。

数学を諦めた経緯などはアインシュタイン自伝等を読めばよいだろう。
426ご冗談でしょう?名無しさん:2009/04/05(日) 23:32:02 ID:???
物理学者の人は死んだらどうなると思いますか?

物理学的には無というのは存在しないとされていますが
人の心には無というものは存在するのですか?
427ご冗談でしょう?名無しさん:2009/04/05(日) 23:33:38 ID:???
情熱大陸 ノーベル賞学者小林誠
http://live23.2ch.net/test/read.cgi/livetbs/1238941500/
428ご冗談でしょう?名無しさん:2009/04/05(日) 23:37:28 ID:???
>>426
物理学で解答できる質問ではございません。心理学あたりできいてください。
429ご冗談でしょう?名無しさん:2009/04/05(日) 23:43:30 ID:???
>>423
421ですが自分らみたいな英語苦手が文系にそのようなイメージ与えたと思っ
ています。その意味で申し訳なく思う。
430ご冗談でしょう?名無しさん:2009/04/05(日) 23:44:30 ID:???
>>426
流派によって違う
431ご冗談でしょう?名無しさん:2009/04/06(月) 00:05:17 ID:jNLgwBoi
あ〜あ、それにしても不老不死になりたいよなぁ…。
432ご冗談でしょう?名無しさん:2009/04/06(月) 00:23:17 ID:???
>>428
わかりました。
>>430
流派なんてあるんですか?

私は死んだら
現在の脳の状態を引き継いで
違う星に生まれてくると思います。
地球が階層1なら
階層2の星へ生まれてくると思います。
それが永遠に続くであります。
433ご冗談でしょう?名無しさん:2009/04/06(月) 00:29:22 ID:uQL/m3+2
Aタイプ:物理もやってるけどちゃんと信仰ももってる。
両者の矛盾?空気嫁よオマエ。

Bタイプ:長年自然科学やってたらありとあらゆる宗教
の教義なんてデタラメだってわかっちゃってる。
434ご冗談でしょう?名無しさん:2009/04/06(月) 00:39:04 ID:???
>>433
Cタイプ:真に物理を極めたものは、創造主の御心を悟り
科学=宗教=哲学=数学を統一した真の信仰を生み出す。
435ご冗談でしょう?名無しさん:2009/04/06(月) 00:41:15 ID:???
東大行った人が
すべての高校物理の問題がma=Fでとける
といっていたがよくわからないので式を含めて具体的な例をだしてほしいです
436ご冗談でしょう?名無しさん:2009/04/06(月) 00:44:40 ID:???
>>435
>東大行った人が
そいつは、生物・化学で入試受けたんだよ。
437ご冗談でしょう?名無しさん:2009/04/06(月) 08:17:06 ID:???
>>432
流派とは仏教の流派のことでは、物理と関係ない。
>私は死んだら・・
そう考えると楽しいね。
438ご冗談でしょう?名無しさん:2009/04/06(月) 08:20:22 ID:???
議論しても終わらない&板違いの話題はそろそろご勘弁願いたいです。
439ご冗談でしょう?名無しさん:2009/04/06(月) 09:09:20 ID:???
>>424
こちらはテキストなのでテフは読めない
エントロピーの式は皆知っているけどアマゾンの本の紹介もリンクが途中
で切れている。
440ご冗談でしょう?名無しさん:2009/04/06(月) 09:16:10 ID:???
300ではないが、Amazonのリンク先は以下のようだ。
http://tinyurl.com/ch74er
441ご冗談でしょう?名無しさん:2009/04/06(月) 15:05:51 ID:???
>>300
まず分子1個や2個でやってみるべき
442ご冗談でしょう?名無しさん:2009/04/06(月) 17:04:09 ID:???
443300:2009/04/06(月) 19:05:30 ID:???
>>439
最後の部分はテキストだとこうです:
< エントロピーは「質量 {M_i} 位置 {X_i} 速度 {V_i} (i = 1, 2, .., n) の粒子からなる系」に対して定義することのできるものなのでしょうか?
< 可能なら、式でいうとどうなるのでしょうか。

> エントロピーの式
こちらの理解は
「状態数の対数」→「状態数ってこの系ではどうやって数えるの?」
「dS=d'Q/T」→「温度の定義にもこう書いていてどっちが先だかわからない><」
という状態です。

Amazon のリンクは >>442 で正しいです。

>>441
1個や2個ができてたら「ここまであってる?」て書きますよ…orz
444ご冗談でしょう?名無しさん:2009/04/06(月) 19:17:59 ID:???
帰納法というものがあってだな
445ご冗談でしょう?名無しさん:2009/04/06(月) 19:42:14 ID:???
エントロピーは平衡系から
446ご冗談でしょう?名無しさん:2009/04/06(月) 23:34:15 ID:???
>>300
平衡状態をシミュレートするって意外と難しくてそれなりの技法が必要だったはず。昔
ノセとフーバーの方法というのを聞いたような。分子動力学で調べると何かわかるかも。
447ご冗談でしょう?名無しさん:2009/04/07(火) 01:31:42 ID:???
凄いアフォみたいな質問なんですが…
大きな地震があった時、空を飛んでいる鳥も落ちたって話を聞いたんですが、空を飛んでる鳥に地面の揺れが
何の影響を及ぼしたんでしょうか?それとも地震そのものとは直接影響は無く、例えば地震で発生した火災等で
気流が乱れたりして、それが原因とかなんでしょうか?そもそも地震で鳥が落ちるっていうもの自体が都市伝説なんでしょうか?

よくわからん質問でスマソ
448ご冗談でしょう?名無しさん:2009/04/07(火) 02:43:58 ID:???
>>447
よく分かりませんが
地震の時に、岩盤(花崗岩など?)に圧力がかかる
→地場が発生
→鳥は地場を感じて場所を把握しているのでビックリ

この地場の発生が地震雲を作っているって言っている人もいるらしいですけど、
まだわかっていないみたいです

449ご冗談でしょう?名無しさん:2009/04/07(火) 02:50:00 ID:???
とても…嘘くさいです…
450ご冗談でしょう?名無しさん:2009/04/07(火) 02:50:31 ID:???
地場じゃなぁ…
451ご冗談でしょう?名無しさん:2009/04/07(火) 06:17:13 ID:???
>>448
> よく分かりませんが

よくわからないのなら、そんな胡乱なことを書くな。
452ご冗談でしょう?名無しさん:2009/04/07(火) 07:22:23 ID:???
448は一般的な人の科学リテラシーでは。これは問題発言。
453ご冗談でしょう?名無しさん:2009/04/07(火) 07:24:54 ID:???
447さん
地学板で聞いてください。ここでは答えられる質問ではございません。
454ご冗談でしょう?名無しさん:2009/04/07(火) 07:41:03 ID:???
>>452
わからなくて他人に訊く、これは正しい姿勢です。
わからないのに怪しげな俗説で答えるのはアウトでしょうね。
455ご冗談でしょう?名無しさん:2009/04/07(火) 08:26:40 ID:???
>>454
ここ来て間もないけど、もちろん448はダメ。それに関して
質問する側も少しは考えてほしいね。この質問はどこですればいいのか
分野が分からないのはリテラシーの問題です。他のスレみても物理とは
関係ないスレ大杉。
456ご冗談でしょう?名無しさん:2009/04/07(火) 12:53:25 ID:???
ピエゾだから電場くらい発生してもおかしくはない
457ご冗談でしょう?名無しさん:2009/04/07(火) 13:11:42 ID:???
文字どおり飛ぶ鳥を落とす勢いで発生していたらおかしい
458ご冗談でしょう?名無しさん:2009/04/07(火) 13:15:03 ID:???
>>456  思いつきレベルの説明を真顔で語られても…
459ご冗談でしょう?名無しさん:2009/04/07(火) 13:20:19 ID:???
そもそもホントに鳥が落ちたってのかい?
460ご冗談でしょう?名無しさん:2009/04/07(火) 13:20:40 ID:L3GdQzvi
糖尿病のワクチンは、砂糖黍の成分にあるかも。
461ご冗談でしょう?名無しさん:2009/04/07(火) 13:28:40 ID:???
糖尿病は伝染病じゃないだろうに。
ワクチンじゃなく、特効薬と書きたいのか。
462ご冗談でしょう?名無しさん:2009/04/07(火) 14:42:06 ID:???
思いつき?そういう研究をしている人もいるのだが
463ご冗談でしょう?名無しさん:2009/04/07(火) 14:56:13 ID:???
そういう考えの人もいるということだね。

で、それは説明になっているとは言い難いのだが。
464ご冗談でしょう?名無しさん:2009/04/07(火) 15:39:04 ID:???
そういう研究をしている人もいるという程度でいいなら、
ほとんどありとあらゆるヨタ話は思いつきレベルを脱している。
465ご冗談でしょう?名無しさん:2009/04/07(火) 16:51:44 ID:dD1XKP2f
466ご冗談でしょう?名無しさん:2009/04/07(火) 16:58:26 ID:dD1XKP2f
467ご冗談でしょう?名無しさん:2009/04/07(火) 20:49:15 ID:VAo8BL2y
例えば、コインを10回投げて、4回表が出て、6回裏が出たとする。
この実験結果:4回とか6回に対して誤差を評価できるのだろうか?
468ご冗談でしょう?名無しさん:2009/04/07(火) 21:03:00 ID:???
>>465>>466
スレチのジジイ向けクソ動画を二回も貼るな。著作権もシカトだし。
469ご冗談でしょう?名無しさん:2009/04/08(水) 00:20:35 ID:???
なんか勘違いしてるみたいだけど岩石に応力を加えたらどうなるかを
研究してる人がいるのであって鳥がどうのというのとは関係無い
470ご冗談でしょう?名無しさん:2009/04/08(水) 00:21:29 ID:dQF+E0lw
いきなり太陽がなくなったら真っ暗になる前に地球が公転をやめますか?
暗くなってから公転をやめますか?
471ご冗談でしょう?名無しさん:2009/04/08(水) 00:26:20 ID:???
いきなり太陽がなくなるということは無いので考えるだけ無駄です。
472ご冗談でしょう?名無しさん:2009/04/08(水) 01:42:00 ID:???
>>458
別に思い付きじゃないだろ
地震のときにピエゾ効果で発生した電磁波のノイズとか

実験室では岩石に圧力かけて破壊が生じるときにそれを観測したりしてるし
473ご冗談でしょう?名無しさん:2009/04/08(水) 01:57:08 ID:???
>>471
>いきなり太陽がなくなるということは無いので考えるだけ無駄です。
LHCで造られたミニブラックホールが太陽に命中して、太陽を一瞬で
飲み込んでしまえばどうですか?
474ご冗談でしょう?名無しさん:2009/04/08(水) 02:12:17 ID:???
マイクロブラックホールが太陽を一瞬で飲み込むことなどできないし、一瞬で飲み込んだとしても
太陽の質量はそのままそこに残るのだから重力は(しばらくは)そのままだ。
475ご冗談でしょう?名無しさん:2009/04/08(水) 02:14:33 ID:???
>>474
LHCで造られたマイクロブラックホールは、ほとんど光速度だから
太陽を飲み込んで光速度で太陽系から一瞬で飛び出してしまうだろ?
476ご冗談でしょう?名無しさん:2009/04/08(水) 02:21:55 ID:???
光速に近くても大した運動量は持っていない。
素通りするか、ぶつかって止まるだけ。
477ご冗談でしょう?名無しさん:2009/04/08(水) 02:51:47 ID:???
太陽を一瞬で吹っ飛ばすほどの運動量を持ったBHが仮に出来たとして、それは地球から
発射されるんだから当然反作用で地球のほうが先に吹っ飛ぶ。

運動量保存則くらい理解しとけ。
478ご冗談でしょう?名無しさん:2009/04/08(水) 02:52:34 ID:???
あ、>>477>>475に対して言ってる。念のため。
479ご冗談でしょう?名無しさん:2009/04/08(水) 04:10:26 ID:???
質問します。
地球の直径より20q大きい直径を持つ鉄の輪っか(リング)があったとします。
このリングは地上10qの地点に静止するんでしょうか?と言うか、見た目が土星のリングみたいになるのでしょうか?
480ご冗談でしょう?名無しさん:2009/04/08(水) 06:19:40 ID:???
安定じゃないからすぐ落下する
481ご冗談でしょう?名無しさん:2009/04/08(水) 06:44:09 ID:???
>>479 鉄の輪っかは剛体じゃないから、全長に渡って少しだけ圧縮されて落下する。

地球一周が40000kmとすると0.05%圧縮されるだけ。
そのスケールの物体はどんな材料で作っても、流体みたいなもんだよ。
482ご冗談でしょう?名無しさん:2009/04/08(水) 06:46:59 ID:???
>>472 それで鳥が落ちるのか?元々の議題がわかってないじゃないのか。
483ご冗談でしょう?名無しさん:2009/04/08(水) 06:55:01 ID:???
>>482
それは生物学とか獣医学の問題で、物理で取り扱える問題ではないだろ
484ご冗談でしょう?名無しさん:2009/04/08(水) 07:01:44 ID:???
>>481
20qにしたのはエベレストの高さを考慮しただけで、200qでも構いません。
485ご冗談でしょう?名無しさん:2009/04/08(水) 07:34:02 ID:???
>>484 同じこと。至る所でフニャフニャに落下する。

逆を考えてみよう。地表の凹凸は考えずに、地球一周する鉄のレールを設置する。
設置した温度から30度あげると、熱膨張で鉄は0.05%延びる。
これで、全体が10km浮く?いや、至る所で飴のように曲がるだろ?
486ご冗談でしょう?名無しさん:2009/04/08(水) 07:46:20 ID:???
つーか剛体でもすぐ落下する
487ご冗談でしょう?名無しさん:2009/04/08(水) 07:58:32 ID:???
SFネタで恐縮だが、リングワールドが中心恒星に対して不安定というのは、
続編以降の読者なら既知。
488ご冗談でしょう?名無しさん:2009/04/08(水) 08:08:31 ID:???
>>483
それは最初の質問者に向けて言うべきだろ
489ご冗談でしょう?名無しさん:2009/04/08(水) 08:10:50 ID:???
>>487対称性の自発的破れ!!ww
490ご冗談でしょう?名無しさん:2009/04/08(水) 08:21:44 ID:???
>>489 単なるニュートン力学で説明可能。
491ご冗談でしょう?名無しさん:2009/04/08(水) 08:54:36 ID:???
回せよ!
492ご冗談でしょう?名無しさん:2009/04/08(水) 08:54:37 ID:???
>>489
>>489
wwwwwww
493ご冗談でしょう?名無しさん:2009/04/08(水) 09:06:13 ID:RT43B6az
男同士で、子供できるよ 2nnより。
494ご冗談でしょう?名無しさん:2009/04/08(水) 09:13:02 ID:???
>>477
>太陽を一瞬で吹っ飛ばすほどの運動量を持ったBHが仮に出来たとして
ブラックホールはなんでも吸い込みますが、吹っ飛ばすなぞしません。
ちゃんと勉強してからカキコしてください。
495ご冗談でしょう?名無しさん:2009/04/08(水) 10:40:03 ID:???
>>469
勘違いねぇ。>>447で始まった話題が続いているときに、
>>456が出てきたら、普通は>>447の裏付けのつもりで
書かれたと思うよ。何の脈絡もなくピエゾの話を始めた
のだと解釈するほうが無理筋。それでもあくまで>>447とは
何の関係もなく唐突に>>456を書いたと主張したいのなら、
紛らわしいことすんなという批判くらいは甘んじて受けろな。

>>472
ピエゾ自体を思いつきと言ってるんじゃなくて、それを
定量的な考察もなく安易に地震雲や動物行動異常などの
宏観現象に結びつける姿勢が思いつきだっての
496ご冗談でしょう?名無しさん:2009/04/08(水) 10:42:08 ID:???
>>487
鉄の輪っかがぐるぐる回ってりゃ落っこちることないんじゃないの?
497ご冗談でしょう?名無しさん:2009/04/08(水) 10:53:15 ID:???
>>496 動いていようが重力は働くの。人工衛星が落ちてこない原理とは違う。
498ご冗談でしょう?名無しさん:2009/04/08(水) 11:00:05 ID:???
>>497
回ってれば曲がりはしないよ
理解できないなら適当に積分でもして計算してくれ
499ご冗談でしょう?名無しさん:2009/04/08(水) 11:04:59 ID:???
>>498
おまえバカだろ。質点の周りの円環は、重力的に不安定だって。
ちょいと擾乱を与えれば、中心からずれる。
500ご冗談でしょう?名無しさん:2009/04/08(水) 11:12:42 ID:???
バカはお前だろ。
501ご冗談でしょう?名無しさん:2009/04/08(水) 11:15:14 ID:???
輪っかが剛体なら回転しててもしてなくても不安定、というのは常識としてOKなのだが、
では、剛体でないならどうかというと、誰か知っている人はいる?

例えば、土星の輪が質点の離散的な集合ではなく、流体の連続的な集合だった場合、
輪が土星に向けて落下するとはあまり思えない。
そして、巨大な固体は流体と変わらないという指摘は>>481の通り。
どうなるんだろう?

502ご冗談でしょう?名無しさん:2009/04/08(水) 11:15:20 ID:???
>>499
曲がりはしないって書いたんだけど
落ちないなんて書いてないけど
って言うか「曲がりはしない」って言葉に「安定もしない」ってニュアンスを含ませたつもりだったんだけど
503501:2009/04/08(水) 11:33:16 ID:???
ごめん。流体なら宇宙に飛び散ったり中心星に降り注いだりして
やがて輪っかは消滅しそうだ。。
伸縮する弦ならどうなるんだろ?
504ご冗談でしょう?名無しさん:2009/04/08(水) 12:39:05 ID:???
>地球一周が40000kmとすると0.05%圧縮されるだけ。
どうでもいいけど20km大きいのは周長ではなくて直径なので、
周長の変化率としてはそのπ倍だね
505ご冗談でしょう?名無しさん:2009/04/08(水) 12:44:12 ID:???
>>504 おう、そうだった。量的には問題だが、話の本質はほぼそのままなので、読み替えてご容赦願いたい。
506ご冗談でしょう?名無しさん:2009/04/08(水) 14:47:41 ID:???
>>503  流体が環状の流れを持続できるとは思えない。弦だとしても剛体と同じ。
回転中心が徐々にずれて、どこかが中心天体表面なり大気なりに接触してジ・エンド。

最初の部分だけ、ちょっと補足。流体の流れと言っても、流速にあわせた系から見たら、
流体が細長い形状で静止している。断面積と比較すると、無限に長い棒のような形状。
これは、現実の流体では無理。やがて表面張力で、粒子に細分される。
507ご冗談でしょう?名無しさん:2009/04/08(水) 14:49:15 ID:???
教えてください。

宇宙では息ができないので、頭になんか被ってますよね?
そんな宇宙で素肌を出したらどうにかなるんですか?
508ご冗談でしょう?名無しさん:2009/04/08(水) 14:57:12 ID:???
モイスチャーを失ってカサカサになるし、太陽交戦を直接浴びたら被爆するし焦げる(?)
509ご冗談でしょう?名無しさん:2009/04/08(水) 16:09:41 ID:???
>>506
返事ありがとう。
流体については、言われてみれば確かに
円筒表面は極小曲面じゃないから細分化そうだ。

でも、弦についてはまだ良く分からない。
円形の弦があってその回転中心が少しずれたとして、
なおも弦が伸縮せずに円形を保つなら
剛体と変わるところはないだろうけど……
510ご冗談でしょう?名無しさん:2009/04/08(水) 16:16:00 ID:???
お舞ら落ち付け。
遠心力=中心力で釣り合って安定に決まってるじゃん。
511ご冗談でしょう?名無しさん:2009/04/08(水) 16:32:35 ID:???
何だその論理
512507:2009/04/08(水) 16:56:38 ID:???
>>508さん

ありがとうございます。
宇宙服って防寒着みたいだから
寒いのかと思ってたけど、
太陽光線ってことは暑いのかな?
とにかく宇宙に放りだされないようにしよう。

みなさんパタリロって見たことありますか?
それにスターダスト計画っていうのがあるんですけど
本当にそんなことできるのでしょうか?
宇宙空間にダイヤモンドを持ってきて
それを地球に落とすと大量の雹が降ってくる。
そんな説明でした。
513ご冗談でしょう?名無しさん:2009/04/08(水) 17:29:49 ID:???
>>512 
ダイヤは800℃で燃える。現在ある最大のダイヤでも跡形もなくCO2に成り果ててしまう。
514ご冗談でしょう?名無しさん:2009/04/08(水) 17:56:57 ID:???
>>512
暑いときは暑いし寒いときはめっちゃ寒い
515ご冗談でしょう?名無しさん:2009/04/08(水) 18:08:26 ID:???
表面張力の原因(もとになっている力)って
なんなんですか?分子間に作用する斥力?引力?
516ご冗談でしょう?名無しさん:2009/04/08(水) 18:09:02 ID:???
つ 万有引力
517ご冗談でしょう?名無しさん:2009/04/08(水) 18:39:18 ID:???
>>515
ファンデルワールス力、その大元は電磁気力。
518ご冗談でしょう?名無しさん:2009/04/08(水) 18:41:56 ID:???
水の場合は水素結合の力の方が強いか。
519ご冗談でしょう?名無しさん:2009/04/08(水) 18:46:03 ID:???
もしかして、それ、強い相互作用って奴?
520ご冗談でしょう?名無しさん:2009/04/08(水) 18:49:20 ID:???
分子間では強い相互作用は無視できるレベル。
521ご冗談でしょう?名無しさん:2009/04/08(水) 18:49:23 ID:???
じぇんじぇんちがう
522ご冗談でしょう?名無しさん:2009/04/08(水) 19:02:18 ID:???
コイントスをすると投げる前に見える面の方がほんの少しだけ有利というのをある本で読みました。
それによると慣性の力が働いているからだそうなんですが、分かりやすい説明をお願いします。
523ご冗談でしょう?名無しさん:2009/04/08(水) 19:04:09 ID:???
>>522 ウソ
524ご冗談でしょう?名無しさん:2009/04/08(水) 19:06:34 ID:???
>>523
>ウソ
kwsk
525ご冗談でしょう?名無しさん:2009/04/08(水) 19:29:09 ID:BcLN1sIK
本当である理由が無いから。
526ご冗談でしょう?名無しさん:2009/04/08(水) 19:39:10 ID:???
>>524 そんな説明で納得する方がどうかしている。
「慣性の力が働いているから見えてる面の反対側が出る」と説明されても納得するの?
527ご冗談でしょう?名無しさん:2009/04/08(水) 22:06:42 ID:???
>>490
別にニュートン力学で系の対称性が自発的に破れたっていいだろうが〜
528ご冗談でしょう?名無しさん:2009/04/08(水) 22:29:22 ID:???
>>527 怒らないから、ちゃんと説明してみなさい。
529ご冗談でしょう?名無しさん:2009/04/08(水) 22:55:27 ID:???
>>494
> ブラックホールはなんでも吸い込みますが、吹っ飛ばすなぞしません。

お前BHのこと何にも知らんのだろ。
530ご冗談でしょう?名無しさん:2009/04/08(水) 23:02:42 ID:???
>>529
いや、ブラックホールは吹っ飛ばすなぞしないと思うが。
531ご冗談でしょう?名無しさん:2009/04/08(水) 23:10:48 ID:???
太陽を一瞬で吹っ飛ばすほどの運動量と書いただけなんだが。
532ご冗談でしょう?名無しさん:2009/04/08(水) 23:11:52 ID:???
どうも>>494はBHに飲み込まれるとその質量が無くなると思ってるとしか思えん。
533ご冗談でしょう?名無しさん:2009/04/08(水) 23:35:27 ID:???
おまいら、つまらん揚げ足取りでつぶす時間があったら勉強するなりエロビデオ見るなりしろよ。。。
534ご冗談でしょう?名無しさん:2009/04/08(水) 23:42:21 ID:???
確かに数学や物理を勉強するのに射精するのは重要。
偉大な物理学者、数学者は射精回数が多い。
535ご冗談でしょう?名無しさん:2009/04/08(水) 23:54:48 ID:???
揚げ足取りじゃなくてさ

飲み込まれようがなんだろうが、太陽の質量は消えてなくて、それがそっくり光速に近い
速度で吹っ飛んでいくんだろ。

>>477の結論は変わらん。
536ご冗談でしょう?名無しさん:2009/04/08(水) 23:58:51 ID:???
>>522
毎回「表」が見えたとしても、表が出る結果は1/2に収束する。
537ご冗談でしょう?名無しさん:2009/04/09(木) 00:06:48 ID:???
>>522
慣性なぞ関係ないが、表を向いているほうが有利ではある。
場合分けを考えると
0.5回転以内 : 表 : 表1/1
1.0回転以内 : 表裏 : 表1/2 裏1/2
1.5回転以内 : 表裏表 : 表2/3 裏1/3
2.0回転以内 : 表裏表裏 : 表1/2 裏1/2
2.5回転以内 : 表裏表裏表 : 表3/5 裏2/5
という組み合わせになるから、表の出る確率のほうが高い。
538ご冗談でしょう?名無しさん:2009/04/09(木) 00:11:18 ID:???
>>537
回転が止まるまでの回転数の分布に拠る。
コインの回転が 0.5回以内で止まるような条件でコイントスをするというのは考えにくい。
539ご冗談でしょう?名無しさん:2009/04/09(木) 00:14:59 ID:???
初期条件がある分だけ確率としては、ずれるだろう。
収束するのは異論は無いが。
540ご冗談でしょう?名無しさん:2009/04/09(木) 00:16:03 ID:???
>>537
ワロス ナルホドww
541ご冗談でしょう?名無しさん:2009/04/09(木) 00:25:22 ID:???
訓練すれば制御できるはず >コイントス
542ご冗談でしょう?名無しさん:2009/04/09(木) 00:27:55 ID:???
そもそもそのコイントスって地面に落とすの?手の甲でキャッチするの?
543ご冗談でしょう?名無しさん:2009/04/09(木) 00:34:19 ID:???
それはギャグか?
544ご冗談でしょう?名無しさん:2009/04/09(木) 00:40:55 ID:???
(・、.)?
545ご冗談でしょう?名無しさん:2009/04/09(木) 07:41:26 ID:???
>>539
> 初期条件がある分だけ確率としては、ずれるだろう。
> 収束するのは異論は無いが。

偏りがあることに異論はないが、どちらに偏るかが自明ではない。
>>537なんかは論外だが。

ちなみに、バターを塗ったトーストが、バターの面を下にカーペットに落ちるのは、
床までにちょうどそれだけ回転するからという記事が、以前サイエンティフィック・
アメリカンに載ったことがあったように覚えている。

これなんかは偏りがちょうど0.5回転あたりに偏っているからなんだが。

初期条件と反対側に偏ることだってあるのよ。
546ご冗談でしょう?名無しさん:2009/04/09(木) 08:50:46 ID:???
>>545
>バターを塗ったトースト
表と裏の形状・密度は同じじゃない。イカサマ賽子と同じね。
547ご冗談でしょう?名無しさん:2009/04/09(木) 09:10:01 ID:???
>>546
バターを塗らなくても同じ結果です。
548ご冗談でしょう?名無しさん:2009/04/09(木) 09:19:10 ID:???
マーフィーの法則を検証した暇人がいるのか
549ご冗談でしょう?名無しさん:2009/04/09(木) 09:36:57 ID:???
あった。
日本版の日経サイエンスだと97年7月号「マーフィーの法則を科学する」。
550ご冗談でしょう?名無しさん:2009/04/09(木) 10:03:04 ID:???
うろ覚えだけど、3/2回転して着地じゃなかった?

後半の、知的生命体がどうのこうのっていう議論は下らなかった。
551ご冗談でしょう?名無しさん:2009/04/09(木) 11:41:01 ID:???
空間は三次元。仮に無限遠方まで飛ぶ鉄砲があるとする。
射手の位置を原点とし、左右をx軸、頭から足の方向をy軸、前後方向をz軸とする。
と、射手はx,yの照準を合わせるだけで、前方の全ての空間に弾を飛ばすことが
できる。
z方向の照準をあわせなくてもいい理由を教えてくれ。

552ご冗談でしょう?名無しさん:2009/04/09(木) 12:00:59 ID:???
設定が意味不明
553ご冗談でしょう?名無しさん:2009/04/09(木) 12:17:21 ID:???
モンキーハンティング!?
554ご冗談でしょう?名無しさん:2009/04/09(木) 12:47:45 ID:hedm4fAK
>>551
弾道が線だから
555ご冗談でしょう?名無しさん:2009/04/09(木) 13:07:01 ID:???
バターの方が密度が大きいからだろ
556ご冗談でしょう?名無しさん:2009/04/09(木) 13:12:21 ID:???
>>555 >>547読めないのか。
557ご冗談でしょう?名無しさん:2009/04/09(木) 14:19:15 ID:KvmPmh3V
数学がどの程度必要なのかわかりません。
解析学とかがっつりやったほうが良いのでしょうか?
558ご冗談でしょう?名無しさん:2009/04/09(木) 14:25:02 ID:???
>>551
3次元直交座標だとZも合わせないと目標に飛ばないよ。
559ご冗談でしょう?名無しさん:2009/04/09(木) 16:14:36 ID:???
Zなんてどう合わせるんだ?
560ご冗談でしょう?名無しさん:2009/04/09(木) 16:20:52 ID:???
時限信管
561ご冗談でしょう?名無しさん:2009/04/09(木) 17:59:14 ID:???
>>559
3次元直交座標だとx.y座標だけなら方向さえ決まらないだろ。
562ご冗談でしょう?名無しさん:2009/04/09(木) 18:12:31 ID:F479kaml
>>561
弾道が直線ならz座標はいらないだろ
放物線を描くときはまた別の話
563ご冗談でしょう?名無しさん:2009/04/09(木) 18:50:17 ID:MCZSi3sv
>>561
三次元直交座標で合わせる鉄砲なんかどこに存在するの?
564ご冗談でしょう?名無しさん:2009/04/09(木) 19:02:28 ID:???
>>551 意味がわからない。x,yの照準なんか決められない。照準できるのは角度のみ。

角度で勘違いされるとなんなので、正しくは方向余弦。
565ご冗談でしょう?名無しさん:2009/04/09(木) 19:08:02 ID:???
>>562
重力ありで放物線軌道だとしても、それを考慮してxy照準をしかるべく合わせればいい罠
566ご冗談でしょう?名無しさん:2009/04/09(木) 19:30:07 ID:???
>>562
3軸直交座標の意味わかってる?
>>563
質問者が指定してんだからそっちに聞いてくれ。
567ご冗談でしょう?名無しさん:2009/04/09(木) 19:33:28 ID:F479kaml
>>551
>前方の全ての空間に弾を飛ばすことができる

この前提があるから外力は無しってことだろ
だとしたら質問の意味がわからない
568ご冗談でしょう?名無しさん:2009/04/09(木) 20:38:02 ID:???
>>545
「論外」の根拠が無いぞ。>>537 は 0.5 回転ごとに飛距離を延ばしてみて、それ以下の
距離での出る面を場合わけしてみたもの。
表スタートという初期条件から、表が出る場合は裏が出る場合の数より少なくなることは
ありえないことを表している。
従って表が出る確率は、裏が出る確率より少なくなることはありえない。
ということを示している。どこらへんが「論外」なんだ?
569ご冗談でしょう?名無しさん:2009/04/09(木) 20:57:01 ID:???
コイントスの出る面って、空中で回転した回数によって決まるのか?
570ご冗談でしょう?名無しさん:2009/04/09(木) 20:58:46 ID:???
>>568
> 「論外」の根拠が無いぞ。

しょっぱなからだ。見えている面の定義すらされていないのが最初の難点。
表と裏を入れ替えても通じるではないか。

第二、コイントスで一桁以下の回転とかあるか。常識くらい持て。
571ご冗談でしょう?名無しさん:2009/04/09(木) 21:00:38 ID:???
>>568
> 0.5回転以内 : 表 : 表1/1
最高点で0.25回転になるように投げ上げたら落ちた時は0.5回転=裏になるんでは?
572ご冗談でしょう?名無しさん:2009/04/10(金) 02:46:16 ID:pT2vz0YV
経路積分は大学何年生で習うのですか??
573ご冗談でしょう?名無しさん:2009/04/10(金) 03:04:17 ID:NKsc0SWP
鏡で写る像は、本当に左右が反転しているの?
中学校のときに「前後が反転する」とテストに書いて、珍回答として晒されて以来の
トラウマなんだけれど……
574ご冗談でしょう?名無しさん:2009/04/10(金) 03:37:10 ID:???
>>573
おまいが正しい
575ご冗談でしょう?名無しさん:2009/04/10(金) 03:38:49 ID:???
左右反転と認識されるのは人間の脳の問題だね。
576ご冗談でしょう?名無しさん:2009/04/10(金) 05:12:36 ID:???
滅茶苦茶馬鹿にしてやるからその珍教師の名前晒してみたら?ww
577ご冗談でしょう?名無しさん:2009/04/10(金) 06:22:31 ID:???
>>575
> 左右反転と認識されるのは人間の脳の問題だね。

右手系と左手系の座標系の変換だろ。数学的なモノで脳とは関係ない。
要は >>573で正しい。
578ご冗談でしょう?名無しさん:2009/04/10(金) 07:01:18 ID:???
>>573
お前が正しい
579ご冗談でしょう?名無しさん:2009/04/10(金) 07:04:40 ID:???
>>577
> 右手系と左手系の座標系の変換だろ。数学的なモノで脳とは関係ない。

えぇ? 「左右反転と認識されるのは」習慣的なモノで脳と関係あるっしょ
580ご冗談でしょう?名無しさん:2009/04/10(金) 07:34:53 ID:???
>>572
大学による。




つーかマルチ死ね。
581ご冗談でしょう?名無しさん:2009/04/10(金) 09:24:27 ID:???
>>573
鏡は鏡面に直行する座標を反転させる装置だから、真横を向いて鏡の前に立ったら左右反転と言えなくもない
ようは上下、前後、左右の定義の仕方による違い。したがってその教師はアホ
582ご冗談でしょう?名無しさん:2009/04/10(金) 09:26:40 ID:???

訂正 ×座標 ○座標軸
583ご冗談でしょう?名無しさん:2009/04/10(金) 09:27:44 ID:???
>>573
例えば鏡を床に置いてその上に立ってみたらどうなると思う?
584ご冗談でしょう?名無しさん:2009/04/10(金) 11:01:41 ID:???
座標軸の問題は結構、難しいよ。
585ご冗談でしょう?名無しさん:2009/04/10(金) 11:03:08 ID:???
あと、10年、20年で、世界の原油は、足りなくなりますが、新エネルギーとしては、何を考えていますか?
水素ですか?太陽光パネルですか?
586ご冗談でしょう?名無しさん:2009/04/10(金) 11:18:00 ID:???
40年前からずっと、「あと10年でなくなる」と、
言われ続けているかな〜
587ご冗談でしょう?名無しさん:2009/04/10(金) 11:18:56 ID:???
原子力と石炭
588ご冗談でしょう?名無しさん:2009/04/10(金) 11:41:29 ID:m4Kawi+9
形状記憶合金の完全に曲がったところの分子を使って、何かできませんか?
589ご冗談でしょう?名無しさん:2009/04/10(金) 11:56:06 ID:???
>>585
揚げ足とるみたいだけど水素はエネルギー源ではないよ
自然界に水素分子は存在しないので他のエネルギー源を使って生成しなければならない

長いスパンで見たら核融合じゃないかな
590ご冗談でしょう?名無しさん:2009/04/10(金) 12:02:26 ID:???
水素はエネルギー通貨みたいなもん
591ご冗談でしょう?名無しさん:2009/04/10(金) 13:32:38 ID:???
水素が環境にやさしい新エネルギーとはとても思えん。
589の言うとおり温暖化を加速させる。
592ご冗談でしょう?名無しさん:2009/04/10(金) 13:39:02 ID:I/vyz73j
既出かも知れませんが、教えて下さい、超音速で移動している時、後ろからの音は聞こえないと想像出来るのですが、前方からの音はどう聞こえるのですか?
593ご冗談でしょう?名無しさん:2009/04/10(金) 13:46:04 ID:REoplOrC
>>589
最近、核融合の話題が少ないんだが、実用化に向けて少しは前進してるんだろうか?
594ご冗談でしょう?名無しさん:2009/04/10(金) 13:47:02 ID:???
>>589 長いスパンで見ると核融合も、地球上で発生しない熱エネルギーを
大気に加えるから、熱汚染を引き起こすわな。
当面問題になってる温室効果よりは現状ではさして大きくないけど。

>>591 >>589はそんなことを言っていない。曲解しまくり。

水素は中間生成物なんだから、その生成の過程で温暖化に寄与しなければok。
まあ、最終生成物のH2Oも温室効果皆無じゃないけどCO2なんかより遙かにマシ。
595ご冗談でしょう?名無しさん:2009/04/10(金) 13:48:46 ID:???
>>592 衝撃波面で遮断される。
596ご冗談でしょう?名無しさん:2009/04/10(金) 13:55:49 ID:I/vyz73j
なにも聞こえないと言う事でokですか?
597ご冗談でしょう?名無しさん:2009/04/10(金) 14:02:33 ID:???
>>596
衝撃波は移動体の前方に形成されるから、その解釈でも間違ってはいない。

ただし音を拾おうとしても、そもそも集音装置が機能しない。
民生用のマイクレベルなら楽勝で破壊される。
598ご冗談でしょう?名無しさん:2009/04/10(金) 15:05:35 ID:???
光触媒みたいなもので発生させれば完璧だな
599ご冗談でしょう?名無しさん:2009/04/10(金) 16:04:34 ID:???
>>591 >>590みたいにエネルギーを貯蔵する手段として水素が研究されてる
夜間の余剰電力で水素をプラントで大量に生成するみたいな

>>593 たぶん今年だったと思うけどアメリカでレーザー核融合の点火実験が始まるよ
実用化はどんだけ早くても50年はかかりそうって教授が言ってた
今の若い技術者が頑張らないといけないね
600ご冗談でしょう?名無しさん:2009/04/10(金) 16:12:12 ID:ULkRtgeU
質問です
宇宙は空間が膨張しているので銀河同士は遠ざかっているそうですが
空間ごと膨張するならば銀河自身も膨張するので相対的な位置関係は変わらず銀河間の距離も観測できないように思えます
僕は何を勘違いしているのでしょうか?
601ご冗談でしょう?名無しさん:2009/04/10(金) 16:14:43 ID:???
太陽光利用の水素大量生成技術ができれば
もはや核融合を無理して実用化しなくていいような
602ご冗談でしょう?名無しさん:2009/04/10(金) 16:15:08 ID:???
>>599
水素はかさばるし、沸点が低いから貯蔵・輸送に適してない。
オルト水素・パラ水素の問題もあるし。

メタンやプロパンなどに転換した方が使い勝手がよい。
603ご冗談でしょう?名無しさん:2009/04/10(金) 16:19:38 ID:???
>>600
sci.physics FAQ より
http://math.ucr.edu/home/baez/physics/Relativity/GR/expanding_universe.html
`If the universe is expanding, does that mean atoms are getting bigger?
Is the Solar System expanding?'
604ご冗談でしょう?名無しさん:2009/04/10(金) 16:19:48 ID:???
>>600
>僕は何を勘違いしているのでしょうか?
科学者が何か勘違いをしているのかもしれないよ。
605ご冗談でしょう?名無しさん:2009/04/10(金) 16:21:42 ID:???
>>603
僕の中学英語では読めません。
翻訳お願いします。
606ご冗談でしょう?名無しさん:2009/04/10(金) 16:26:19 ID:???
じゃここだけ

..., nor even the galaxy, is expanding.
銀河でさえ膨張していない
607ご冗談でしょう?名無しさん:2009/04/10(金) 16:36:31 ID:???
>>600
銀河系や太陽系を構成する天体は、互いに万有引力でまとまっている。
ハッブルの法則は、遠く離れて重力の影響が無視できる銀河に適用される。

ちなみに、銀河系とアンドロメダ星雲は近いので重力で影響し合っている。
離れるどころか、光速の0.1%くらいの速度で近づいており、そのうち合体すると
予想されている。

>>605
英語はできるに越したことはないから、あまり他人に頼らない方がよい。
それでも待てないならgoogleやyahooの翻訳サービスを使ってはどうだろうか。

http://translate.google.co.jp/translate_t?hl=ja#
http://honyaku.yahoo.co.jp/
608ご冗談でしょう?名無しさん:2009/04/10(金) 16:43:45 ID:???
辞書を引く情熱も無いのならばあきらめろ
609ご冗談でしょう?名無しさん:2009/04/10(金) 17:16:34 ID:???
諦めました.ごめんなさい。ぺこり
610ご冗談でしょう?名無しさん:2009/04/10(金) 19:22:32 ID:ifxsa1lp
形状記憶合金の完全に曲がったところの分子を使った
                        特許
611ご冗談でしょう?名無しさん:2009/04/10(金) 19:26:08 ID:???
合金に分子なんてあるの?
612ご冗談でしょう?名無しさん:2009/04/10(金) 19:45:46 ID:???
>>611
常駐してる基地外だから相手しないで。
613ご冗談でしょう?名無しさん:2009/04/10(金) 20:07:18 ID:???
重低音を出すにはスピーカーは大きくならざるを得ないの?
携帯の着信音で重低音だすのは技術的にムリ?物理的にムリ?
614ご冗談でしょう?名無しさん:2009/04/10(金) 20:49:29 ID:???
スピーカーが小さく・波長が長くなるほどエネルギー密度は低くなるから、小さなスピーカーだと音が小さくて聞こえなくなる。
また指向性も落ちる。
615ご冗談でしょう?名無しさん:2009/04/10(金) 22:09:55 ID:???
>>614
やっぱりそうなのか・・・。なんかうまい方法ないかな
着メロでズンズン響いたらDQNに馬鹿売れなのに
616ご冗談でしょう?名無しさん:2009/04/10(金) 22:17:28 ID:js/xU6OH
初歩的な質問ですみません。

高さh(x)が水平距離xの関数として、h(x)=h。(1+cos(x/k))と表される坂道を質量mのボールが0≦x≦πの範囲で落下する。摩擦力を無視し、時刻t=0でボールの速度v=0、h。=0、k=1重力加速度とする。

x1=π/6、x2=π/2、x3=πでの速度v1、v2、v3、加速度a1、a2、a3を求めよ。

という問題で、高さが距離の関数になっているので、いまいちどうやって微分をすればいいかわかりません。
どなたか解き方を教えてください。
617ご冗談でしょう?名無しさん:2009/04/10(金) 22:29:33 ID:???
>>616
h。=0 という条件だから常に h(x)=0 で、 従ってボールは永久に停止したままじゃないのか。
そうでない場合でも、時刻t=0 の時のボールの位置が与えられていないから v1、v2、v3 は
求まるはずがないし、k=1重力加速度というのも謎だし。
618ご冗談でしょう?名無しさん:2009/04/10(金) 22:29:51 ID:???
イヤホンで聴けばいい
619ご冗談でしょう?名無しさん:2009/04/10(金) 22:32:06 ID:???
>>616
はなからおかしいやん。k=1重力加速度では、cos(x/k)が定義できない。
kの単位は長さでないと。
620617:2009/04/10(金) 22:37:04 ID:js/xU6OH
すみません。
kは単位がmで、重力加速度はgです。h。はt=0のとき0のようです。
621ご冗談でしょう?名無しさん:2009/04/10(金) 22:41:47 ID:???
>>616
まず問題文を落ち着いて読み直せ。>>617 が言うように、このままでは問題が意味をなさない。
622ご冗談でしょう?名無しさん:2009/04/10(金) 22:47:23 ID:jfHrJnqN
エアコンについて!
例えば夏場、エアコン室内機室外機共に室内で動かしたらどうなるんでしょうか??涼しくなるのか…はたまた暑くなるのか(・ω・)
623ご冗談でしょう?名無しさん:2009/04/10(金) 22:47:55 ID:js/xU6OH
すみません!所々打ち間違えていました。

高さh(x)が水平距離xの関数として、h(x)=h。(1+cos(x/k))と表される坂道を質量mのボールが0m≦x≦πmの範囲で落下する。摩擦力を無視し、x=0m、時刻t=0sでボールの速度v=0m/s、h。=1m、k=1m、重力加速度gとする。

x1=π/6、x2=π/2、x3=πでの速度v1、v2、v3、加速度a1、a2、a3を求めよ。

です。
624ご冗談でしょう?名無しさん:2009/04/10(金) 23:00:30 ID:???
>>594
人間が発生させるエネルギーなんかたいした量じゃねえよ…
625ご冗談でしょう?名無しさん:2009/04/10(金) 23:10:57 ID:???
>>623
不安定ではあるが、頂上で止まったままのような。
626ご冗談でしょう?名無しさん:2009/04/10(金) 23:19:27 ID:???
>>622
冷房は電力を消費して室内の熱を室外に捨てる。暖房はまったくその逆
その際の電気エネルギー(消費電力)は熱エネルギーへと変化して室内の熱と一緒に大気へ
あとは分かるな?
627ご冗談でしょう?名無しさん:2009/04/10(金) 23:41:55 ID:???
>>623
簡単じゃねえの。
h(0)=2h。
h(x1)=h。(1+√3/2)
h(x2)=h。
h(x3)=0
各点で位置エネルギー+運動エネルギーが等しい。
2h。g=h。(1+√3/2)g+v1^2/2=h。g+v2^2/2=v3^2/2

v1=√(gh。)・(√6-√2)/2
v2=√(gh。)・√2
v3=√(gh。)・2

各点での加速度は各点での傾斜の正弦x重力加速度

a1=1/2・g
a2=√2/2・g
a3=0

628ご冗談でしょう?名無しさん:2009/04/10(金) 23:50:29 ID:TfG/+xYE
力のモーメント(ベクトル)の方向は、偶力の場合
回転方向を示しているわけではないですよね?

モーメント↑Nの方向が、何を意味しているのか、よく
理解できません。

コマの歳差運動では、モーメント↑N の方向は、角運動量↑Lの
時間変化の方向を向いているというのは、理解できたんですが・・。

角加速度ベクトル↑ωみたいな、
導入すると都合がいいものみたいな理解でいいんでしょうか・・
629623:2009/04/10(金) 23:53:09 ID:js/xU6OH
遅れました。

>>625>>627本当にありがとうございます。
630628:2009/04/10(金) 23:54:39 ID:TfG/+xYE
あ、間違い^^;
角速度ベクトル↑ωです
631ご冗談でしょう?名無しさん:2009/04/10(金) 23:58:23 ID:???
>>625 Δvを与えるってことでないの?だからxはtの関数にはならない。
632ご冗談でしょう?名無しさん:2009/04/11(土) 00:08:56 ID:???
>>631
高校物理の問題としては題意は >>627 だと思うけど、
> x=0m、時刻t=0sでボールの速度v=0m/s
を 時刻t=0sでボールの位置x=0m、速度v=0m/s が与えられたと読んで
初期値問題として運動方程式を解いたら解は >>625 でしょ。つまり
> x1=π/6、x2=π/2、x3=π
には到達しないという。
633ご冗談でしょう?名無しさん:2009/04/11(土) 02:32:50 ID:Iqch5bp3
時間と空間について説明お願いします。
634ご冗談でしょう?名無しさん:2009/04/11(土) 09:07:48 ID:???
f(ct,x,y,z)=0だね
635ご冗談でしょう?名無しさん:2009/04/11(土) 13:51:17 ID:???
時間は幻想
636ご冗談でしょう?名無しさん:2009/04/11(土) 16:55:37 ID:???
作用・反作用・・・同じ大きさで向きは反対、一直線上ではたらく
つり合い・・・・・・・同じ大きさで向きは反対、一直線上ではたらく

でいいんですか?
637ご冗談でしょう?名無しさん:2009/04/11(土) 17:03:52 ID:???
意味の異なる事を同じ文言で纏めても、正しい理解にはならないと思う。
638ご冗談でしょう?名無しさん:2009/04/11(土) 17:09:15 ID:???
作用・反作用・・・同じ大きさで向きは反対、一直線上ではたらき異なる物体間にはたらく
つり合い・・・・・・・同じ大きさで向きは反対、一直線上ではたらき同じむったいに働く

ではどうでしょう?
639ご冗談でしょう?名無しさん:2009/04/11(土) 17:21:29 ID:???
むったいw
640ご冗談でしょう?名無しさん:2009/04/11(土) 17:45:57 ID:???
>>638
別に間違ってはいないが、それで理解したことになるのかな?
641ご冗談でしょう?名無しさん:2009/04/11(土) 17:56:57 ID:???
>>638
質点だけを考える極度に単純化された状況ならそれでもOK。
連続体の釣り合いを考える場合はモーメントの概念が必要。
642ご冗談でしょう?名無しさん:2009/04/11(土) 18:00:24 ID:kvu30Gii
熱力学について質問です。エンタルピーは圧力一定で定義された状態量なのに、なぜギブスエネルギーの式を導出する際はdH=PdV+VdPと圧力の微分も考えるのでしょうか?
643ご冗談でしょう?名無しさん:2009/04/11(土) 18:07:22 ID:???
>>637-641
おおざっぱにそれとなくそういうことなら満足です。
ありがとうございました。
644ご冗談でしょう?名無しさん:2009/04/11(土) 18:20:47 ID:Iqch5bp3
アリはたしか…小学校5年生の時だ。校長先生が全校生徒の前で「質問がありましたら、私が何でも聞いて応えてあげます」と言ったので、手を挙げて質問した「時間と空間について説明をお願いします」と唱えた。。
したら…校長先生ったら、生徒の質問には一言も応えられずに退場されたのぉょ。

だれか応えてチョンマゲ
645ご冗談でしょう?名無しさん:2009/04/11(土) 18:38:07 ID:???
>>638
作用・反作用は常に2力(ペアでワンセット)だが、つりあいってのは
3力やら4力やら(以下同文)たくさんの力でも「消し合って0」なら
成立する概念だぞ。
646ご冗談でしょう?名無しさん:2009/04/11(土) 18:41:29 ID:???
>>642
「圧力一定で定義された」という言葉の意味を誤解している。

エンタルピーはH(S,P)のようにSとPの関数として定義されていて、
「SとPが一定ならば変化しない量」になっている。

という意味のことを「圧力一定で定義されているから圧力変化
しない」と誤解しているんじゃないか???
647ご冗談でしょう?名無しさん:2009/04/11(土) 18:42:20 ID:???
「つりあい」は単なる条件なのに対して、
「作用・反作用」は基本法則なんだから、そもそも同列に扱うのが間違いだと思う。
648ご冗談でしょう?名無しさん:2009/04/11(土) 18:43:36 ID:???
実際のところ、dH=TdS + VdPであって、むしろ圧力の微分の方が
大事なんだよな。
649ご冗談でしょう?名無しさん:2009/04/11(土) 18:57:25 ID:???
ニュートリノとニュートリノの反粒子でスピンがそれぞれ左回り、右回りの粒子しかないのは、
見つかってないからなのかそれとも(元々)存在しないからなのか、
主流の考えはどうなってるのでしょうか?
650ご冗談でしょう?名無しさん:2009/04/11(土) 20:56:57 ID:kvu30Gii
>>646>>648
ありがとうございます。
あと、エンタルピーは温度が一定なら必ず0になるという解釈で問題ないでしょうか?
651ご冗談でしょう?名無しさん:2009/04/11(土) 21:22:17 ID:???
>>650
今回のような、H=H(S,P)の場合なら
エンタルピー→エンタルピー変化
温度が一定→dS=0 かつ dP =0
と修正すればおk
652ご冗談でしょう?名無しさん:2009/04/11(土) 21:29:32 ID:???
>エンタルピーは温度が一定なら必ず0になるという解釈で問題ないでしょうか?

問題ありすぎ。大間違い。
653ご冗談でしょう?名無しさん:2009/04/11(土) 21:36:33 ID:???
>>649
 昔は「ないから」が主流だったが、今は(ニュートリノに質量がある
ことがわかったので)「ある」ということになっている。
654ご冗談でしょう?名無しさん:2009/04/11(土) 21:36:41 ID:???
>>649
> ニュートリノとニュートリノの反粒子でスピンがそれぞれ左回り、右回りの粒子しかない
とは考えられていない
655ご冗談でしょう?名無しさん:2009/04/11(土) 21:45:13 ID:kvu30Gii
>>651
分かりやすい解説ありがとうございます。
>>653
物理化学独学のため、このような質問をしてしまいすみませんでした。
656ご冗談でしょう?名無しさん:2009/04/12(日) 01:38:28 ID:???
あるAの地点に止まっているロケットが加速してBの地点に移動した後に、今後は逆方向に加速してBからAの位置に戻って停止したとします。

このロケットのエンジンはとっても高性能で質量を全て、速度に変えることができるとします。

そして加速したり、停止するために逆方向に加速するときに質量が減少すると思います。
従って、出発したときと帰ってきて停止したときでは、帰った時の質量は出発したときと比べて質量が減っていると思います

エネルギー保存則から言えば、この質量はどこにいったのでしょうか?
657ご冗談でしょう?名無しさん:2009/04/12(日) 01:43:50 ID:???
>>656
車でいうとガソリンが減ってる
658ご冗談でしょう?名無しさん:2009/04/12(日) 01:49:17 ID:???
>>656
 ロケットなんだからなんか噴射したはずだろうに。噴射してないならロケット
が加速することは(減速することも)あり得ない。
 運動量保存則があるからな。
659ご冗談でしょう?名無しさん:2009/04/12(日) 01:53:56 ID:???
>>656
つまり、運動量保存則があるから、「質量を全て、速度に変えることができる」
なんてことはあり得ないのだ。
660656:2009/04/12(日) 01:57:52 ID:???
>>657

なるほど。噴出する物体分減りますね。ありがとうございます。

>>658
そもそも前提が間違っていたようです。解説ありがとうございました。

>>659
運動量保存則から言って、質量を全て、速度に変えることはできないんですね。
とっても良くわかりました。ありがとうございました。
661ご冗談でしょう?名無しさん:2009/04/12(日) 02:01:05 ID:???
まず「質量を全て、速度に変える」の意味が判らない・・
662ご冗談でしょう?名無しさん:2009/04/12(日) 02:01:57 ID:???
>>661
ありえないことの意味なんてわからなくていいじゃん。
663ご冗談でしょう?名無しさん:2009/04/12(日) 02:10:46 ID:???
意味も判らず「有り得ない」なんて言えないな。
まあ何にせよ、質問者はもう少し物理的に意味の通る質問文で質問して欲しいな。
664ご冗談でしょう?名無しさん:2009/04/12(日) 02:12:37 ID:???
「質量がぱっと消えたかと思うとロケットが走り出す」みたいな魔法のシステム
があると仮定したってだけのことだろ。そりゃ「あり得ない」以外返しようが
ない。>>663
665ご冗談でしょう?名無しさん:2009/04/12(日) 02:15:55 ID:+8eIBqs0
質量×光速度の二乗がエネルギーと等価ってどういうことですかぁ〜?
666ご冗談でしょう?名無しさん:2009/04/12(日) 02:48:20 ID:???
E=MC^2でぐぐれ。
667ご冗談でしょう?名無しさん:2009/04/12(日) 06:38:17 ID:???
>>665
もともと固有質量に光速度cを掛けたヤツの二乗がローレンツ不変ということ。
それの運動量を0とした時でもエネルギーは0にならないという意味。
エネルギーはE/cという形で4元運動量の第0成分と扱われる。
668ご冗談でしょう?名無しさん:2009/04/12(日) 11:03:03 ID:???
導線に電流を流したら磁界が発生しますよね。
磁界中に電流を流したら電荷が力を受けますよね。

ということは、ある導線に電流を流したら、自身の発生させた磁界で導線はどこかの方向に力を受けるのでしょうか?
それとも磁界は導線の周りにできるだけで、導線に影響を与えるような磁界は発生していない(導線を通過するような
磁界はない)ので、力は受けないという感じですか?
669ご冗談でしょう?名無しさん:2009/04/12(日) 11:13:51 ID:???
>>668
電流も影響受けるよ。導線の表面近くを流れて、中心部は電流が小さい。
670ご冗談でしょう?名無しさん:2009/04/12(日) 12:09:09 ID:???
表皮効果ってそういうことなんか?
671ご冗談でしょう?名無しさん:2009/04/12(日) 12:21:26 ID:???
なんか右ネジの法則とフレミングの法則使うと、中心によりそうな気がする。
672ご冗談でしょう?名無しさん:2009/04/12(日) 12:41:58 ID:???
>>671  そのとおりだ。
表皮効果は、高周波成分が表面の方に追いやられる現象だな。
673ご冗談でしょう?名無しさん:2009/04/12(日) 23:34:19 ID:???
雨粒はかなり高いところから落ちてくるけど空気抵抗で
ある程度のスピードまでしかでないって聞いたんだけど
じゃあ雨粒が発生するぐらいの高度から地面に向けて
拳銃を発射した場合でもこれ以上は上がらないっていう
スピードはあると考えていいの?
674ご冗談でしょう?名無しさん:2009/04/12(日) 23:37:03 ID:riMZQpxy
超電導のミニ四駆のスレ良いですよ。
675ご冗談でしょう?名無しさん:2009/04/12(日) 23:47:35 ID:???
>>673
そういう(重力と空気抵抗が釣り合う)速度を終端速度と言います。
拳銃弾だと70m/sくらいで、高度3000mくらいから下に発射したのならほぼ終端速度に達してると思われます。
676ご冗談でしょう?名無しさん:2009/04/12(日) 23:49:18 ID:???
>>675  上向けて撃ったら、たぶんその速度で落ちてくると思われる。
677ご冗談でしょう?名無しさん:2009/04/12(日) 23:50:05 ID:???
>>668
まあ高校レベルで答えれば、直線状の導線ならば力は受けません。
678ご冗談でしょう?名無しさん:2009/04/12(日) 23:50:05 ID:???
>>673
そこに空気がある限り、空気中を運動する物体には空気抵抗は平等に働くお!


ただし、物体の運動速度や物体の大きさにもよっても空気抵抗の効き方が異なってくる
点には注意が必要だよ。
679ご冗談でしょう?名無しさん:2009/04/12(日) 23:53:52 ID:???
>>678
物体の形状や、物体表面の状態、回転でも大きく違うけどね。
680ご冗談でしょう?名無しさん:2009/04/13(月) 00:50:11 ID:???
ニュートリノと、ニュートリノって、くっつくの?
681ご冗談でしょう?名無しさん:2009/04/13(月) 01:06:02 ID:???
>>680
まず、素粒子と素粒子が「くっつく」という現象を定義してください。
682673:2009/04/13(月) 01:07:55 ID:???
>>675
>>678
>>679
なるほど 終端速度というのか 勉強になるなあ
つきあってくれた人ありがとう
683ご冗談でしょう?名無しさん:2009/04/13(月) 04:31:07 ID:???
英語だとTerminal Velocity 映画のタイトルにもなった。
684ご冗談でしょう?名無しさん:2009/04/13(月) 08:49:07 ID:???
>>681
定義はしらないけど、例としては クオークが「くっついて」陽子とか中性子とか。複合粒子ね。
685ご冗談でしょう?名無しさん:2009/04/13(月) 09:09:00 ID:???
レプトンにはクオークと違って、グルーオンに相当する粒子がないからくっつかないよ。
686ご冗談でしょう?名無しさん:2009/04/13(月) 09:28:15 ID:???
>>684
ポエムならポエム板でどうぞ。
687ご冗談でしょう?名無しさん:2009/04/13(月) 09:35:20 ID:???
「定義してくれ」に対して「定義はしらないけれど」に始まる独自の言説を展開。

まさにポエムだな。
688ご冗談でしょう?名無しさん:2009/04/13(月) 09:50:52 ID:???
>>685
>レプトンにはクオークと違って、グルーオンに相当する粒子がないからくっつかないよ。
ウイークボソンの交換で束縛系を造らないですか?
現在の実験装置ではエネルギーが足りないのかもしれんが?
689ご冗談でしょう?名無しさん:2009/04/13(月) 09:56:10 ID:???
>>688
      ィ";;;;;;;;;;;;;;;;;;;;;;;;;;;;;;;;;;;;;;;;;;;;;;;;;;;;;;;;;゙t,
     彡;;;;;;;;;;;;;;;;;;;;;;;;;;;;;;;;;;;;;;;;;;;;;;;;;;;;;;;;;;;;;ヽ
     イ;;;;;;;;;;;;;;;;;;;;;;;;;;;;;;;;;;;;;;;;;;;;;;r''ソ~ヾ:;;;;;;゙i,
     t;;;;;;;リ~`゙ヾ、;;;;;;;;;;;;;;;;;;;;ノ    i,;;;;;;!
     ゙i,;;;;t    ヾ-‐''"~´_,,.ィ"゙  ヾ;;f^!   / ̄ ̄ ̄ ̄ ̄ ̄ ̄ ̄ ̄ ̄ ̄ ̄
     ト.;;;;;》  =ニー-彡ニ''"~´,,...,,.  レ')l. < おまえは何を言っているんだ
     t゙ヾ;l   __,, .. ,,_   ,.テ:ro=r''"゙ !.f'l.   \____________
      ヽ.ヽ ー=rtσフ= ;  ('"^'=''′  リノ
    ,,.. -‐ゝ.>、 `゙゙゙゙´ ,'  ヽ   . : :! /
 ~´ : : : : : `ヽ:.    ,rf :. . :.: j 、 . : : ト、.、
 : : : : : : : : : : ヽ、  /. .゙ー:、_,.r'゙: :ヽ. : :/ ヽ\、
  :f: r: : : : : : : : !丶  r-、=一=''チ^  ,/   !:: : :`丶、_
  : /: : : : : : : : :! ヽ、  ゙ ''' ''¨´  /   ,i: : : l!: : : : :`ヽ、
 〃: :j: : : : : : : ゙i   `ヽ、..,,__,, :ィ"::   ,ノ:: : : : : : : : : : : :\
 ノ: : : : : : : : : : :丶   : : ::::::::: : : :   /: : : : : : : : : : : : : : : :\
690ご冗談でしょう?名無しさん:2009/04/13(月) 10:07:01 ID:???
>>688
無茶言うな。束縛系を作るんなら寿命が長くないといけないが、ウィークボソンの質量が
長時間存在したらエネルギー保存則にも不確定性原理にも反する。

そんなもんがあちこちあったらダークマターだ。
691ご冗談でしょう?名無しさん:2009/04/13(月) 10:12:33 ID:???
>>688
学部のとき同じ質問してる人がいたけど無いと言ってたよ
692ご冗談でしょう?名無しさん:2009/04/13(月) 10:20:20 ID:???
>>690
>束縛系を作るんなら寿命が長くないといけないが、ウィークボソンの質量が
共鳴状態のようなものならOKでは?
ウィークボソン自身がレプトンの複合粒子という話もどこかで聞いたような?
693ご冗談でしょう?名無しさん:2009/04/13(月) 13:34:58 ID:???
色々調べてみても分からなかったので質問させて下さい。

国際単位の組立単位で、単位の名称及びLMTの組み合わせは何か?
(長さ:L 質量:M 時間:T)

面 積       ?        ?
体 積       ?        ?
速 度    メートル毎秒   LT^-1
加速度  メートル毎秒毎秒  LT^-2
 力        ?       LMT^-2

調べたものも合っているのか回答お願いします。
694ご冗談でしょう?名無しさん:2009/04/13(月) 13:37:29 ID:???
ググレカス
695ご冗談でしょう?名無しさん:2009/04/13(月) 13:41:34 ID:???
>>693
  面 積    平方メートル   L^2
  体 積    立方メートル   L^3
> 速 度    メートル毎秒   LT^-1
> 加速度  メートル毎秒毎秒  LT^-2
   力      ニュートン    LMT^-2
696ご冗談でしょう?名無しさん:2009/04/13(月) 13:47:31 ID:???
>>695
どうもありがとうございます!!
助かりました!!
697ご冗談でしょう?名無しさん:2009/04/13(月) 15:48:14 ID:???
地球から宇宙に飛び出るためには秒速11キロ以上の速さがないとダメだと言われたんですが意味が分かりません。
時速100キロで垂直に飛び立てば一時間後には地球から100キロ離れていることになるのでは。
698ご冗談でしょう?名無しさん:2009/04/13(月) 15:52:14 ID:???
確かに。
699ご冗談でしょう?名無しさん:2009/04/13(月) 16:19:04 ID:???
>>697
宇宙速度 でぐぐれ
700ご冗談でしょう?名無しさん:2009/04/13(月) 16:20:37 ID:???
いやです。
701ご冗談でしょう?名無しさん:2009/04/13(月) 16:51:13 ID:???
調べるのがめんどくさいなら言ったやつに質問しろ
702ご冗談でしょう?名無しさん:2009/04/13(月) 16:55:35 ID:???
すなおに分かりませんって言えないん?
703693:2009/04/13(月) 17:01:49 ID:???
すみません、1つ見逃してました・・・
同じ問題で「圧力」はどうなるのですか?
704ご冗談でしょう?名無しさん:2009/04/13(月) 17:08:04 ID:???
力を面積で割るんだから考えればわかるだろ。
705ご冗談でしょう?名無しさん:2009/04/13(月) 17:38:33 ID:???
ttp://ja.wikipedia.org/wiki/宇宙速度

>つまり、あくまで理論的には打ち上げ直後から極端に小さい速度を維持しての脱出も可能であるが、
>2008年の時点では多分に非現実的なので実用化には至っていない。

初速11.2キロ毎秒ってのは水平での速度ってことでいいみたいですね。
だから理論的には歩くような速さでも時間さえかければ地球から脱出できると。
なんかここいらは結構誤解を招く表現があるような気がする。
でもブラックホールの「光さえも脱出できない重力」ってのに引っかかる。
秒速30万キロの速さで垂直に飛び出せば、光も脱出できるんじゃね?
706ご冗談でしょう?名無しさん:2009/04/13(月) 17:47:32 ID:???
>>703
国際単位の組立単位 でぐぐれ
707ご冗談でしょう?名無しさん:2009/04/13(月) 18:00:13 ID:???
>705 天才
708693:2009/04/13(月) 18:01:55 ID:???
>>706
それで調べたんですけどLMTの組み合わせでの名称が見つからないんです
709ご冗談でしょう?名無しさん:2009/04/13(月) 18:47:58 ID:???
>>708
小学生以下だな、死ねよ
710ご冗談でしょう?名無しさん:2009/04/13(月) 19:26:02 ID:???
宿題か何かでしょう
スルーで
711ご冗談でしょう?名無しさん:2009/04/13(月) 19:40:47 ID:???
まあまあ、スレタイが泣きまっせ。

圧 力  パスカル L^-1MT^-2
712693:2009/04/13(月) 20:12:34 ID:???
>>711
回答ありがとうございます
他の言葉も入れて色々と検索したらやっと出てきました
713ご冗談でしょう?名無しさん:2009/04/13(月) 20:27:36 ID:???
>>711
丸投げに答えるのは禁止だ
>>1読めボケ
714ご冗談でしょう?名無しさん:2009/04/13(月) 20:56:00 ID:???
>>713
自治厨のレスの方が正直うざい。
715ご冗談でしょう?名無しさん:2009/04/13(月) 21:08:36 ID:???
>>693
面積・体積もわからんってお前中学校で何習ったの?
それとも最近の義務教育ではそれも教えんの?
716ご冗談でしょう?名無しさん:2009/04/13(月) 21:46:06 ID:???
凡人の下らないささやかな疑問にどなたかお答え下さいm(_ _)m

例えば鉄の棒を水平に同じ高さで支えを付けて地球を一周させたら、それは直線の円って事になるの?そしてその支えを同時に外したらその円はどーなるのでしょう?

717ご冗談でしょう?名無しさん:2009/04/13(月) 21:54:18 ID:???
>>716
> 例えば鉄の棒を水平に同じ高さで支えを付けて地球を一周させたら、それは直線の円って事になるの?

思考実験としてなら、円でいいんじゃないの。だから直線ではない。

> そしてその支えを同時に外したらその円はどーなるのでしょう?

既出。スレ読み返してみ。
718ご冗談でしょう?名無しさん:2009/04/13(月) 22:28:19 ID:???
>>717
なるほど、ありがとうございます。凡人には難しいですね。

しかし、サラッとROMってみただけなのに筋肉バカ同士が争ってる画が浮かびました。
719ご冗談でしょう?名無しさん:2009/04/13(月) 22:38:11 ID:0GWSz+yo
生物板とこちらと、どちらがよいか分からないのですが・・・

放射性同位元素を使って心臓の細胞が再生しているかどうか、
調べるとかそういう研究があった
http://www.sciencemag.org/cgi/content/abstract/324/5923/98
というのを聞いて疑問に思ったのですが、

生物の体の中にある元素が入れ替わっているかどうかを、
放射性同位元素を利用せずに知ることはできるのでしょうか?

ある細胞のある遺伝子のある塩基の例えばある窒素原子が、
同じ質量数の「別の」窒素原子に置き換わったかどうかを、
知る方法は、理論的でもいいですが、あるんでしょうか?
それとも現代の物理学には存在しないんでしょうか?
720719:2009/04/13(月) 22:42:45 ID:0GWSz+yo
といいますか、別に生物の体の中の原子じゃなくてもいいですよね
ある時にここにあった窒素原子と、別の時にあっちにあった窒素原子が、
同一の窒素原子だったか、別の窒素原子だったかを、
知る方法があるんでしょうか?
それともこういう疑問そのものが物理の基本を弁えてないものでしょうか?
721ご冗談でしょう?名無しさん:2009/04/13(月) 23:06:07 ID:???
>>720
エンタングルメント(量子もつれ)を用いてそういう事を可能にするというアイデアは有る。
現時点ではとても実用的とは言いかねる状態だが。
722719:2009/04/13(月) 23:24:44 ID:???
>>721
どうもありがとうございます
やはり基本的には今のところ分からないのですね

たまに、真偽は私には分かりませんが、
「人の体も分子レベルで見ると○○年で全部入れ替わってる」
とか書いてあるのを見ますが、あれも同位元素で調べたんでしょうか・・・
このへんは生物板で聞いた方がよいですね・・・
723ご冗談でしょう?名無しさん:2009/04/13(月) 23:26:58 ID:???
>とか書いてあるのを見ますが、あれも同位元素で調べたんでしょうか・・・

これはそのとおりでしょう。
724ご冗談でしょう?名無しさん:2009/04/13(月) 23:57:51 ID:bqKNLVzP
非弾性衝突について、簡単に教えてください。


衝突において、運動エネルギー保存の法則に乗っ取り、熱や音にエネルギーが交換される場合と、他のエネルギーに交換されない場合では、後者はやはり、粒子のような極小の世界でしか有り得ないのでしょうか?
量子力学はより小さな世界では可能であっても、より大きな世界では不可能であるという線引きについて、明確な区別がされているのでしょうか?

よろしくお願いします。
725ご冗談でしょう?名無しさん:2009/04/14(火) 00:00:07 ID:???
日本語OK
726774ワット発電中さん:2009/04/14(火) 08:32:08 ID:tcPYsp/2
もはや古典の域に入っている話だが、パウリ(?)がそれをかなり本気で疑った
そのときには結局ニュートリノという粒子がエネルギーを持ち去っていただけだった
というのが今の理解だと思う
727ご冗談でしょう?名無しさん:2009/04/14(火) 08:49:06 ID:???
>>726 ゴメン。どのレスのことを指してるんだ?
728ご冗談でしょう?名無しさん:2009/04/14(火) 09:42:33 ID:EZw4NpN2
ニュートリノはクッションのような緩衝材のような位置付けだという事でしょうか?

エネルギーは消えてなくて、更に微小になって散乱した可能性は?

エネルギーは消えたのではなくて、小さすぎて観測で消えたように錯覚したとかは無いでしょうか?
そうだとしたら、エネルギー保存±0になって、丸く収まるんですが、どうにもこうにも不可解な事はあるようで。すいませんでした。トンデモ理論でした。


より堅実的に改変、訂正します。m(__)m
729ご冗談でしょう?名無しさん:2009/04/14(火) 10:32:04 ID:???
改変とかせんでいいから物理を1から勉強しなおせ。
 箸にも棒にもかかりゃしない。
730ご冗談でしょう?名無しさん:2009/04/14(火) 12:20:27 ID:???
ポエムをいくら改訂したところで物理にはなり得ない。
731ご冗談でしょう?名無しさん:2009/04/14(火) 12:29:28 ID:???
というか完全弾性衝突って実在するのかよ
732ご冗談でしょう?名無しさん:2009/04/14(火) 12:36:31 ID:???
気体分子の運動とか。
733ご冗談でしょう?名無しさん:2009/04/14(火) 13:02:26 ID:???
すべては近似
734ご冗談でしょう?名無しさん:2009/04/14(火) 15:23:36 ID:???
   宇宙の謎   
http://hideyoshi.2ch.net/test/read.cgi/ms/1239687139/
このコテって、ここでは有名なコテなの? 
735ご冗談でしょう?名無しさん:2009/04/14(火) 15:48:42 ID:???
宣伝乙
736ご冗談でしょう?名無しさん:2009/04/14(火) 16:01:21 ID:???
>>733
実在気体が理想気体と違う挙動をするのは、気体分子が有限の体積を持つことや
分子間力が存在するのが原因で、完全弾性衝突じゃないからではないんだよ。

というか、気体分子が完全弾性衝突でなければ、毎秒無数の衝突をする気体分子は
衝突ごとにエネルギーを失って、速やかに失速、その辺の低いところに溜まるんだが。
737ご冗談でしょう?名無しさん:2009/04/14(火) 16:15:04 ID:ZQghLNqc
(答)ラーメン
738ご冗談でしょう?名無しさん:2009/04/14(火) 16:33:20 ID:e0sBRuEq
気体分子中の電子のスピンがコロコロ反転することってあるのでしょうか。
739ご冗談でしょう?名無しさん:2009/04/14(火) 17:17:51 ID:???
>>736
気体分子は完全弾性衝突なんてしてないんだが・・・
740ご冗談でしょう?名無しさん:2009/04/14(火) 17:27:02 ID:???
>>739
多原子分子なら振動したり回転したりでエネルギー逃げるんだろうけど
アルゴンガスとか単原子分子の気体も非弾性なん?
741ご冗談でしょう?名無しさん:2009/04/14(火) 18:01:23 ID:???
>>739
では、なぜ速やかに失速、沈殿しないのか、説明していただきたい。

空気中の分子の平均速度は毎秒500m、平均自由行程は7e-8mのオーダーだ。
秒あたりの衝突数は膨大と言わざるを得まい。
742ご冗談でしょう?名無しさん:2009/04/14(火) 18:29:51 ID:???
>>741
完全弾性衝突でないってのは衝突前後で運動エネルギーの合計が等しい訳じゃないって事であって、
平衡状態に有る気体分子が運動エネルギーを失い続ける事とイコールじゃないんだぜ。
743ご冗談でしょう?名無しさん:2009/04/14(火) 18:40:51 ID:???
>>742
それはそのとおりだが、>>740の指摘はどうよ?
それから 分類上は回転のエネルギーも運動エネルギーに含まれるんじゃあないか?
まあ反発係数eは並進運動に対して用いられるので、指摘は正しいんだけどさ。
744ご冗談でしょう?名無しさん:2009/04/14(火) 18:57:49 ID:???
2つの系、A:(U,V、N) B(U',V',N') を熱的接触した場合、到達する平衡状態の条件からA,Bの温度が等しくなることを示せ。

この問題がわからないのですが、ヒントか何か教えてもらえないでしょうか
745ご冗談でしょう?名無しさん:2009/04/14(火) 19:06:43 ID:e0sBRuEq
>>740
単原子分子気体と言っても球対称なのは基底状態のヘリウムガスだけではなかろうか。
746ご冗談でしょう?名無しさん:2009/04/14(火) 19:15:43 ID:???
>>745
原子核のスピンは?
747ご冗談でしょう?名無しさん:2009/04/14(火) 19:21:07 ID:???
電磁気学の質問です。ノイマンの公式
M = μ/(4π)*甜C1]甜C2]ds1・ds2/r
で相互誘導係数を求めることができますが、
自己誘導係数を求めるのに公式を使おうとすると、r=0のとき被積分関数が発散して
値を求めることが出来ないように思えますが、一般にこの公式で自己誘導係数を求めることはできないのでしょうか。
748ご冗談でしょう?名無しさん:2009/04/14(火) 19:23:56 ID:e0sBRuEq
>>746
スピン軌道相互作用を無視するという近似での話。
考えたら、たぶん、破れる。
けど微細な効果だろう。
749ご冗談でしょう?名無しさん:2009/04/14(火) 19:26:33 ID:???
>>731
するだろ。
750ご冗談でしょう?名無しさん:2009/04/14(火) 19:46:38 ID:e0sBRuEq
>>748
つっかれると嫌だから書いとく。
スピンスピン相互作用も電子相関もみ〜んな無視。
751ご冗談でしょう?名無しさん:2009/04/14(火) 19:49:46 ID:???
>>749
というと?
752ご冗談でしょう?名無しさん:2009/04/14(火) 19:57:21 ID:???
>>747
Mはループが二つないと定義できないからね。
自己インダクタンスLはループ一つで定義される。
753ご冗談でしょう?名無しさん:2009/04/14(火) 20:27:34 ID:???
There are a lot of blood type O types in the world.
The United States is O type nation.
Japan is A type nation.

South Korea's O type is the fewest in the world.
754ご冗談でしょう?名無しさん:2009/04/14(火) 20:31:17 ID:???
Really? >>753
755ご冗談でしょう?名無しさん:2009/04/14(火) 20:32:54 ID:???
>>754
Yes.
Most of Caucasian is A type and O type.

The blood type is related to the race.
756ご冗談でしょう?名無しさん:2009/04/14(火) 22:56:38 ID:???
ポテンシャルV(x)の中を運動する質量mの粒子がエネルギーEの固有状態にあるとし、その波動関数をΦ(x)と表す。
流れの密度を
j=h/4πmi(Φ*dΦ/dx-ΦdΦ*/dx)と定義したとき
のx微分
dj/dx=0となる事を示したいのですが
どのように考えたらいいか分かりません。
波動関数は一般にexp(iθx)と置けることから解けばいいのでしょうか?

(Φ*はΦの複素共役です)
757ご冗談でしょう?名無しさん:2009/04/14(火) 23:03:56 ID:???
>>756
微分してシュレディンガー方程式
758ご冗談でしょう?名無しさん:2009/04/14(火) 23:52:19 ID:???
>>757
微分すると
dj/dx=h/4πmi(Φ*d^2Φ/dx^2-Φd^2Φ*/dx^2)となり
シュレディンガー方程式
-h^2/8mπ^2×d^2Φ/dx^2=-h/2πi×dΦ/dt
-h^2/8mπ^2×d^2Φ*/dx^2=-h/2πi×dΦ*/dt
を代入ですか?(見づらくてすいません・・・)

代入しても
h/4πmi(Φ*4mπ/ih×dΦ/dt-Φ4mπ/ih×dΦ*/dt)
になって消えないのですが・・・;
759ご冗談でしょう?名無しさん:2009/04/15(水) 00:27:31 ID:???
>>758
i の復素共役は?
760ご冗談でしょう?名無しさん:2009/04/15(水) 00:40:11 ID:???
>>759
>-h^2/8mπ^2×d^2Φ*/dx^2=-h/2πi×dΦ*/dt
この部分が
-h^2/8mπ^2×d^2Φ*/dx^2=h/2πi×dΦ*/dt
こうですね!

計算しなおすと
dj/dx=4mπ/ih×d/dt(Φ*Φ)になったのですが
この先はどうしたらいいのでしょうか?
761ご冗談でしょう?名無しさん:2009/04/15(水) 01:07:04 ID:???
>>760
> エネルギーEの固有状態にあるとし、その波動関数をΦ(x)と表す。
762ご冗談でしょう?名無しさん:2009/04/15(水) 01:30:15 ID:66ybitfs
電磁気力と重力は法則(距離の二乗に反比例)がよく似ているので
マクスウェル方程式の変数を互いに置換できそうだけど無理なのかな?
置換すべき変数をうまく類推できれば宇宙の本質を解き明かせる気がする。

以下、例。

マクスウェル方程式の解釈
・磁場には源がない。
・磁場の時間変化があるところには電場が生じる。
・電場の源は電荷である。
・電場の時間変化と電流とで磁場が生じている。
∴光は磁場と電場の波

置換

・時間には源がない。
・時間の変化があるところには物質が生じる。
・物質の源は質量である。
・物質の運動で時間が生じている。
∴重力は時間と物質の波
763ご冗談でしょう?名無しさん:2009/04/15(水) 01:33:28 ID:???
で反重力飛行円盤っすか?
764ご冗談でしょう?名無しさん:2009/04/15(水) 02:10:10 ID:???
衛星を打ち上げる時に地球の自転を利用するために
ロケットはある程度の高さに到達すると東に向きを変えますよね。

なぜ発射台を初めから斜めにしないのでしょうか?
765ご冗談でしょう?名無しさん:2009/04/15(水) 02:15:02 ID:???
>761
あー時間に依存しない方の固有値方程式を使うんですね。。
多分できたと思います!
ありがとうございました!
766ご冗談でしょう?名無しさん:2009/04/15(水) 02:56:28 ID:???
>>764
旧ISAS系のM-Vなんかの様に、斜めにする方式もあります。
(これはまあ重力ターン方式の名残なんだけど)
大きなロケットですと剛性を保って保持・安全に離床させるのが難しくなるので、
垂直打ち上げが普通になります。
767ご冗談でしょう?名無しさん:2009/04/15(水) 03:29:27 ID:???
>>744を教えてもらえないでしょうか?
あれからずっと気になっていて考えてるのですがよくわからないんです。

768ご冗談でしょう?名無しさん:2009/04/15(水) 07:18:25 ID:???
丸投げだからスルーなんだよw
769ご冗談でしょう?名無しさん:2009/04/15(水) 07:29:13 ID:???
運動量保存の原因は空間の並進対称性
角運動量は回転対称性
じゃあエネルギー保存は?
770ご冗談でしょう?名無しさん:2009/04/15(水) 08:34:18 ID:???
>>768
熱的接触というのは熱だけをやりとりするということでいいのでしょうか?
771ご冗談でしょう?名無しさん:2009/04/15(水) 08:47:01 ID:???
>>769
上の2つを知っていて、それを知らないのか?
772ご冗談でしょう?名無しさん:2009/04/15(水) 09:49:04 ID:gJxGHjjF
電磁気の第三のベクトルトルク力と
重力って関係ないんですか
773ご冗談でしょう?名無しさん:2009/04/15(水) 10:25:58 ID:???
>>766
一応あったことはあったんですね。
ありがとうございました。
774ご冗談でしょう?名無しさん:2009/04/15(水) 12:11:32 ID:CjOLMgTQ
いろんな素粒子があるようだけど

それらはいったい何なの。
そもそも何なの。

そりゃ定義はされてるよ
されてるが、どうしてそれが成り立つのか、など実態が分からない。
775ご冗談でしょう?名無しさん:2009/04/15(水) 12:19:31 ID:CjOLMgTQ
「電場と磁場が交互に現れる」

そりゃいいけど、なんでそうなんの。とか。

わからんことが多過ぎる。

キリンの首が長いのは高いところにある食い物を食うため、
草食動物の目が離れまくってるのは広い視野を持つことで肉食動物から逃れるため
のように、そうなるのは何か理由があるんだろ
その理由って何

776ご冗談でしょう?名無しさん:2009/04/15(水) 12:20:23 ID:???
それは物理の範疇を超える問い。

ていうかその手の書き込みは既視感ありまくり。
777ご冗談でしょう?名無しさん:2009/04/15(水) 14:49:59 ID:???
>>762
相対性理論によれば, 時間だけ単独で切り離すことは出来ず、
時空連続体とみなさなければならない。
電場と磁場も電磁場というひとつの場とみなさなければなならない。

時空と物質(質量・エネルギー複合体)を一体化させる考えは昔からあるが、
それができたとしても電磁場とは違いがいろいろあるし
他の場も考慮する必要があるから、さらに画期的アイデアがないと無理だろう。

とりあえず、>>762はまず特殊相対性理論を学ぶのが先決だな。
778ご冗談でしょう?名無しさん:2009/04/15(水) 15:19:03 ID:???
>>775
電場と磁場が同時に現れるモデルよりも交互に現れるモデルのが
最適であったため同時モデルが淘汰され交互モデルが残った
それだけのことでしょ?宇宙誕生の瞬間にもいろんなモデルが
カンブリア期の奇妙な生物同様に爆発的に出てきて
適応したモデルが残った。それがたまたま今の物理法則ってこと。
宇宙誕生以来淘汰されてきたモデルの方が圧倒的に多いんだから
現在残ったモデルがうまく出来ているなぁ〜と思うのは当たり前。

キリンの首も同じでもともと短い首のキリンの集団の中から突然変異で
長い首のキリンが時折出現し短い首のキリンと同時期を過ごしていた。
やがて環境的に食料となる木の葉が少なくなってくると、
集団の圧倒的多数に属する首の短いキリンは不利となってきて
首の長いキリンがより高い木の葉を偶然にも食べることができたことから
その集団内の首の短いキリンと首の長いキリンの数に逆転が起きてくる。
こうしてより環境に適応した首の長いキリンのが子孫を多く残すことができ
本来の首の短いキリンは子孫を残すこことが厳しくなりやがて淘汰された。
突然変異と環境淘汰が合致して今の形の生物が偶然に残っただけのこと。
779ご冗談でしょう?名無しさん:2009/04/15(水) 15:56:38 ID:???
>>775 >>778
電場と磁場が交互に現れるて、どういうこと?
780ご冗談でしょう?名無しさん:2009/04/15(水) 16:08:40 ID:???
そこからかよwww
781ご冗談でしょう?名無しさん:2009/04/15(水) 16:33:22 ID:???
779はよくわかっててあえてツッコんでる希ガス。
電磁波なら電場と磁場は同時に(位相を合わせて)増減するが、
交互に現れる、とは一体どういう状況なのか、と
782ご冗談でしょう?名無しさん:2009/04/15(水) 17:02:24 ID:???
>>765
エネルギー固有状態の波動関数は Φ(x,t) = exp(-i E t/hbar) φ(x) のように変数分離できる(hbar=h/(2π))。
783ご冗談でしょう?名無しさん:2009/04/15(水) 17:15:58 ID:???
>>781
話について行けずにようやっと見つけたあら捜しがそこだったってだけやろ。
784ご冗談でしょう?名無しさん:2009/04/15(水) 17:24:14 ID:???
俺のが知っている的な一種の自己顕示欲みたいなものか。
どの道、主題となる問題の本質からは外れた茶々入れ見たいなものでしょ?
まぁよくあることだね。
785ご冗談でしょう?名無しさん:2009/04/15(水) 18:44:23 ID:???
一億度の物質の黒体放射や熱制動放射は可視光ではどのように見えるのでしょうか?
786ご冗談でしょう?名無しさん:2009/04/15(水) 18:56:45 ID:???
見えないから黒いんでしょうが。
787ご冗談でしょう?名無しさん:2009/04/15(水) 19:43:44 ID:???
>>786
ちょ、おま
788ご冗談でしょう?名無しさん:2009/04/15(水) 19:49:52 ID:???
>>786 黒体輻射を知らないバカ発見。

>>785 シュテファン・ボルツマンの法則、ヴィーンの変位則でググれ。

後者によると波長のピークはX線域。可視光域は紫側が強いけど、程度モノで
全域にわたって強い。ほとんど白色。十分遠方からみて青白いという程度。


789ご冗談でしょう?名無しさん:2009/04/15(水) 20:45:43 ID:y1BN5azm
こんばんは
これらの問題がわからないので、解き方、使う公式、もしできれば回答例などを教えて下さい。

問,電子を10kvで加速した時
@電子エネルギーは何J?何eV
A電子速度は何m/s?
B電子の運動量は何kg・m/s?
C電子の物質波は何m?
ヒント:m=9.1×10^-31kg=0.51MeV
h=6.6×10^-34J・s
e=1.6×10^-19
1eV=1.6×10^-19J

です。
よろしくお願いします。
790ご冗談でしょう?名無しさん:2009/04/15(水) 20:46:21 ID:???
剛体の力学で質問

角運動量保存則が成り立つのってどんな場合なの?
一様球が坂を滑らずに転がり落ちるときってωが動くはずだけど、慣性モーメントも球が変形しなければ普遍だよな?
このときの角運動量って当然ながら変化してるよな?
重力という外力が加わってるから保存されないのか?
保存されるのは閉鎖系においてのみ??
791ご冗談でしょう?名無しさん:2009/04/15(水) 20:48:48 ID:???
当たり前じゃん。運動量だって自然落下のとき増えるだろ。
792ご冗談でしょう?名無しさん:2009/04/15(水) 20:51:03 ID:???
 運動量は必ずしも保存しないの?
793ご冗談でしょう?名無しさん:2009/04/15(水) 20:51:04 ID:/eH0dyXW
ですよね。書いてるうちになんてアホな質問したんだと恥ずかしくなりますた。
794ご冗談でしょう?名無しさん:2009/04/15(水) 20:52:50 ID:???
>>792 全体で見たら保存してるよ。
 物質を熱したときエネルギーは増えるが
 それがエネルギー保存則に反していないのと同じ。
795ご冗談でしょう?名無しさん:2009/04/15(水) 22:09:54 ID:???
764 に便乗。
自転の利得を無視して西向きにロケットを打ち上げるという行為には
「無駄なのでやらない」という点以外に問題があったりするでしょうか?
796ご冗談でしょう?名無しさん:2009/04/15(水) 22:15:22 ID:???
大中国から報復のロケットが飛んできても知らないよ。
797ご冗談でしょう?名無しさん:2009/04/15(水) 22:17:41 ID:???
他の衛星はほとんど順行なので、逆行衛星は順行衛星と衝突の頻度が高くなる。

また、相対速度も高いので、小さなデブリでもダメージが大きい。
追突か、正面衝突か。
798ご冗談でしょう?名無しさん:2009/04/15(水) 22:22:57 ID:???
>>789
まず 電子ボルト でぐぐれ。それから
>>1
> どこまでやってみてどこが分からないのかを書くこと。
799ご冗談でしょう?名無しさん:2009/04/15(水) 23:22:47 ID:???
>>795
対地速度が増えるので用途によってはメリット・デメリット有るだろうな。
(昔 静止バックアップ用に逆行気象衛星を提案してた人がいた、高頻度全球走査が魅力)
実際問題イスラエルは隣国との関係から、地中海側(西向き)に衛星打ち上げを行っている。
800ご冗談でしょう?名無しさん:2009/04/15(水) 23:30:40 ID:???
ド・ブロイ波長の次元解析で質問です。

10eVで加速させている電子のドブロイ波長の次元解析がどうしてもm(メートル)にならないのです。

λ=h/(2・m・10eV)^1/2
だから次元解析では

[J・s]/[kg・C・J/C]=[s/kg]となってしまいます
[m]になるはずなのですがどこが間違っていますか?
801ご冗談でしょう?名無しさん:2009/04/15(水) 23:36:34 ID:???
平方根を忘れてる?
802ご冗談でしょう?名無しさん:2009/04/15(水) 23:43:06 ID:???
>>801
なるほど!
導出できました。ありがとうございました。
803ご冗談でしょう?名無しさん:2009/04/16(木) 00:00:23 ID:???
馬鹿な質問をさせていただいてもいいでしょうか。

ヘリウムを入れたゴム風船とかがどれくらい浮くのか?ってのの計算方法って
どうやるのでしょうか。
風船はどうして浮くのか?ってアルキメデスの原理で説明したのは多数あるのですが、
計算方法などは見つからなくて。
高空に行くと風船も圧力が減って膨張するから割れやすくなるでしょうし。

小型ヘリウムガスボンベと大型のゴム風船、調整弁があれば、いくつかの風船を
割れないように調整すれば宇宙まで行けるんじゃね?
って疑問が仕事中にずっと脳内を駆け巡っておりまして・・・
804795:2009/04/16(木) 00:09:10 ID:???
ありがとうございました。
805ご冗談でしょう?名無しさん:2009/04/16(木) 00:29:45 ID:8/VXROyd
自由落下のように物理の教科書に載っているのではない、加速度的な運動(もしくは成長)の具体的な例って何がありますか?
例えば宇宙の膨張のような
806ご冗談でしょう?名無しさん:2009/04/16(木) 00:38:48 ID:vfWpIM0w
一つ質問させて頂きたいです。
今夜中に「いくら吸っても水が一定の高さより上がらないストロー」を作らなければならないのですが、自分は物理を
やったこともない人間なので、どうしたものかと困り果てております。
この四日間考え、いくつか試作を作り、一番うまくいったものが、
「ストローに、小さな穴をあける」でした。
しかしこれでは、肺活量が大きい人が吸うと普通に吸えてしまいます。
他にも、水が同じ高さを保とうとする特性(?)が使えないものか・・
と素人なりに考えています。
1、素材はストロー及びチューブのみ。
2、吸う人間の肺活量に左右されない
の条件で、そんなものは、果たして作れるのでしょうか?
どうぞ知恵をお貸しください・・・
807ご冗談でしょう?名無しさん:2009/04/16(木) 00:44:16 ID:???
>>806
普通のストローでも「いくら吸っても水が一定の高さより上がらないストロー」になるよ。
長さは10m必要だけど。
808ご冗談でしょう?名無しさん:2009/04/16(木) 00:49:01 ID:???
たとえば
一様密度のまっすぐな棒が (x、y)=(-3、0) と (x、y)=(3、0) を端としてx軸に平行で無重力の宇宙空間に静止してるとします。

このとき(x、y)=(-3、0)の部分にy軸上方向に一瞬だけ力を加えた場合、回転しながら動き出すと思うのですが
・重心の運動方向はそのままy軸上方向なのか? それとも上方向とはいえ左端に力を加えたから右方向(x+方向)にも多少成分が発生するのか?
・力を加える場所が重心だった場合、回転しないので全ての力が進む速さだけになるが、
 今回は回転しちゃってるので、その分進む速さの絶対値は重心に加えた場合より遅くなる。のか、
 それとも回転関係なし(力加えた場所関係なし)に、加えた力だけで重心の移動速度は決まる、のか

どうなんでしょう?
御人的なイメージとしては 右方向にも発生。速さも多少遅くなる。なんですが・・・。
809ご冗談でしょう?名無しさん:2009/04/16(木) 00:50:21 ID:vfWpIM0w
すみません。
ひとつ書き忘れていました。
「10mのストロー」も一度は考えたのですが、
その考え方は「禁止」だと言われてしまったので
使えないのです・・・
810ご冗談でしょう?名無しさん:2009/04/16(木) 01:19:56 ID:???
直径を太くしてみたら?
空き缶ぐらいの。
それかめっちゃ末広のロート型のストローとか
とにかく大量の水の質量で一定以上吸わせないようにしてやれば?
811ご冗談でしょう?名無しさん:2009/04/16(木) 01:23:47 ID:7Kwa3zzy
>>795
無駄なのでやらないに尽きると思うよ。
どういう利点が考えられるか挙げてみたら。
812ご冗談でしょう?名無しさん:2009/04/16(木) 01:31:04 ID:vfWpIM0w
ありがとうございます。
質量が大きいと吸えなくなるんですね・・・早速作ってみます。

ところで、「ストローを二本使ってやる」方法があるそうなのですが、そちらはわかりますでしょうか。
ちなみに私は、「水の入り口ひとつに対し、高さの違う二つの出口を作り、高い所から吸い込む」
ことをしてみましたが、無理でした。何だか気になって仕方がありません・・・
813ご冗談でしょう?名無しさん:2009/04/16(木) 01:34:29 ID:G+4MaIsM
ストローをどの程度までなら改造していいのか分からないから何とも言えないな。長くするのが駄目ならストロー一本では無理なんじゃない?
814ご冗談でしょう?名無しさん:2009/04/16(木) 01:37:30 ID:vfWpIM0w
数本つなげたり、ベンを作ったりならおkだそうで・・・
815ご冗談でしょう?名無しさん:2009/04/16(木) 01:42:31 ID:G+4MaIsM
ストロー2本咥えて吸うってのはどうかな?ただし1本の先は水の中、もう1本の先は空気中。これは吸わないのと同じことか?
816ご冗談でしょう?名無しさん:2009/04/16(木) 01:54:16 ID:vfWpIM0w
いっぱい咥えてやったらできましたが、なんだか違和感が・・・
817ご冗談でしょう?名無しさん:2009/04/16(木) 01:54:37 ID:???
>>808
・力積(ベクトル)=運動量変化(ベクトル)なので、併進運動の方向はy軸方向。
・力をどのように加えるかをもう少しきちんと決めないと答えるのが面倒になりますが、
例えば加える力積が同じならば(例えば一定の力で一定の時間押した)、端を押しても重心でも併進運動の速さは変わりがありません。
しかし端を押した方が簡単に動きますよね、で押した部分の動いた距離*力(つまりなされた仕事)は端を押した方が大きくなります、
ちょっと考えればわかる様に、この差は棒の回転のエネルギーになってます。
818ご冗談でしょう?名無しさん:2009/04/16(木) 01:58:47 ID:???
>>810
その方法は2.の条件にそぐわないと思うが
819ご冗談でしょう?名無しさん:2009/04/16(木) 02:02:41 ID:G+4MaIsM
つまり2の条件は掃除機で吸うでOK?もちろん隙間のないよううまく接続する
820ご冗談でしょう?名無しさん:2009/04/16(木) 02:04:41 ID:J+JZHxVy
ねえ
ずいぶん前に書いたんだけどもう一回質問
フレミングの3本のベクトルのうち力に相当するものって
何?
821806:2009/04/16(木) 02:07:05 ID:vfWpIM0w
あ・・・言われてみれば条件に・・・

2は掃除機で吸う・・・でもよいと思います。
822ご冗談でしょう?名無しさん:2009/04/16(木) 02:13:45 ID:???
>>788
横レスだが、ちょっと考えると面白いな。
どんどん温度が上がっていくガスを可視光だけで観察していると、
白からだんだん青くなった後、今度は再び白に近づいてくのか。
823ご冗談でしょう?名無しさん:2009/04/16(木) 02:13:57 ID:J+JZHxVy
一定の位置で空気が完全に漏れるように
吸い口だけ残してほかを鳥かご状にしたら
824ご冗談でしょう?名無しさん:2009/04/16(木) 02:14:55 ID:J+JZHxVy
やっぱ答えてもらえないか
何でかな
825ご冗談でしょう?名無しさん:2009/04/16(木) 02:22:11 ID:???
マジで弁付けろよ、フロートを使った弁なら簡単。
826806:2009/04/16(木) 02:29:55 ID:vfWpIM0w
鳥かご状というのは、ストローに穴を空ける・・・ようなことでしょうか?

フロートを使った弁というのはどういうものでしょうか?
827ご冗談でしょう?名無しさん:2009/04/16(木) 02:30:51 ID:J+JZHxVy
うんスカスカで格子だけの状態のこと
828ご冗談でしょう?名無しさん:2009/04/16(木) 07:23:30 ID:???
>>813
まったくだ。
センサー装備で電子制御弁を付ければ実現できるのは明らかだが、ストローと呼べるのか。
829ご冗談でしょう?名無しさん:2009/04/16(木) 08:14:15 ID:???
>>822
ていうか、シュテファン・ボルツマンの法則を適用すると、面積あたりの放射エネルギーは
絶対温度の4乗に比例するから、太陽表面の約7.7京倍。

1kmどころか1天文単位も近くに寄りたくない。
830ご冗談でしょう?名無しさん:2009/04/16(木) 10:45:25 ID:???
もし、現代の科学力を総動員してカタナを造るとしたら
どんな素材を刃に使いますか?
831ご冗談でしょう?名無しさん:2009/04/16(木) 11:01:06 ID:???
アーク溶断白熱剣
832ご冗談でしょう?名無しさん:2009/04/16(木) 11:24:02 ID:???
>>822
 いや、温度上げていくと青になって、後はずっと青のままだと思うぞ。
 高温になるってことはレイリー・ジーンズに近づくということで、ということ
は短波長が高いグラフになる。
833ご冗談でしょう?名無しさん:2009/04/16(木) 11:45:13 ID:???
色合いはスペクトルの分布で決まる
赤より青の方が強ければ青く見える

十分高温では可視光領域はどこもほとんど一緒なんだから真っ白になるはず
834ご冗談でしょう?名無しさん:2009/04/16(木) 12:20:09 ID:???
>>830
カーボンナノチューブを使った単分子ワイヤーが最適
スラスラ切れて頑丈だしね
835ご冗談でしょう?名無しさん:2009/04/16(木) 12:29:59 ID:???
昼光スペクトルは緑あたりにピークがあって青は若干少なめ。
これを白色と感じるとすると、青から赤までどこもほとんど一緒の
スペクトルの光は若干青みがかって青白く感じる希ガス
836ご冗談でしょう?名無しさん:2009/04/16(木) 14:26:21 ID:J+JZHxVy
超音波振動付き日本刀
837ご冗談でしょう?名無しさん:2009/04/16(木) 16:35:50 ID:F/fGtK/7
どなたか>>805お願いします
838ご冗談でしょう?名無しさん:2009/04/16(木) 16:51:04 ID:???
>>834
いくらカーボンナノチューブが強くても、単分子だとすぐ切れちゃうよ。
839ご冗談でしょう?名無しさん:2009/04/16(木) 17:11:19 ID:NZqxpsKJ
>>830 タングステン
840ご冗談でしょう?名無しさん:2009/04/16(木) 17:14:43 ID:???
>>837
>>1
> どこまでやってみてどこが分からないのかを書くこと。
841ご冗談でしょう?名無しさん:2009/04/16(木) 17:25:08 ID:???
>>840
「例えば宇宙の膨張のような」って書いてあるだろ。

他に何があるか?って尋ねてるのに、なにを求めてるの?

正直、適切な例とは思えないし、他にも思いつかないんだが。
842ご冗談でしょう?名無しさん:2009/04/16(木) 17:33:28 ID:???
>>837
爆発
843ご冗談でしょう?名無しさん:2009/04/16(木) 18:02:40 ID:???
>>837
全力疾走する体操服少女のおぱーい

844ご冗談でしょう?名無しさん:2009/04/16(木) 18:18:08 ID:???
>>838-839 んじゃ、ニーヴンの小説の 「シンクレアの単分子チェーン」 という良く切れる刃物は
ウソっぽいのね。
845ご冗談でしょう?名無しさん:2009/04/16(木) 18:30:36 ID:???
>>844
好きなシリーズだが、現実とSFを混同しちゃアカンだろ。

まあ、好意的に取ると、単分子を何千本か束ねたものってことで。
846ご冗談でしょう?名無しさん:2009/04/16(木) 19:19:38 ID:???
ニーヴンは結構トンデモ多いから注意
847ご冗談でしょう?名無しさん:2009/04/16(木) 20:03:50 ID:???
超硬にダイヤモンドコーティング
848ご冗談でしょう?名無しさん:2009/04/16(木) 20:11:27 ID:???
>>833
>十分高温では可視光領域はどこもほとんど一緒なんだから真っ白になるはず

 その、「十分高温では可視光領域はどこもほとんど一緒」という思いこみは
どっから来たんだ??
 黒体輻射のグラフ書いてみな。高温なら短波長の方が上がるグラフに絶対
なるから。

849ご冗談でしょう?名無しさん:2009/04/16(木) 20:29:46 ID:ERYSGQGx
>>789なんですが、
@は、
10KV=10KeV
1.6×(10^-19)×10000=1.6×10^-15(J)
でいいんですか?

AE=1/2m(v^2)より、
1,6×(10^-15)=1/2×9.1×(10^-31)×v^2
v^2=3.5×10^15
までできたんですけど、ここから先は数字が無茶苦茶になるんですが、計算ミス
なんでしょうか?

BとCはvを求めないと出ないみたいなのでまだできませんでした。
誰か教えてくれませんか?
お願いします。
850ご冗談でしょう?名無しさん:2009/04/16(木) 20:37:34 ID:???
>>849
無茶苦茶になるってどうなるの?それと、途中の式で単位を省略するのは間違いのもと。
851ご冗談でしょう?名無しさん:2009/04/16(木) 20:44:23 ID:???
ルートを取ったら桁数が大きくなったから間違ったと思っているのかな?

世の中、割り切れない答えなんていくらでもあるんだが。

そのために概数とか四捨五入とかがあるんだが。
852ご冗談でしょう?名無しさん:2009/04/16(木) 20:46:38 ID:???
 これまで簡単なれんしゅうもんだいしか解いてないと、ちょっと桁数が
増えただけで、おかしいとか大変とか言いやがる。

この軟弱者が。
853ご冗談でしょう?名無しさん:2009/04/16(木) 20:48:40 ID:???
>>851
1.6÷9.1では困ってないらしいんだが。
854ご冗談でしょう?名無しさん:2009/04/16(木) 20:52:38 ID:???
不思議な奴棚
855ご冗談でしょう?名無しさん:2009/04/16(木) 21:42:40 ID:???
いくら吸っても水が一定の高さより上がらないストロー

ストローではないが、水の中に沈めてしまえばどうだろう。
いくら吸っても、水の高さは減りこそすれ上がらないw
856ご冗談でしょう?名無しさん:2009/04/16(木) 22:41:04 ID:???
>>848
「十分高温なら」
意図的に無視してるのかも知れんがこう書いてあることを確認しとく
857ご冗談でしょう?名無しさん:2009/04/16(木) 23:22:36 ID:???
宇宙の最初って何があったんですか?
無から何かが生まれたりするんでしょうか?
858ご冗談でしょう?名無しさん:2009/04/16(木) 23:30:06 ID:???
>>856
どんなに高温でも一緒だってば。

黒体輻射の式を振動数で微分して、十分高温なら常に正、
つまり右上がりグラフになること角煮すればいい。
859ご冗談でしょう?名無しさん:2009/04/16(木) 23:32:41 ID:???
>>857
宗教による。

俺は最初も無ければ終わりもないと思ってる。
860ご冗談でしょう?名無しさん:2009/04/16(木) 23:41:06 ID:???
君の信じる神様は偽物だ。
861ご冗談でしょう?名無しさん:2009/04/16(木) 23:41:58 ID:???
>>822だが
白に「近づいていく」と書いたんだよ。
つまり色味の変化が逆転して見えるのが面白いと。
862ご冗談でしょう?名無しさん:2009/04/16(木) 23:41:59 ID:???
>>856
高温になる→量子効果が無視できる→等分配則が成り立つ

と考えているのかな?

 高振動数モードの方が数多いよ。

 1次元で考えているなら別だが。
863ご冗談でしょう?名無しさん:2009/04/16(木) 23:44:05 ID:???
>>861
逆転しない。

色味としては水色っぽい青で飽和する。
864ご冗談でしょう?名無しさん:2009/04/16(木) 23:45:35 ID:???
目がつぶれるほどまぶしいのは何色か?って問題でしょ
865ご冗談でしょう?名無しさん:2009/04/16(木) 23:47:06 ID:???
そんなこと言い出したら、

見えねえ。もう何も見えねえ。

で終わりだ。
866ご冗談でしょう?名無しさん:2009/04/16(木) 23:55:55 ID:???
>>859
無神論者ですが、キリスト教には興味があります
最初に何かがあったから、その次が存在すると考えるのは間違えなんでしょうか?
元々宇宙があったとしても、その最初には何かがあったはずだと思うのです
867ご冗談でしょう?名無しさん:2009/04/17(金) 00:00:04 ID:???
キリストよりモハマドの方が偉い予言者だぞ。
868ご冗談でしょう?名無しさん:2009/04/17(金) 00:20:30 ID:???
>>866
間違い。
時間も空間も物質も光も何も無い。しかし揺らぎはある。
しいていうなら風呂に水を張ってかすかな揺らぎがあるのを想像してみて。
そしてごくたまにその揺らぎと揺らぎがぶつかり一滴の水がぽちょんと跳ね上がる。
それが時間も空間も物質も光も何も無いがゆえに静止することなく際限なく広がる。
それが今の宇宙。
869ご冗談でしょう?名無しさん:2009/04/17(金) 00:21:53 ID:n8wDNhos
まず宇宙ありきって発想は存在しないんでしょうかね
870ご冗談でしょう?名無しさん:2009/04/17(金) 00:22:14 ID:???
それはどこの宗教?
871ご冗談でしょう?名無しさん:2009/04/17(金) 00:23:07 ID:n8wDNhos
空間も物質もはじめから存在しており始まりなど存在しないと言う考えとか
872ご冗談でしょう?名無しさん:2009/04/17(金) 00:29:00 ID:5Bu/4t0k
>>869
現実を否定しない限り、どんな発想もあり得ると思うよ。
誰も論理的には否定できない。
「真実は神のみぞ知る」世界なんだから。
873ご冗談でしょう?名無しさん:2009/04/17(金) 00:37:41 ID:IpGT7eZ2
数学板でもしつもんしてるのですが、
物理屋さんのなわばりかなとも思いましたので、お願いします。


同一半径の球を無限大の箱に詰め込んだとき、
隣り合う球との距離がすべての球について互いに等しい配置で、
且つ、全ての位地で一様な連続構造。

これが存在するとしたら、どんな配置になります?
あほだからわからん。 お願いします。

どこかのサイトのURLでもいいです。
874ご冗談でしょう?名無しさん:2009/04/17(金) 01:00:29 ID:???
>>864
まぶしさ(可視光域の総フラックス)はいくらでも調整できるよ

ちょっと計算したら、可視光の波長の領域を[λmin,λmax]として
λminでの強度I(λmin)とλmaxでの強度I(λmax)の比(色合い)は
T->∞の極限ではある一定値に収束するんだな。
1ではないので一様分布ではないけど。
875ご冗談でしょう?名無しさん:2009/04/17(金) 01:08:15 ID:???
>全ての位地で一様な連続構造。
よくわからんが、球の半径が無視できる程度のスケールで見たら一様ということなら
面心立方構造もしくは六方最密充填構造が候補。他の配置がありうるかは知らん。

球の半径と同程度のスケールでみても一様ということなら、そんな配置は無理ぽ
876ご冗談でしょう?名無しさん:2009/04/17(金) 03:20:00 ID:n8wDNhos
フェムト秒とかピコ秒パルスとかってありますよね
キロ秒とか長い方はできるんでしょうか
理論上どういう合成になれば可能だと思いますか
877ご冗談でしょう?名無しさん:2009/04/17(金) 04:46:19 ID:???
長いパルスならふつうに手でSWを操作すれば作れるじゃん。
878ご冗談でしょう?名無しさん:2009/04/17(金) 07:54:22 ID:???
やっぱ馬鹿すぎて>>803はスルーされてる><
879ご冗談でしょう?名無しさん:2009/04/17(金) 09:29:56 ID:???
自分の置きたい設定に従って運動方程式立てて解けばいいだけじゃん
運動方程式立ててからわからないところ聞けば?
880ご冗談でしょう?名無しさん:2009/04/17(金) 09:35:51 ID:???
>>803
物体の表面にかかる圧力の和が、浮力となって現れる
簡単な例として、密度が一様な流体中の直方体に働く浮力の計算は、高校物理で習う

丸い物体や、場所によって流体の密度が変化する場合は
計算が面倒になるが、本質は変わらない

原理的に、浮力によって宇宙へは行けない
船が水面を離れられないのと同じ
881ご冗談でしょう?名無しさん:2009/04/17(金) 09:52:48 ID:???
一般の形の物体の浮力は、ベクトル解析の定理を使わないと計算できません。
882ご冗談でしょう?名無しさん:2009/04/17(金) 10:04:15 ID:???
>>880>>881
難しいこと言ってるが、積分しちまえば「浮力=物体が排除した流体の質量×重力加速度」。

これは密度が不均一でも一緒。ただし、静的な状態下で。
以上、アルキメデスの原理。
883ご冗談でしょう?名無しさん:2009/04/17(金) 10:21:40 ID:???
>>881
>一般の形の物体の浮力は、ベクトル解析の定理を使わないと計算できません。
アルキメデスの時代にベクトル解析ってあったの?
884ご冗談でしょう?名無しさん:2009/04/17(金) 10:36:27 ID:???
>>882
そりゃそうだが、それじゃ質問への答えにならないじゃん
885ご冗談でしょう?名無しさん:2009/04/17(金) 11:05:37 ID:???
882が答えてない質問って何?
886ご冗談でしょう?名無しさん:2009/04/17(金) 11:32:56 ID:???
>>884
計算方法は示してるし、宇宙へ行けるかどうかは>>880が答えている。
不足ないだろ。
887ご冗談でしょう?名無しさん:2009/04/17(金) 11:59:58 ID:???
>>883
静止した流体内に一般の形の閉じた面を仮想的に設置してみる。
その仮想面に囲まれた部分の流体は動かないから、面内の流体の重さと、
面外の流体からかかる力(すなわち浮力)がつり合っていなくてはならない。
ベクトル解析を知らなくてもアルキメデスの原理が成り立つことが導かれる。
888876:2009/04/17(金) 12:04:57 ID:n8wDNhos
光の波長を伸ばしたいんですけど
889ご冗談でしょう?名無しさん:2009/04/17(金) 12:05:22 ID:a/PUWaSz
電荷Q1とQ2は普通に足せるんですか?
890ご冗談でしょう?名無しさん:2009/04/17(金) 12:33:49 ID:???
>>888
> 光の波長を伸ばしたいんですけど

(1) 光速に近い速度(目標に応じてお好きに)で遠ざける。
(2) 重い天体(目標に応じてお好きに)の表面に置く。
(3) 遠方(目標に応じてお好きに)に置く。

>>889
電荷保存則に従えば、普通に足せるな。
891ご冗談でしょう?名無しさん:2009/04/17(金) 12:51:22 ID:???
乾電池を買ってきます
ほっときます
何十年か後、たぶん+極の電位はゼロになります

ここでプラス極は周期が100年くらいの電磁波の4分の1周期くらいのパルスを出したことになります
892ご冗談でしょう?名無しさん:2009/04/17(金) 13:49:11 ID:???
>>876
念のため言っとくが、フェムト秒パルスとはパルス幅がフェムト秒ということであって
波長がフェムト秒×光速というわけでは必ずしもないぞ
893ご冗談でしょう?名無しさん:2009/04/17(金) 14:19:44 ID:???
核の人はフェルミって呼んでたな
894ご冗談でしょう?名無しさん:2009/04/17(金) 14:25:08 ID:???
日本人ならユカワと呼ぶのが正しい。
895ご冗談でしょう?名無しさん:2009/04/17(金) 14:29:41 ID:???
日本人は人工物に人名を付けるのを好まない。
モノじゃないけど、単位も違和感ありまくり。
896ご冗談でしょう?名無しさん:2009/04/17(金) 14:38:17 ID:???
単に日本語の母音や子音が少なく日本語自体の発音が音域が狭くダサく聞こえるからだろう。
英語はその点、単語の発音一つ一つが洗練されており正に聞かせて魅了させるかの言語。
その違いだろう。
897ご冗談でしょう?名無しさん:2009/04/17(金) 14:42:16 ID:???
J-POPがダサく聞こえるのも日本語自体に問題があるだよなぁ〜
898ご冗談でしょう?名無しさん:2009/04/17(金) 14:47:09 ID:???
>>896 あんた大丈夫か?

「人名を」って書いたんだぞ。「日本語を」じゃねえ。
899ご冗談でしょう?名無しさん:2009/04/17(金) 15:03:22 ID:???
同じことでしょ。
単語であれ人名であれ日本語の発音はあまり美しく聞かせるような発達はしてきていないし、
現に単位や星の名前に日本人の名前やその発音では違和感があるでしょ?
なぜ違和感があるのかというと、英語と比較した時に聞こえが美しくないから。
簡単なこと。
900ご冗談でしょう?名無しさん:2009/04/17(金) 15:04:55 ID:n8wDNhos
合成波でピコ秒周期になる部分を作ったりする技術でしょ
おいおい知らないで答えるなよ
901ご冗談でしょう?名無しさん:2009/04/17(金) 15:48:31 ID:???
>>895
ボルト、アンペア、ワット、オーム、ファラッド なんかも違和感ある?
902ご冗談でしょう?名無しさん:2009/04/17(金) 16:03:35 ID:???
>>901
これはしたり。「日本人の人名を」と書くべきだった。
903ご冗談でしょう?名無しさん:2009/04/17(金) 16:06:15 ID:???
904ご冗談でしょう?名無しさん:2009/04/17(金) 16:20:32 ID:???
フジタ・スケール
905ご冗談でしょう?名無しさん:2009/04/17(金) 16:55:24 ID:???
>>896
やけに英語べた褒めだけど、フランス人が英語を評価するときは
美しくないと酷評されてるよ。

人名に限らず日本語として意味を取れてしまう単語が、
その意味とは関係ないところで現れるから違和感を感じる。
慣れてしまえばどうということはない。ユカワポテンシャルとか。
906ご冗談でしょう?名無しさん:2009/04/17(金) 16:56:40 ID:???
そう悲観すること無いんじゃん。
海外で使われている、HENTAI、BUKAKE
っといった日本発の素晴らしい単位もあるし。
907ご冗談でしょう?名無しさん:2009/04/17(金) 17:44:48 ID:???
>>906は 50HENTAI 位?
908ご冗談でしょう?名無しさん:2009/04/17(金) 18:35:18 ID:???
水中音響装置における後方拡散の強度ってなんなの?
909ご冗談でしょう?名無しさん:2009/04/17(金) 18:42:09 ID:???
>>908
板ちがい
910ご冗談でしょう?名無しさん:2009/04/17(金) 20:02:28 ID:???
>>907

906=100゚H

゚H(Do_Hentai)
911ご冗談でしょう?名無しさん:2009/04/17(金) 21:01:44 ID:/UImgs5o
セグウェイ考案した人がものすごく効率のよい飲料水産生用の蒸留装置を作ったという記事をどこかで見かけましたが、
詳しい図解みたいなのはとりあえず見当たりませんでした

ところで減圧すると水の沸点が下がるということなので、
普通に抱えられるくらいの密閉蓋付きの甕を繋いだみたいな代物で、
自転車空気入れみたいなのを逆に繋いで内部を減圧するとか、あるいは沸騰初期の蒸気で空気を追い出すとかして
ほとんど熱源なしか、せいぜい焚き火のトロ火にかけておくくらいで
大人一人が一日生きていけるくらいの蒸留水を作り出すブツができれば大地震でも戦争でも安心だと思いますが、
そういうものってできそうにないのでしょうか?

産油国の巨大プラントでもない限りあんまり世にそういう(ポータブルな)ものが無いことからすると、
どんなに効率よく作っても、5リットルの浄水を得ようとしたらどうしても風呂焚き一週間分くらいの薪で丸一日がかりとか?
せいぜい飯炊き程度の燃料では絶対無理っぽいのか?

また、沸騰が始まれば甕の中は蒸気でパンパンになるけどそれがあらたな気化を抑えるというか最初に減圧したのは何のため?
ってなりませんか?凝集部の冷却が十分なら負圧が保たれるってことか? さっぱりわからないけど

あさってあたり図書館行って子供の理科の本からべんきょうしてみようとは思うものの、
ちょっとそのまえによかったらざっくりとした見当をおしえてください。
912ご冗談でしょう?名無しさん:2009/04/17(金) 21:13:00 ID:???
>>911
沸点を下げたところで、水を水蒸気にするための気化熱はそう変わらない。
つまり蒸溜方式では、沸点を下げてもエネルギーは必要。

濾過方式が簡単でイイ。複数フィルターを使って、最後に浄水器で
使ってるような分子フィルターを使えば雑菌まで取り除ける。

産油国の巨大プラントでも最近は海水から逆浸透方式で水を得る方式が
主流。
913ご冗談でしょう?名無しさん:2009/04/17(金) 21:49:07 ID:u1B2g9Ob
ホログラフィック宇宙を語るときブラックホールが出てくるのは何故ですか?
文系に分かるように教えてください。
914ご冗談でしょう?名無しさん:2009/04/17(金) 22:17:01 ID:gsDfffib
いやすいませんが濾過方式は目的は同じでもまた話が別ということで。
減圧しても必要な熱量はあまり変わらないというのは(理屈はよくわかってないけど)わかりました。どうもありがとう。

見当を、というのは…たとえば、
売り物の「卓上型蒸留水製造装置 WS-400型」のスペックを見るとAC200V 13A で毎時3.5ℓとか。
電気代で言えば一日5ℓ採るのに3kwhで月1500円、十分生きるには生きていけるようですが

このエネルギーをカマドの薪とか灯油ランプに換算すると、どのくらいの燃料の量なのだろうか?
売り物の機械には効率でかなわない分、話三分の一としてその三倍くらいの量の燃料があればおナベとか甕とか組み合わせた代物でも乾き死にはまぬかれますよね。
あるいはなんか多段式とか排熱をどっかに戻すとかそういう難しいことをしないと三分の一どころじゃないのかな
「そんなの物理分野の問題じゃない」と言われたらそれは仕方ないですけども
915ご冗談でしょう?名無しさん:2009/04/17(金) 22:23:11 ID:???
チェレンコフ光についてなんだが、
荷電粒子が光の位相速度越えたら放射ってどういうこと?
衝撃波乱流とかの不連続面はある種の相転移と聞いたが
位相速度が臨界核の成長に何らかの指標を与える感じのモデルになるのか?
不連続面の両側の真空の差が輻射?
916ご冗談でしょう?名無しさん:2009/04/17(金) 22:30:20 ID:???
Wikipediaの「チェレンコフ放射」でも読めば?
917ご冗談でしょう?名無しさん:2009/04/17(金) 22:30:45 ID:???
>>913
>ホログラフィック宇宙を語るときブラックホールが出てくるのは何故ですか?
>文系に分かるように教えてください。
ホログラフィック宇宙=妄想理論だから、
ブラックホール=妄想が出てきて不思議じゃない。
918ご冗談でしょう?名無しさん:2009/04/17(金) 22:32:08 ID:LT29xYEZ
数学電卓の操作方法がよくわかる、マニュアルを教えてください。
919ご冗談でしょう?名無しさん:2009/04/17(金) 22:33:44 ID:???

920ご冗談でしょう?名無しさん:2009/04/17(金) 23:15:26 ID:LT29xYEZ
>>918より使い勝手のいい数学電卓も教えてください。
921ご冗談でしょう?名無しさん:2009/04/17(金) 23:21:05 ID:???
質問者はきちんと質問意図の判る文章で質問してください。
922ご冗談でしょう?名無しさん:2009/04/17(金) 23:24:16 ID:???
文系君は物理・科学の基礎がないから、
奇抜な最新科学と称するトンデモ理論
に簡単に騙されちゃうんだよね。
923ご冗談でしょう?名無しさん:2009/04/17(金) 23:29:31 ID:???
クロノセイド
924ご冗談でしょう?名無しさん:2009/04/17(金) 23:32:14 ID:???
数学電卓ってなんですか?
グーグル先生も教えてくれないんですけど
925ご冗談でしょう?名無しさん:2009/04/17(金) 23:35:34 ID:???
926ご冗談でしょう?名無しさん:2009/04/17(金) 23:35:43 ID:???
皆さん、馬鹿な質問にお付き合いいただきありがとうございます。

http://www.isas.ac.jp/publications/hokokuSP/hokokuSP44/15-25.pdf
このような高高度を飛べる気球があったから、どうしてこれ以上は飛べないのか、
空気が薄すぎて浮力が得られないのか、それとも成層圏の高い気温で
風船とかが割れてしまって無理なのかが気になって計算してみようと思ったのですが、
私の馬鹿な頭では無理でした。

素人が宇宙空間に物を飛ばすのはやはり無理なんですね。
927ご冗談でしょう?名無しさん:2009/04/17(金) 23:36:51 ID:???
関数グラフ付き電卓のことでしょ。
CASIOの15,000円程度のでいいんじゃない?
928ご冗談でしょう?名無しさん:2009/04/18(土) 00:20:26 ID:???
>>926
気球で行けるんなら大気があるってことだろう
929ご冗談でしょう?名無しさん:2009/04/18(土) 00:36:43 ID:???
いまだにFX-602Pが現役
930ご冗談でしょう?名無しさん:2009/04/18(土) 00:45:59 ID:MczkNsIs
2ちゃんねる初心者なんですが、メスコジって名前をよく見掛けます。意味とかあるんですか?
931ご冗談でしょう?名無しさん:2009/04/18(土) 00:57:21 ID:???
>>930
俺は見かけたことないよ。それで合ってるか?
932ご冗談でしょう?名無しさん:2009/04/18(土) 01:00:28 ID:5+zHXF+j
サルコジのいとこのことだよ。(嘘
933ご冗談でしょう?名無しさん:2009/04/18(土) 10:16:30 ID:???
今年は、野菜が中級のものしかできない。困ったものだ。コロニーか、宇宙船作らないか?
934ご冗談でしょう?名無しさん:2009/04/18(土) 10:21:19 ID:???
>>929
うらやましいな。私の 502Pはもう17回忌過ぎたよ。
I/Oに載ってたプログラムをよく打ち込んでいたものだ。
935ご冗談でしょう?名無しさん:2009/04/18(土) 10:51:53 ID:???
新社会人は、株のプログラム作っている人が居るが、ノーベル経済学賞を取った人でも作れないのに、買わないように。日本語ヘタクソですみません。
936ご冗談でしょう?名無しさん:2009/04/18(土) 12:30:38 ID:???
日本語がヘタとかいうレベルか?
937ご冗談でしょう?名無しさん:2009/04/18(土) 12:40:59 ID:???
翻訳すると

株で儲けを出すプログラムというものが市販されているが、
そんなものはノーベル賞級の経済学者でも作ることはできない。
急に金が入って使い道が分からない新社会人も多いだろうが、そんなものに手を出すべきではない。
938ご冗談でしょう?名無しさん:2009/04/18(土) 12:48:40 ID:???
>>937はすごいな。合点がいった。

>>935本人かとも思ったが、だったらはなから書くだろうから違うんだろう。
939ご冗談でしょう?名無しさん:2009/04/18(土) 12:51:01 ID:???
でも残念ながらスレ違い
940ご冗談でしょう?名無しさん:2009/04/18(土) 13:22:22 ID:???
株で儲けられるソフトがある
株で儲けられるはずである
しかしながらソフトで収入を得ることを考えている
何故か
単純
941ご冗談でしょう?名無しさん:2009/04/18(土) 13:28:32 ID:0bqzLOVC
エヴァっていまみるとすごく恥ずかしいわ
942ご冗談でしょう?名無しさん:2009/04/18(土) 13:35:03 ID:???
今日、日本は被爆した。アメリカの混合核が、上空で爆発したみたい。重力計か放射能測定器で、測ってみて。トップシークレットだから、まだ情報は未確認だけど。一応ね。
943ご冗談でしょう?名無しさん:2009/04/18(土) 14:02:11 ID:???
他の地域は、拡散核、分散核でいったみたい。
944ご冗談でしょう?名無しさん:2009/04/18(土) 14:06:22 ID:???
>>940
「もちろんこの素晴らしいソフトを皆さんに知っていただきたいからですよ」
945ご冗談でしょう?名無しさん:2009/04/18(土) 15:25:33 ID:JQaUPlGM
相対論から、情報が光速を越えて伝わらないことが言われますが
それは相対性原理と光速普遍からどうやって導かれるんですか?
946ご冗談でしょう?名無しさん:2009/04/18(土) 15:39:54 ID:rbg3iJ7w
この前大学でフェルミって人の話を聞いたんです。
興味がわいたのでGoogle検索してみました。
そしたら一番初めに
http://www.web-giga.com/fermi/
とか出てきて。フェルミ氏って今日本でアニメとか作ってるんですか?
947ご冗談でしょう?名無しさん:2009/04/18(土) 15:52:59 ID:???
ツマンネ、0点
948ご冗談でしょう?名無しさん:2009/04/18(土) 16:00:50 ID:???
>945
少なくとも正の質量を持った粒子は、相対性理論のというよりローレンツ変換の
帰結から光速を越えて加速させようとしても、光速度を持った上記の粒子の
運動エネルギーは無限になるため、有限のエネルギーをいくら込めても無理。
ちなみに、光子など光速で動ける粒子は質量を持たない。
また、今のところトンデモ理論な負の質量を持った粒子は発見されていない。

したがって、情報を伝える粒子が光速を越えて移動できない以上、情報は光速
を越えて伝わらない。
949ご冗談でしょう?名無しさん:2009/04/18(土) 16:22:07 ID:???
>>948
> ちなみに、光子など光速で動ける粒子は質量を持たない。

ニュートリノはー?

> また、今のところトンデモ理論な負の質量を持った粒子は発見されていない。

正と負てーか、実と虚だろ。
950ご冗談でしょう?名無しさん:2009/04/18(土) 16:35:07 ID:???
虚数だったか、スマソ。

ニュートリノは質量が発見された=光速ではなかった、ということだと思っていた。
ニュートリノが光速ってどっかで検証されていたのか。
951ご冗談でしょう?名無しさん:2009/04/18(土) 16:47:16 ID:???
ニュートリノの速度を調べてみた。
SN 1987A 観測結果からして、光速に極めて近い速度を維持しているのは
間違い無いんだな。16万光年をほぼ同一の時間で踏破したから。
そこからの結論は、ニュートリノの質量は極めて0に近いとされているけど。
952ご冗談でしょう?名無しさん:2009/04/18(土) 17:56:25 ID:???
ニュートリノ振動の結果から言えることは、世代の違うニュートリノの
質量の差が0ではないということなので、最も軽いニュートリノの質量が
0である可能性はまだ排除されていない。
953ご冗談でしょう?名無しさん:2009/04/18(土) 18:32:46 ID:???
ぶったまげるけどな
954ご冗談でしょう?名無しさん:2009/04/18(土) 19:03:43 ID:???
そもそもニュートリノって光の通った後にできる波紋みたいなものだらら
質量が0ってのは当然のこと。つまり波や風自体に質量が無いのと同じ。
955ご冗談でしょう?名無しさん:2009/04/18(土) 19:26:10 ID:???
なるほどよくわかりましたありがとう
956ご冗談でしょう?名無しさん:2009/04/18(土) 19:28:25 ID:???
>>954
なにその妄想
957ご冗談でしょう?名無しさん:2009/04/18(土) 19:42:19 ID:???
思考盗聴って統合失調症の前触れですかね?
958ご冗談でしょう?名無しさん:2009/04/18(土) 19:51:22 ID:???
オーケー板違いキチガイ
959ご冗談でしょう?名無しさん:2009/04/18(土) 20:15:53 ID:???
ハミルトンの運動方程式が
dx/dt=(1/μ)(Px+eAx)
dPx/dt=(-1/μ)*(Py+eAy)deAy/dx
と書けるとき
d^2/dx^2ってどのように書けますか?

μd^2/dt^2=dPx/dt-eB/2*dy/dt=-eBdy/dtとなっているのですが
真ん中の式の展開が分かりません。

磁束密度Bは一様な静二場で、Ax=-By/2 , Ay=Bx/2 としています。
960次スレテンプレです。誰かお願いします:2009/04/18(土) 20:25:49 ID:???
■ちょっとした疑問や質問はここに書いてね108■
http://science6.2ch.net/test/read.cgi/sci/1237466921/

質問する前に
教科書や参考書をよく読もう
http://www.google.com/ などの検索サイトを利用し、各自で調べること
宿題を聞くときは、どこまでやってみてどこが分からないのかを書くこと。
丸投げはダメだからね
(丸投げ君は完全無視。答えるだけ無駄。)

質問に対する返答には、何かしらの返答ちょうだいね
★書き込む前に>>2の注意事項を読んでね
★数式の書き方(参考)はこちら>>3
(予備リンク:>>2-10
荒らし厳禁、煽りは黙殺、忘れないうちに定期age
単発質問スレを発見したらこのスレッドへの誘導をよろしくね。

定番FAQ
英語最新版
http://math.ucr.edu/home/baez/physics/
旧版日本語訳
http://research.kek.jp/people/morita/phys-faq/
他にも参考にあるサイトなどあればぜひ。
例)http://ja.wikipedia.org/
※wikipedia内の説明はすべてが正確なわけではありません。
このスレでの受け答えもそうですが。相互に補完しつつ精度を高めましょう。
961ご冗談でしょう?名無しさん:2009/04/18(土) 21:27:50 ID:???
>>959
> d^2/dx^2って
二階微分演算子なら運動方程式関係なくそのままだろう
962ご冗談でしょう?名無しさん:2009/04/18(土) 21:36:44 ID:???
次スレ立てました
■ちょっとした疑問や質問はここに書いてね109■http://science6.2ch.net/test/read.cgi/sci/1240057473/

963ご冗談でしょう?名無しさん:2009/04/18(土) 22:04:48 ID:???
カノニカルでは、系のハミルトニアンから分配関数を計算して
自由エネルギーFを媒介して、さまざまな熱力学量を計算しますが、
グランドカノニカルでは、熱力学ポテンシャルをそのままFと置き換えて考えれば良いのでしょうか?
教科書をみても、熱力学ポテンシャルって載ってなくて・・・。
964ご冗談でしょう?名無しさん:2009/04/18(土) 22:06:22 ID:???
そんな教科書は捨てよ。
965ご冗談でしょう?名無しさん:2009/04/18(土) 22:08:48 ID:???
>>964
キッテルなのですが・・・。
カノニカルのときはFからエントロピーとか、比熱とか色々求めていたんですけど
グランドカノニカルだと平均粒子数のことしか書いていません。
他の教科書でも平均粒子数のことばかり記述されているのですが、
この違いは何なのでしょうか?
966ご冗談でしょう?名無しさん:2009/04/18(土) 22:11:20 ID:???
>>963
>平均粒子数のことしか書いていません
グランドカノニカルだから。
967ご冗談でしょう?名無しさん:2009/04/18(土) 22:11:24 ID:???
>>961
詳しくお願いします!
968ご冗談でしょう?名無しさん:2009/04/18(土) 22:19:19 ID:???
>>963
分配関数をZとおくと、F=kTlogZでHelmholtzエネルギーが定義できたように、
大分配関数をΞとおくと、J=kTlogΞでグランドポテンシャル(と通常呼ばれる熱力学ポテンシャル)が定義できる
ZとΞの表式を考えればFとJの間に関係式を立てることができるでしょ
JはFをLegendre変換しているだけだということが分かるから、
そこからエントロピーとか比熱とか求める関係式も容易に出せる

Jの熱力学的素性を使うのが分かりにくかったら、
グランドカノニカル分布では熱浴・粒子浴を使った代わりに、
熱浴・「圧力浴」(要するに可動壁)を使った時の分布(T-p分布)を使って分配関数Yを出して、
試しにG=kTlogYを計算して同様にFとの関係を考えればよい
969ご冗談でしょう?名無しさん:2009/04/18(土) 22:24:22 ID:???
あーごめん、>>968の熱力学ポテンシャルの定義式は適当に符号補完して
970ご冗談でしょう?名無しさん:2009/04/18(土) 22:30:37 ID:???
>>966
グランドカノニカルでは、大分配関数からエントロピーと求めたり内部エネルギーを求めたり・・・
ってのは重要な問題ではないのですか?

>>968-969
分かりやすい解説ありがとうございます。
971ご冗談でしょう?名無しさん:2009/04/18(土) 22:32:45 ID:???
>>970
求めたければ、求めればよろしい。
教科書なんてものは制限があるから全部書いてるわけではない。
いちいち教科書と相談しないといけないようでは、研究者には向いていないな。
972ご冗談でしょう?名無しさん:2009/04/18(土) 22:36:47 ID:???
グラカノなんて使ったことが無い
973ご冗談でしょう?名無しさん:2009/04/18(土) 22:45:12 ID:???
>>967
「xで二階微分する」という操作そのものは同じ記号でいいでしょ。何を微分するかはその後に書くわけで。ちなみに >>959
> μd^2/dt^2=dPx/dt-eB/2*dy/dt=-eBdy/dt
は時間で微分するという操作と何らかの量が等しいという支離滅裂な式に見える。ミスプリか写し間違いかと。
974ご冗談でしょう?名無しさん:2009/04/18(土) 22:59:09 ID:???
>973
なるほど。

ですよね〜。
写し間違えはないので、ミスプリかなーって思ったんですが
右辺の量は合ってるっぽいんですよ。
だから全然分からなくて・・・。
975ご冗談でしょう?名無しさん:2009/04/18(土) 23:01:34 ID:???
>>972
まあ凝縮系だけやってれば使わないかもね
一応Fermi分布・Bose分布の導出にはグランドカノニカル分布を使うけど
976ご冗談でしょう?名無しさん:2009/04/18(土) 23:33:45 ID:???
センター英語で150点で、大学に入ってから全く英語勉強してないんですけど
3ヶ月くらいで洋書をスラスラ読めるようになりますか?
院試の勉強をかねて、洋書を読み出したんですけど
辞書片手に読んでも4ページに一時間くらいかかってしまいました。
977ご冗談でしょう?名無しさん:2009/04/18(土) 23:35:39 ID:???
数学の英語は簡単だか物理の英語は結構難しい。
978ご冗談でしょう?名無しさん:2009/04/18(土) 23:35:56 ID:???
物理の本なら難しい文ないしそう難しくはないはず。とりあえず電子辞書かweb辞書を使え。
979ご冗談でしょう?名無しさん:2009/04/18(土) 23:39:33 ID:???
>>977-978
どっちなんですか?
3ヶ月くらいで院試にも通用するようになるでしょうか?
もちろん電子辞書は使ってますし、文法で悩むこともあまりないんですが
専門用語と物理特有の言い回しみたいなので時間がかかってしまい、
読み終わった後はあまり内容が頭に入ってきませんでした。
980ご冗談でしょう?名無しさん:2009/04/18(土) 23:41:04 ID:???
英語が出来ない理系は死ぬ、ソースは俺
981ご冗談でしょう?名無しさん:2009/04/18(土) 23:43:00 ID:???
>>976
>センター英語で150点で
センターをウン十年まえに受けたことしか覚えてない僕に、
150点というのがいいのかわるいのかどんなものなのか教えてください><
982ご冗談でしょう?名無しさん:2009/04/18(土) 23:44:40 ID:???
センター150点だと語彙力がなくて文法もあやふやなレベル
983ご冗談でしょう?名無しさん:2009/04/18(土) 23:46:11 ID:???
>>976
教科書を4頁/時のペースで進めたら速いと思うが。
984ご冗談でしょう?名無しさん:2009/04/18(土) 23:46:17 ID:???
>>976
「洋書で勉強すると一石二鳥になって良い」なる意見はたびたび見るが、
個人的には虻蜂取らずになる可能性もかなりあると思うんだけど

和訳ある本なら、和訳読んだ後に原書読むのは英語の勉強になると思う

>>981
院に行くことを考えれば悪い部類じゃないかなあ
985ご冗談でしょう?名無しさん:2009/04/18(土) 23:47:05 ID:???
>>981
宮廷以上に入るには160点以上が条件です。
150点だと、英語は読めるけど、試験としてはあまりよくないレベルって感じです。
986ご冗談でしょう?名無しさん:2009/04/18(土) 23:51:13 ID:???
>>979
ファイマンを1冊、3ヶ月やり込め。
それでかなり違ってくる。
分からない単語は英辞朗で調べろ。
専門用語も結構載ってる。
物理特有の言い回しなんて存在しない。
普通の評論文程度だ。
987ご冗談でしょう?名無しさん:2009/04/18(土) 23:52:24 ID:???
理系英語なんざ、数学も物理も語彙さえ押さえちまえば楽勝だろ。

反復して覚えろよ。
988ご冗談でしょう?名無しさん:2009/04/18(土) 23:52:28 ID:???
なぜファインマン?
989ご冗談でしょう?名無しさん:2009/04/18(土) 23:55:06 ID:???
ファインマンは文は平易だし、テーマがわかりやすいので読みやすい。
990ご冗談でしょう?名無しさん:2009/04/18(土) 23:55:48 ID:???
英語の勉強にはならんが、日本人の英語の著書は分かりやすい。
991ご冗談でしょう?名無しさん:2009/04/18(土) 23:59:47 ID:???
>>990
それは単なる罰ゲームではないのか。
992ご冗談でしょう?名無しさん:2009/04/19(日) 00:14:38 ID:???
ハエが車の中の空中で静止していて
車が急発進したらハエはどうなりますか?
993ご冗談でしょう?名無しさん:2009/04/19(日) 00:16:41 ID:???
>>992
ハエの視点からすれば、後ろから風が吹き始めたと感じるでしょうね。
994ご冗談でしょう?名無しさん:2009/04/19(日) 00:21:41 ID:???
天体に重力が発生する仕組みを分かりやすく教えてください。
995ご冗談でしょう?名無しさん:2009/04/19(日) 00:22:57 ID:???
質量が大きいから万有引力も大きい。
終わり。
996ご冗談でしょう?名無しさん:2009/04/19(日) 00:23:39 ID:???
ちがう。
天体はグラビトン(重力子)を大量に放出・吸収する。
その結果重力が生じたように見えるわけ。
997ご冗談でしょう?名無しさん:2009/04/19(日) 00:25:46 ID:???
>>995-996
ありがとうございます
998ご冗談でしょう?名無しさん:2009/04/19(日) 00:29:03 ID:???
>>996
>天体はグラビトン(重力子)を大量に放出・吸収する。
妄想 乙
999ご冗談でしょう?名無しさん:2009/04/19(日) 00:29:30 ID:???
すみません、光速のロケットと光速のロケットが互いにすれ違うと
いったい相手のロケットは相対速度で光速を越えているようにみえるのでしょうか?
1000ご冗談でしょう?名無しさん:2009/04/19(日) 00:29:55 ID:???
光速で運動している観測者は、進行方向前後に空間を認識できない。
真横の平面上が自身が存在する空間となる。
二次元生物だな。w
10011001
このスレッドは1000を超えました。
もう書けないので、新しいスレッドを立ててくださいです。。。